Craniomaxillofacial

Pataasin ang iyong marka sa homework at exams ngayon gamit ang Quizwiz!

The correct response is Option C. Resection is the most appropriate management for the symptomatic lesion described in the photographs. Oropharyngeal tumors must always be biopsied to assess malignancy potential. If the tumor is then deemed to be benign, appropriate management is determined based on symptoms. This patient presented with a biopsy result and a sizable lesion that is still growing. Typically, an imaging study such as MRI is performed of the head and neck to assess the extent of the disease. For this type of isolated lesion that is ulcerating and interfering with eating, resection is the most appropriate treatment. Observation is not the best option, as the patient is symptomatic. Embolization is not indicated, as this is not a vascular structure. Sclerotherapy yields poor results with a microcystic lesion and would likely result in painful necrosis of the involved tissue. Radiation therapy is not a described treatment for benign lymphatic lesions. Carbon dioxide laser treatment has been described for these types of lesions, but it often requires multiple treatments and leaves a raw surface for healing following each treatment.

2013 A 13-year-old girl is brought to the office because of a mass of the tongue that has been growing for 10 years. The patient says she has recently noted ulcers from the mass, and bleeding occasionally occurs. Examination of a specimen obtained on biopsy shows a microcystic lymphatic malformation with microthromboses and chronic inflammation. Photographs are shown. Which of the following is the most appropriate management? A ) Embolization B ) Radiation therapy C ) Resection D ) Sclerotherapy E ) Observation only

The correct response is Option E. The CT scan shows an isolated medial wall fracture with entrapment of the medial rectus muscle. The transcaruncular approach can be used for isolated medial wall surgery or combined with a retroseptal transconjunctival approach to the orbital floor (with or without a lateral canthotomy). Although a bicoronal incision will give the necessary exposure to reduce the entrapment and repair the fracture, it has a higher morbidity associated with it, primarily in regards to blood loss and scarring. In this case, it would be excessive in regards to what is needed to expose and treat an isolated medial wall fracture. The bicoronal incision is more appropriate in complex cases where multiple fractures are involved or if access to a naso-orbital-ethmoid or frontal sinus fracture is needed. In this case, subciliary, orbital rim, or lateral brow incisions would not give adequate exposure to reduce the entrapped muscle and repair the fracture.

2013 A 14-year-old boy is brought to the emergency department after a fall. Physical examination shows ocular entrapment. A CT scan is shown. Surgical correction is planned. Which of the following is the most appropriate location for incision in this patient? A ) Bicoronal B ) Lateral brow C ) Orbital rim D ) Subciliary E ) Transcaruncular

The correct response is Option B. A recent article showed for the first time that children who underwent Le Fort III advancement had recurrence of their initial pathology due to minimal mid face sagittal growth, but with normal mandibular growth. This study displayed this, despite excellent early advancement and bony stability up to 1 year. After 5 years, the lack of mid face growth ultimately relegates these patients to at least another advancement surgery. Definitive orthognathic surgery is required following the completion of skeletal growth to improve maxillomandibular relationships and to achieve optimal occlusion. In multiple studies, the average age of children undergoing Le Fort III osteotomies was close to age 6 years and age was not an independent factor for recidivism. Finally, studies of distraction after a Le Fort III osteotomy show better advancement and may help to minimize the recurrence of pathology.

2013 A 16-year-old boy who successfully underwent Le Fort III advancement with bone grafting 6 years ago because of severe maxillomandibular disharmony comes to the office due to severe malocclusion. Physical examination shows an Angle class III malocclusion and severe mid face deficiency. Which of the following is the most likely explanation for the reappearance of this patient's condition? A ) Age during original surgery B ) Discrepancy in the growth rate of the operated mid face and the mandible C ) Lack of bony stability in the first postoperative year D ) Poor follow-up E ) Poorly performed orthognathic surgery

The correct response is Option C. Significant hemorrhage is uncommon in orthognathic surgery, but when it occurs, it is most likely secondary to the maxillary osteotomies. The vessels at risk with the maxillary osteotomy include the greater palatine vessels, maxillary artery, and pterygoid plexus. The incidence of significant hemorrhage with mandible osteotomies is rare. The vessels at risk include the inferior alveolar artery, facial artery, retromandibular vein, and the pterygoid venous vein. Distraction osteogenesis is associated with lower risk for bleeding than any of the open procedures.

2013 A 16-year-old girl with facial asymmetry secondary to hemifacial microsomia comes to the office for evaluation of orthognathic surgery. Which of the following procedures puts her at the highest risk for perioperative bleeding? A ) Bilateral sagittal split osteotomy B ) Distraction osteogenesis C ) Le Fort I osteotomy D ) Mandibular vertical ramus osteotomy E ) Osseous genioplasty

The correct response is Option D. Satisfactory lateral pharyngeal wall movement and sagittal or circular velopharyngeal closure patterns should be treated with a pharyngeal flap. A large posterior gap with coronal, circular, or bowtie patterns of closure and good velar elevation, but poor lateral wall motion, should be treated with a sphincter pharyngoplasty. A palatal lift is used in patients with adequate tissue, but poor control of coordination. An obturator is used to assist with closure when there is inadequate palatal tissue. Finally, intravelar veloplasty is used in unrepaired clefts or a submucous cleft with a small posterior gap on closure.

2013 A 6-year-old boy who underwent repair of cleft palate 5 years ago is brought to the clinic for follow-up. The patient is hypernasal. Nasoendoscopy shows good lateral pharyngeal wall motion and a sagittal closure pattern. Cephalometric analysis shows a posterior gap of 10 mm. Which of the following is the most appropriate management? A ) Intravelar veloplasty B ) Obturator C ) Palatal lift D ) Pharyngeal flap E ) Sphincter pharyngoplasty

The correct response is Option C. The classification scheme by Tessier is perhaps the most accepted and basic approach to describe these rare craniofacial clefts. As it only provides an anatomical description, others have further developed description schemes. Some have an embryonic emphasis to further characterize these disorders. These severe clefts can be associated with encephalocele, which if not repaired can be life-threatening. Nasal dermoids arise from contact between dura and ectoderm through the foramen cecum. These can be dangerous if there is intracranial extension. Heterotopic glial tissue is equivalent to the term glioma because these are generally not connected to the intracranial space and are treated as benign lesions. The Rathke pouch occurs during development when the stomodeum ectoderm invaginates toward the hypophysis and remnants are usually located in the nasopharynx as a cyst. An encephalocele is similar to a glioma but contains meninges and/or brain (encephalomeningocele) and can communicate with the ventricle (encephalomeningocystocele). Fifteen percent of these are intranasal. Resection of these without knowing about their intracranial communication can be disastrous. Left untreated, these lesions can significantly increase the risk of meningitis and be life-threatening.

2013 A 2-week-old male newborn is brought to the office for evaluation of median craniofacial dysraphia. His parents say that aside from the deformity, the child is thriving. Physical examination shows Tessier No. 0 and 14 clefts. A clinical photograph and a three-dimensional CT scan are shown. Which of the following additional findings is most likely in this patient? A ) Choanal atresia B ) Contact between the dura and ectoderm through the anterior fontanelle C ) Failure of closure of the foramen cecum D ) Heterotopic glial tissue E ) Rathke pouch cyst

The correct response is Option E. Bilateral lower eyelid colobomas are commonly found in patients with Treacher Collins syndrome (TCS). TCS is also known as mandibulofacial dysostosis, first and second branchial arch syndrome, and Franceschetti-Zwahlen-Klein syndrome. Edward Treacher Collins described the syndrome in 1900. It is autosomal dominant with variable penetrance and has an incidence of 1 in 7,000 live births. It has significant dysmorphology, which includes lower eyelid colobomas, clefted and hypoplastic zygomas, clefted lateral orbit, hypoplastic mandible, lateral canthal vertical dystopia, antimongolian palpebral fissure, ear deformities, long anterior sideburns, anterior open bite, cleft palate, and macrostomia. Crouzon, Apert, and Pfeiffer syndromes involve craniosynostosis, typically bicoronal. These patients also have underdevelopment of the mid face also. They do not have any eyelid abnormalities. Apert patients also may have a cleft palate and syndactyly of the hands and feet. Goldenhar syndrome involves epibulbar dermoids of one or both eyes and underdevelopment of one or both sides of the face. It is also known as hemifacial and bifacial microsomia. The soft tissue as well as the bone is hypoplastic.

2013 A 2-year-old girl is brought to the office because of the lower eyelid colobomas and malar deficiency shown in the photograph. Which of the following syndromes is most likely? A ) Apert B ) Crouzon C ) Goldenhar D ) Pfeiffer E ) Treacher Collins

The correct response is Option B. The patient described has Apert syndrome. This autosomal dominant syndrome is characterized by bicoronal craniosynostosis leading to turribrachycephaly, mid face hypoplasia, and complex hand and foot syndactyly. Patients with Crouzon syndrome, an autosomal dominant disorder, typically have craniosynostosis involving the coronal, sagittal, and lambdoid sutures, as well as turribrachycephaly. Other findings include mid face hypoplasia, exorbitism, and proptosis. The extremities are normal. Defects in the Fibroblast Growth Factor Receptor-2 (FGFR2) are found. 22q deletion syndrome, which has several presentations, including DiGeorge syndrome, velocardiofacial syndrome, and Shprintzen syndrome, is caused by the deletion of a small piece of chromosome 22. The deletion occurs near the middle of the chromosome at a location designated q11.2; i.e., on the long arm of one of the pairs of chromosome 22. Characteristic signs include congenital heart disease, cleft palate, learning disabilities, mild elongation of facial features, and mental illness in the teenage years. Mutations in the IRF6 gene cause van der Woude syndrome. Van der Woude syndrome is an autosomal dominant form of cleft lip and palate. Affected individuals usually have lip pits. Mutations in the TCOF1 gene cause Treacher Collins syndrome. The official name of this gene is "Treacher Collins-Franceschetti syndrome 1." Patients with Treacher Collins syndrome, or mandibular dysostosis, have hypoplasia of the zygoma, maxilla, and mandible, downward slanting of the palpebral fissures, colobomas of the lower eyelids, absence of eyelashes, and auricular defects.

2013 A 20-month-old girl is brought to the office for evaluation of mid face hypoplasia, craniosynostosis, and anomalies of the hands and feet. The most likely cause of these findings is a genetic error in which of the following? A ) 22q.11 B ) FGFR2 C ) IRF6 D ) TCOF1

The correct response is Option C. The middle third of a line drawn between the tragus and the middle of the upper lip defines the course of the parotid duct. The buccal and zygomatic branches of the facial nerve lie in close proximity to the parotid duct, which lies superficial to the masseter muscle. Deep penetrating trauma in this region is likely to injure all three of these structures. Evidence of injury to the zygomatic or buccal branch of the facial nerve with a central cheek laceration should raise concern for a parotid duct injury. The lacrimal sac is outside of the described zone of injury, as is the mandibular branch of the facial nerve. The pterygoid muscle is deep to the mandible and would be outside of the described zone of injury. The maxillary sinus and the zygomatic branch of the facial nerve could have been injured, but not in combination with the pterygoid muscle, which is outside the zone of injury. Concomitant injury of the zygomatic arch and zygomatic branch of the facial nerve is possible, but because this injury was full thickness in the central third, the orbicularis muscle would not have been involved.

2013 A 22-year-old man comes to the emergency department after he sustained a machete laceration of the left cheek extending from the tragus through the midpoint of the upper lip. The wound is full thickness along the central third. Examination shows left upper lip droop and flattening of the associated nasolabial fold. Which of the following structures were most likely injured? A ) Lacrimal sac, mandibular branch of the facial nerve, and pterygoid muscle B ) Maxillary sinus, zygomatic branch of the facial nerve, and pterygoid muscle C ) Parotid duct, buccal branch of the facial nerve, and masseter muscle D ) Zygomatic arch, zygomatic branch of the facial nerve, and orbicular muscle

The correct response is Option D. The patient described has cervical necrotizing fasciitis (CNF) that likely extends into the mediastinum. Pharyngeal and tonsillar infections, along with dental abscesses, are the most common sources of infection. Diabetes and other immunocompromised states are frequent comorbidities in this disease. Early presentation may be clinically indistinguishable from a superficial soft-tissue infection and, therefore, requires a high index of suspicion. Skin necrosis is often a late finding, as the infection begins in the fascial and deep tissues of the neck. CT scan is usually obtained to define the extent of the disease and may or may not show gas within the soft tissues. Patients are treated with broad-spectrum antibiotics and aggressive surgical debridement. Mastoiditis is an infection of the mastoid air cells usually arising in the setting of untreated otitis media. Mastoiditis can spread to surrounding structures, including the brain. Meningitis is an infection of the meninges that is associated with neck pain as well as fever, headache, and photophobia. It does not present with unilateral neck erythema and swelling. Parotitis is an infection of the parotid gland that usually arises in the setting of an obstructed parotid duct and is a very rare but potential cause of CNF. Sinusitis is not usually associated with CNF but may spread to the orbit, resulting in orbital cellulitis or orbital abscess, and, occasionally, cavernous sinus thrombosis. Cavernous sinus thrombosis is a life-threatening condition.

2013 A 22-year-old man who has type 1 diabetes mellitus comes to the office because of a 1-day history of cervical and chest pain. Temperature is 102.2°F (39.0°C). Physical examination shows right-sided cervical erythema and moderate swelling. A broad-spectrum antibiotic is administered, and he undergoes incision and drainage. During the procedure, extensive soft-tissue necrosis not confined by fascial planes is noted. Which of the following is the most likely cause of this infection? A ) Mastoiditis B ) Meningitis C ) Parotitis D ) Pharyngitis E ) Sinusitis

The correct response is Option D. The vignette illustrates findings consistent with a Marcus Gunn pupil. A lesion at the level of the posterior globe (retina) and optic nerve anterior to the chiasm alters the afferent response normally expected from shining a light in the eye. The test is done by shining a light in the pupil of the affected side first, and this will cause little or no constriction on that side, yet there will be a consensual response on the normal side. Then, the light is shined in the normal pupil, resulting in constriction of both pupils (normal ipsilateral and consensual response). The third step is to swing the light back to the injured side, and this typically will cause paradoxical dilatation of the affected pupil. A fracture in the posterior orbit may cause pressure on the optic nerve directly by impingement of a fracture fragment or from hematoma. Extraocular muscles control the directionality of the globe for the purpose of binocular vision. There is no direct distal anatomical connection to the pupillary sphincter muscle (parasympathetic) or the iris dilator (sympathetic), both of which control pupillary size. However, there is a proximal commonality in that some extraocular muscles (superior/medial/inferior recti, inferior oblique) and the pupillary muscles are supplied by the oculomotor nerve. Most commonly, it is the inferior oblique with or without the inferior rectus muscle that is entrapped in an orbital floor fracture, and thus has no direct bearing on pupillary size. Therefore, pupils should react normally in an isolated orbital floor fracture with muscle entrapment. Hyphema is the presence of blood in the anterior chamber. It does not cause a Marcus Gunn pupil, or any other abnormal pupillary reaction. The pupils in a patient with inadequate brain perfusion are usually dilated and fixed, or react sluggishly to light, but always symmetrical. In tentorial herniation, due to a laterally expanding intracranial hematoma, the oculomotor nerve is susceptible to compression as it exits the mid brain. The pupillary examination will show one ipsilateral dilated pupil that is fixed or sluggish, and the globe will be laterally displaced due to the unopposed tone generated by cranial nerve VI on the lateral rectus muscle over a paralyzed medial rectus muscle. Normal pupillary constriction as a reaction to light begins with a light stimulus on the retina that triggers a signal via afferent fibers in the optic (II) nerve. These fibers connect to the pretectal nucleus and both Edinger-Westphal nuclei. A response is generated from there through parasympathetic fibers via the oculomotor (III) nerve, which supplies the sphincter pupillae.

2013 A 23-year-old man is brought to the emergency department unconscious after a head-on motor vehicle collision. Physical examination shows left periorbital bruising. When a light is shined into the left eye, no pupillary constriction is noted, and thereafter, the light is shined into the right eye, and both pupils constrict. When the light is subsequently shined again into the left eye, the right pupil constricts and the left pupil dilates. Which of the following best explains this finding? A ) Extraocular muscle entrapment B ) Hyphema C ) Inadequate brain perfusion D ) Optic nerve injury E ) Tentorial herniation

The correct response is Option B. Benign masseteric hypertrophy may present as a bilateral or a unilateral condition. When unilateral, it is associated with repetitive unilateral clenching of the teeth. Both the masseter and temporalis muscles of the affected side show varying degrees of enlargement. When mild, medical management may be offered first and can include muscle relaxants, anxiolytics, antiepileptic drugs, and botulinum toxin type A. Surgical resection of a portion of the masseter and/or bone contouring are appropriate surgical procedures for correction of the resulting cosmetic deformity. Unilateral masseteric hypertrophy must be distinguished from unilateral condylar hyperplasia, the latter consisting of the enlargement or overgrowth of the mandibular condyle. Condylar hyperplasia may also present with unilateral facial enlargement (type IB or type II), deviation of the mandibular midpoint toward the unaffected side, class III occlusion on the ipsilateral side, and a crossbite on the contralateral side. Condylar resection is the mainstay of treatment. Condylar reduction is appropriate for cases of condylar dislocation. This condition can occur unilaterally in patients with a hypermobile or stretched temporomandibular joint, or in patients with dystonia (hyperfunction of the lateral pterygoid muscle). Condylar dislocation occurs suddenly, and causes pain and a class III occlusion on the involved side. Condylar reduction is performed with the aid of muscle relaxants. Eminence surgery may be necessary (eminectomy, eminoplasty). Orthodontics are unnecessary for someone with bruxism, which is a typical feature of masseteric hypertrophy. However, a mouth guard is appropriate. Radiation is inappropriate for benign masseteric hypertrophy. It does not correct the underlying cause or the deformity associated with this condition. Sagittal split osteotomy is indicated in cases of class II or III malocclusion in which reduction or lengthening of the anteroposterior length of the mandible will achieve dental harmony. This procedure is not appropriate for class I occlusion or flaring of the mandibular angle.

2013 A 25-year-old man comes to the office for consultation regarding a 10-year history of gradual swelling of the right side of the face. Physical examination shows class I occlusion, normal interincisal distance, and smooth occlusal surfaces on the right molar teeth. The right cheek is enlarged when he clenches the teeth. CT scan shows a right masseter that is twice as large as the left one, and there is an outward curvature of the angle of the mandible. Which of the following is the most appropriate treatment? A ) Condylar reduction B ) Masseter resection C ) Orthodontics D ) Radiation therapy E ) Sagittal split osteotomy

The correct response is Option C. This case described represents the classic presentation of a posttraumatic carotid-cavernous fistula (CCF). Symptoms include pulsatile proptosis, ocular and orbital erythema, chemosis, diplopia, headaches, and visual loss. This pathology occurs through abnormal connections between the internal carotid artery and the cavernous sinus, and, on rare occasions, may appear between the internal carotid or branches of the external carotid artery and the venous plexus of the skull. Traumatic CCF can lead to blindness as the disorder progresses and, in rare cases, can result in paralysis, unconsciousness, and even death. It is important that clinicians operating on the craniofacial region, and especially those who manage craniofacial trauma, have a thorough understanding of this potentially lethal entity. Although CCF occurs rarely after craniofacial trauma, this disorder is thought to occur relatively frequently in patients with basilar skull fractures. Once there is a suspicion, a prompt evaluation of the arterial vasculature around the cavernous sinus is required. Most commonly, a cerebral angiogram is used to make the diagnosis and, using related techniques, a variety of curative measures (e.g., embolization) can then be enacted. A craniotomy-type modality would be relegated to only the most refractory of cases and would be for cure and not diagnosis. A noncontrasted CT scan would only show posttraumatic bony derangements. Beta-2 transferrin assay could be used if there is an accompaniment of clear rhinorrhea to rule out a craniospinal fluid leak. CCF can be mistaken for other pathologies such as an orbital apex syndrome or even stroke. The latter suspicion may lead to the ordering of a carotid duplex, which would be expected to be normal.

2013 A 25-year-old man comes to the office for evaluation of unilateral proptosis, visual impairment, and limitation of ocular movements. History includes basilar skull fractures and repair of panfacial fractures 4 weeks ago. On physical examination, the left eye appears to be pulsating. Which of the following is the most appropriate next step? A ) Beta-2 transferrin assay B ) Carotid duplex C ) Cerebral angiography D ) Craniotomy E ) Noncontrast CT scan

The correct response is Option C. Mandible fractures are a frequent injury because of the mandible's prominence and relative lack of support. Numerous investigators have reported studies on populations on all continents; fractures of the mandible have been reported to account for 36 to 70% of all maxillofacial fractures. All reports apparently show a higher frequency in males aged 21 to 30 years. In patients with mandible fractures, 53% of patients had unilateral fractures, 37% of the patients had two fractures, and 9% had three or more fractures. Fractures sustained in altercations - low-energy fractures - tend to have single, simple patterns. The indications for closed versus open reduction have changed dramatically over the last century. The ability to treat fractures with open reduction and rigid internal fixation (ORIF) has dramatically revolutionized the approach to mandibular fractures. Traditionally, closed reduction (CR) and ORIF with wire osteosynthesis have required an average of 6 weeks of immobilization by maxillomandibular fixation for satisfactory healing. Difficulties associated with this extended period of immobilization include airway problems, poor nutrition, weight loss, poor hygiene, phonation difficulties, insomnia, social inconvenience, patient discomfort, work loss, and difficulty recovering normal range of jaw function. In contrast, rigid and semirigid fixation of mandible fractures allow early mobilization and restoration of jaw function, airway control, improved nutritional status, improved speech, better oral hygiene, patient comfort, and an earlier return to the workplace.

2013 A 25-year-old man comes to the office for treatment of malocclusion 6 days after he was involved in an altercation. Physical examination shows right-sided facial swelling. X-ray study shows an unfavorable fracture through the angle of the right mandible. No other associated injuries are noted. Which of the following is the most appropriate treatment? A ) Closed reduction and maxillomandibular fixation B ) Open reduction and maxillomandibular fixation C ) Open reduction and rigid fixation D ) Open reduction, wire fixation, and maxillomandibular fixation E ) Observation only

The correct response is Option B. A drooping eyelid is called ptosis or blepharoptosis. In ptosis, the upper eyelid falls to a position that is lower than normal. In severe cases of ptosis, the drooping eyelid can cover part of or the entire pupil and interfere with vision, resulting in derivational amblyopia. The eyelids are elevated by the contraction of the levator palpebrae superioris. In most cases of congenital ptosis, a droopy eyelid results from a localized myogenic dysgenesis. Rather than normal muscle fibers, fibrous and adipose tissues are present in the muscle belly, diminishing the ability of the levator to contract and relax. Therefore, the condition is commonly called congenital myogenic ptosis. Most cases of congenital ptosis are idiopathic. Surgical correction of congenital ptosis can be undertaken at any age depending on the severity of the disease. Earlier intervention may be required if significant amblyopia or ocular torticollis is present. If intervention is not urgent, surgery is often delayed until age 3 to 4 years. Waiting until this age allows for more accurate measurements preoperatively. Congenital ptosis can also occur when the innervation to the levator is interrupted through neurologic or neuromuscular junction dysfunction. Nerve compression by external forces such as tumor must be ruled out. Specifically, when ptosis presents acutely or subacutely in a child over 1 year of age, compression of cranial nerve III is a concern. Rhabdomyosarcoma is the most common primary malignancy of the orbit in children. MRI shows a well-circumscribed mass that typically enhances with gadolinium. On T1-weighted imaging, the tumor usually appears isointense to extraocular muscles but hypointense to orbital fat. On T2-weighted imaging, the lesion appears hyperintense to extraocular muscles and orbital fat. This tumor can grow rapidly and is treated with combined chemotherapy and radiation. A history of difference in the size of the pupil may be helpful in diagnosing Horner syndrome. Patients with Horner syndrome have ptosis and miosis on the same side. Cervical or apical thoracic tumors can cause damage to the sympathetic chain and result in this condition. Neuroblastoma, which is one of the most common childhood cancers, should be ruled out with this presentation. Intermittent patch therapy is indicated for strabismus to strengthen the weak rectus muscles.

2013 A 26-month-old boy is brought to the office for evaluation because of worsening congenital ptosis. A photograph is shown. Which of the following is the most appropriate next step in management? A ) Eye lubrication B ) MRI C ) Patching of the non-affected eye D ) Surgical correction E ) Observation only

The correct response is Option D. It is recommended that injury to the orifice of the parotid duct be managed with a stent or a drain to maintain patency during healing and guide the flow of saliva into the mouth. Allowing the ductal injury to heal secondarily will likely result in blockage of the orifice with scar tissue, which could lead to a sialocele or cutaneous fistula. Ligation of the proximal duct is recommended for extensive injuries of the glandular/duct system in which neither end of the duct is amenable to repair. This will result in eventual atrophy and loss of function of the parotid gland. Primary repair alone would not be recommended given the degree of soft-tissue loss and contusion with this mechanism of injury. Superficial parotidectomy would be considered for management of chronic glandular/duct system fistulas.

2013 A 27-year-old man is brought to the emergency department after he sustained a small-caliber gunshot wound through the cheek, obliterating the intraoral orifice of the parotid duct. Which of the following is the most appropriate management? A ) Ligation B ) Primary repair C ) Secondary intention healing D ) Stenting E ) Superficial parotidectomy

The correct response is Option C. The salivary gland neoplasms are uncommon and generally benign. Most benign tumors can be easily cured by wide local excision. Pleomorphic adenoma is the most common salivary gland tumor, with a propensity for local recurrence. Simple enucleation is discouraged. Removal is usually done by a superficial parotidectomy. The superficial lobe is anatomically defined by the traversing facial nerve. The nerve is preserved, as pleomorphic adenoma is a benign tumor that does not infiltrate the nerve. The parotid gland is also the most frequently affected major salivary gland, and the palatal salivary tissue is the most commonly affected minor salivary gland. Pleomorphic adenomas are the most common neoplasm of salivary glands, comprising 45 to 75% of all tumors in most series. These tumors typically affect patients in their 20s to 50s and there is a female predilection. Warthin's tumor is the second most common tumor of the salivary glands and constitutes approximately 14 to 21% of salivary gland neoplasms. The tumor is almost exclusively found in the parotid gland, typically affects males in their 50s to 60s, and often may be bilateral.

2013 A 32-year-old man comes to the office because of a pleomorphic adenoma in the right parotid. A mass is palpated over the angle of the mandible. Examination of the oral cavity and neck shows no abnormalities. Which of the following is the most appropriate surgical treatment? A ) Enucleation B ) Radical parotidectomy C ) Superficial parotidectomy D ) Total parotidectomy E ) Total parotidectomy and neck dissection

The correct response is Option A. A deformity upon facial animation characterized by descent of the nasal tip, shortening of the upper lip, and a transverse crease in the mid philtral area may be created or accentuated by the action of the depressor septi nasi muscles. These are small, paired muscles located on each side of the nasal septum, which originate at the medial crura foot plates and insert either on the incisive fossa of the maxilla or into the fibers of the orbicularis oris muscle. Physical examination upon facial animation should be part of the routine preoperative evaluation of the rhinoplasty patient. Those who present with the dynamic deformity as described may benefit from excision or transection of the depressor septi nasi muscles. Several surgical techniques have been described, as well as the use of botulinum toxin type A. The nasalis muscle compresses the cartilaginous part of the nose and draws the ala toward the septum. Although this may generate some depression of the tip of the nose, it should not cause shortening of the upper lip. The procerus muscle depresses the medial angle of the eyebrows, creating transverse rhytides over the bridge of the nose. The risorius muscles retract the angles of the mouth, as in a grinning expression. The zygomaticus major muscles draw the angles of the mouth posteriorly and superiorly, as in laughing. These muscles do not cause depression of the tip of the nose.

2013 A 32-year-old woman comes to the office for consultation regarding cosmetic improvement of her nose. On examination, facial animation (smiling) causes marked descent of the nasal tip, shortening of the upper lip, and a transverse crease in the mid philtral area. These findings are most consistent with the action of which of the following muscles? A ) Depressor septi nasi B ) Nasalis C ) Procerus D ) Risorius E ) Zygomaticus major

The correct response is Option E. Frontal sinus fractures can be assessed by patency of the nasofrontal duct and by whether the fracture involves the anterior wall of the sinus, the posterior wall, or both. In this case, CT scanning shows significant damage to the anterior wall and a minimal fracture in the posterior wall. The normal nasofrontal duct would allow salvage of the sinus with minimal risk of a mucocele or an infection. The underlying brain is uninvolved, so craniotomy is unnecessary. The obvious deformity indicates open reduction and fixation, and because the posterior wall is minimally involved, repair of the anterior wall without craniotomy could be performed.

2013 A 33-year-old man is brought to the emergency department after he is struck in the forehead with a baseball bat. The patient is awake and alert. Examination shows an obvious deformity in the frontal region. Emergency CT scan shows a displaced and comminuted fracture of the anterior wall of the frontal sinus and a nondisplaced fracture of the posterior wall. The frontonasal duct is patent. Which of the following is the most appropriate treatment? A ) Cranialization of the sinus B ) Craniotomy C ) Obliteration of the sinus and fat grafting D ) Observation and intravenous administration of antibiotics E ) Open reduction and internal fixation of the anterior wall of the frontal sinus

The correct response is Option D. The periapical cyst is the most common odontogenic cyst. The usual etiology is a nonviable tooth that becomes infected, leading to necrosis of the pulp. Toxins exit the apex of the tooth, leading to periapical inflammation leading to a radiolucency seen on x-ray study. The dentigerous cyst is the second most common odontogenic cyst which develops within the normal dental follicle surrounding an unerupted tooth such as a mandibular or maxillary third molar or maxillary canine. Most are asymptomatic and found incidentally on x-ray study. The odontogenic keratocyst or OKC is the third most common odontogenic cyst. It is a great mimic and can have a wide variety of clinical presentations. These cysts are rapid growing and aggressive and can be difficult to remove; recurrence rates are high. They are also a component of the basal cell nevus syndrome. Mucous retention cyst, or mucocele, is a pseudocyst that arises from trauma to the minor salivary glands in the lips. Traumatic bone cyst is a self-limiting radiolucent lesion of uncertain etiology that forms an empty or fluid-filled cavity most commonly within the mandible but also elsewhere in the body.

2013 A 35-year-old man is evaluated because of pain in one of the right lower molar teeth 6 months after he underwent open reduction and internal fixation of a right mandibular angle fracture. Examination shows grayish discoloration and tenderness to axial percussion in the mandibular second molar. A panoramic x-ray study (Panorex) shows an intact mandibular second molar with 1.5-cm radiolucency at its apex. Which of the following is the most likely cause of these findings? A ) Dentigerous cyst B ) Mucous retention cyst C ) Odontogenic keratocyst D ) Periapical cyst E ) Traumatic bone cyst

The correct response is Option B. The patient described has a persistent thyroglossal duct cyst. The thyroid gland is the first of the body's endocrine glands to develop, at approximately 24 days' gestation. The gland originates as a proliferation of endodermal epithelial cells on the median surface of the developing pharyngeal floor known as the foramen cecum. The foramen cecum originates from between the first and second pouches and represents the opening of the thyroglossal duct into the tongue. Descent of the thyroid gland carries it anterior to the hyoid bone and anterior to the laryngeal cartilages. As the thyroid gland descends, it forms its mature shape. The thyroid completes its descent in the seventh gestational week, coming to rest in its final location immediately anterior to the trachea. If the thyroglossal duct does not atrophy, then the remnant can manifest clinically as a thyroglossal duct cyst. While half of these generally midline cystic masses are located at or just below the level of the hyoid bone, they may be located and can track anywhere from the thyroid cartilage up the base of the tongue. Because the hyoid bone develops in an anterior direction and may surround the thyroglossal duct, the surgeon should resect the central portion on the hyoid bone along with the cyst. The thyroid gland is ensheathed by the visceral fascia, which attaches it firmly to the laryngoskeleton (i.e., Berry ligament). This firm attachment of the gland to the laryngoskeleton is responsible for movement of the thyroid gland and related structures during swallowing. This also causes a thyroglossal duct cyst to move on physical examination. Ectopic thyroid gland may occur anywhere along the path of initial descent of the thyroid, although it is most common at the base of the tongue, just posterior to the foramen cecum. Ectopic parathyroid glands occur in 15 to 20% of patients. The glands may be located anywhere near or even within the thyroid or thymus. For example, if parathyroid IVs do not descend entirely, they may be located as high as the bifurcation of the common carotid artery. Conversely, if parathyroid IVs do not release from the thymus, they may be located intrathoracically, as low as the aortopulmonary window. Other common ectopic locations include the anterior mediastinum, posterior mediastinum, and retroesophageal and prevertebral regions. However, even when the parathyroid glands are in an ectopic location, they still often are symmetrical from side to side, making localization somewhat easier. Ossification of cartilages from the second and third pharyngeal arches gives rise to the hyoid bone.

2013 A 4-year-old boy is brought for evaluation because his mother is concerned about a growth on his neck. Physical examination shows a nontender mass in the midline of the neck that moves vertically when the patient swallows. Which of the following is the most likely cause of this patient's condition? A ) Failure of vascular apoptosis at 12 weeks' gestation B ) Failure of the thyroglossal duct to atrophy C ) Ossification of cartilage from the second and third pharyngeal arches D ) Persistent ectopic parathyroid tissue in the neck E ) Persistent ectopic thymus tissue in the neck

The correct response is Option D. In comparison to adult fractures, pediatric fractures are treated more conservatively due to the stage of mixed dentition, the elasticity of the craniofacial skeleton, and the potential for remodeling of the bone and fracture site with growth. Consideration must be given to the effects of invasive surgery on growth of the jaw. Many surgeons believe that conservative management prevents further growth disturbance. Given the extent of fractures and displacement of the parasymphysial fracture, observation alone is not an option. Closed reduction of all of the fractures followed by maxillomandibular fixation (MMF) will not yield a stable result due to the unfavorable forces displacing the right hemi mandible. Open reduction and internal fixation (ORIF) of all fractures is not possible due to the intracapsular nature of the condylar fracture. The fragments are too small to internally fixate. ORIF of the displaced parasymphysial fracture and closed reduction of the condylar and ramal fractures is the most appropriate management. The occlusion is restored, and bone anchor screws are used to obtain MMF. Due to the child's dentition, arch bars are not an option. MMF can be accomplished with a custom splint and drop wires with circummandibular wires or with bone anchor screws. Care needs to be taken to place the screws to avoid the dentition. MMF is typically removed at 3 to 4 weeks in this age group to prevent any ankylosis of the temporomandibular joint. If a titanium plate is used to fixate the parasymphysial fracture, removal of this 2 to 3 months following surgery is also recommended to prevent any growth restriction and embedding of the plate.

2013 A 4-year-old girl is brought to the emergency department for evaluation of mandibular fractures 12 hours after being kicked in the face by a horse. Physical examination shows an anterior open bite and bilateral crossbites. CT scans are shown. Which of the following is the most appropriate management at this time? A ) Closed reduction of all mandibular fractures and maxillomandibular fixation B ) Open reduction and internal fixation of all mandibular fractures C ) Open reduction and internal fixation of the parasymphysial and condylar fractures, closed reduction of the ramal fracture, and maxillomandibular fixation D ) Open reduction and internal fixation of the parasymphysial fracture, closed reduction of the condylar and ramal fractures, and maxillomandibular fixation E ) Observation only

The correct response is Option E. Surgical exploration is most likely to show the ascending course of this pharyngeal fistula to be superficial to the stylopharyngeal muscle. Second pharyngeal cleft and pouch anomalies (including cysts, fistulas, and sinuses) account for 67 to 95% of the total anomalies of the pharyngeal apparatus. Cysts are the most common finding, occurring three times more often than fistulas. Fistulas usually present at birth. They derive from the ventral portion of the second pharyngeal cleft and pouch. The external opening is usually found along the anterior border of the sternocleidomastoid muscle, between the hyoid bone superiorly and the suprasternal notch inferiorly. Fistulas have a muscular coat, which is continuous superficially with the platysma and internally with the palatopharyngeal muscle. If this muscle coat is well developed, swallowing causes a pull on the fistulous opening, resulting in puckering. The anatomical relations between a second pharyngeal cleft and pouch fistula and the surrounding cervical structures are dictated by the embryogenesis of the pharyngeal apparatus. As an anomaly of the second cleft and pouch, the fistula is expected to course deeply to the second arch structures and superficially to the structures derived from the third to sixth arches. Therefore, the described fistula is expected to course deep to the stylohyoid muscle and posterior belly of the digastric muscle (derived from the second pharyngeal arch), and superficial to the internal carotid artery and stylopharyngeal muscle (derived from the third pharyngeal arch). The hypoglossal nerve and associated infrahyoid muscles do not develop in the mesenchyme of the pharyngeal apparatus, instead being derived from occipital somites in the paraxial mesoderm. All pharyngeal anomalies derived from ectoderm (e.g., fistulas) will be found superficial to the hypoglossal nerve and the infrahyoid strap muscles. Other cervical structures not mentioned in this scenario but which are relevant to the course of the described fistula include the external carotid artery (second pharyngeal arch) and the glossopharyngeal nerve (third pharyngeal arch). The expected fistula course is deep to the former and superficial to the latter.

2013 A 4-year-old girl is brought to the office because of a congenital mucous-draining skin sinus located on the lower third of the neck, overlying the anterior border of the left sternocleidomastoid muscle. On physical examination, swallowing causes noticeable puckering of the sinus. Intraoperative probing shows that it communicates with the left tonsillar fossa. On surgical exploration, which of the following is the most likely ascending course of this sinus? A ) Deep to the hypoglossal nerve B ) Deep to the internal carotid artery C ) Superficial to the posterior belly of the digastric muscle D ) Superficial to the stylohyoid muscle E ) Superficial to the stylopharyngeal muscle

The correct response is Option B. Blepharospasm is an involuntary eye movement disorder characterized by frequent blinking and squeezing of the eyelids. Blepharospasm is treated with botulinum toxin type A. Treatment usually brings some relief, although patients require constant monitoring and re-treatment with botulinum toxin type A. Use of tinted glasses may reduce a trigger of essential blepharospasm but is not the primary treatment. Benzodiazepines are not used as the first-line management of essential blepharospasm. Myectomy and neurectomy are reserved for severe cases that are refractory to medical management and are therefore not indicated unless the patient fails all other forms of management.

2013 A 45-year-old man comes to the office because of frequent blinking and squeezing of the eyelids. Examination shows idiopathic blepharospasm. Which of the following is the most appropriate management? A ) Administration of a benzodiazepine B ) Injection of botulinum toxin type A C ) Myectomy D ) Neurectomy E ) Use of tinted glasses

The correct response is Option D. Parry first described the syndrome of progressive facial atrophy in 1825, followed by Romberg in 1846. Parry-Romberg syndrome (also known as progressive hemifacial atrophy) is a rare neurocutaneous syndrome characterized by progressive shrinkage and degeneration of the tissues beneath the skin, usually on only one side of the face (hemifacial atrophy) but occasionally extending to other parts of the body. The syndrome often begins with a circumscribed patch of scleroderma in the frontal region of the scalp which is associated with a loss of hair and the appearance of a depressed linear scar extending down through the mid face on the affected side. This scar is referred to as a "coup de sabre," and is indistinguishable from the scar observed in frontal linear scleroderma. Symptoms and physical findings usually become apparent during the first or early during the second decade of life. The average age of onset is 9 years, and the majority of individuals experience symptoms before age 20 years. The disease may progress for several years before eventually going into remission. Most patients do not have severe enough disease to warrant immunosuppression. However, for those with more severe and progressive disease, treatments used include methotrexate (for which there is limited evidence in linear scleroderma), corticosteroids, cyclophosphamide, and azathioprine, but benefits are unclear. The timing of surgical intervention is generally agreed to be the best following exhaustion of the disease course and completion of facial growth. When Parry-Romberg syndrome is severe, the skeleton may be affected, and bony restoration may be required. Soft-tissue reconstruction in patients with severe disease may require free tissue transfer. It is much more common to begin reconstruction with autologous fat transfers. Off-the-shelf cutaneous fillers have also been employed. The use of highly active antiretroviral therapy with protease inhibitors can result in a syndrome of peripheral wasting, facial fat atrophy, and central adiposity in as many as 64% of patients who are HIV-positive who are treated with this regimen for 1 year. This is usually a bilateral process, but also responds to similar treatment options. Switching antiretroviral drugs may be effective but is only employed in those patients with complete viral suppression. Injections of poly-L-lactic acid (Sculptra) are now licensed for cosmetic management of facial lipoatrophy.

2013 A 45-year-old man comes to the office because of slowly progressive unilateral loss of facial volume on the right side that began 30 years ago. Physical examination shows facial asymmetry and a line of cutaneous sclerosis on the forehead. No abnormalities in ocular mobility and visual acuity are noted, and no malocclusion is present. Which of the following is the most appropriate management? A ) Change of antiretroviral medications B ) Immunosuppression with oral administration of corticosteroids and methotrexate C ) Restoration of skeletal support with bone grafting D ) Soft-tissue augmentation with fat grafting E ) Observation only

The correct response is Option E. The most important mechanism by which apraclonidine caused improvement of this patient's eyelid ptosis as described is stimulation of alpha-adrenergic receptors in the superior tarsal (Müller) muscle. Botulinum toxin type A is a protease that, through degradation of the SNAP-25 protein within axonal terminals, prevents fusion of cytoplasmic vesicles to the presynaptic membrane and subsequent release of neurotransmitters, in particular acetylcholine (ACh). Release of ACh into the synaptic cleft is necessary for normal skeletal muscle contraction. Transient eyelid ptosis is a potential side effect of injection of botulinum toxin type A into the upper third of the face, with reported incidence between 2 and 11%. It occurs when the injected toxin migrates through the orbital septum and reaches the levator palpebrae superioris (LPS) muscle, a skeletal muscle innervated by the oculomotor (III) nerve. The superior tarsal (Müller) muscle acts in conjunction with LPS to elevate the upper eyelid. Müller muscle is a smooth muscle, innervated by the sympathetic nervous system via alpha-adrenergic receptors. Stimulation of these receptors (as caused by apraclonidine - a selective alpha-2 receptor agonist with weak alpha-1 activity) can compensate for partial LPS dysfunction, correcting 1 to 3 mm of eyelid ptosis. Apraclonidine is not known to cause significant displacement of botulinum toxin type A from its presynaptic receptors. Contraction of the LPS muscle, as for other skeletal muscles, occurs by stimulation of its cholinergic receptors within the neuromuscular junctions. Inhibition of alpha-adrenergic receptors in Müller muscle would cause it to relax, increasing the upper eyelid ptosis.

2013 A 48-year-old woman comes to the clinic with new left eyelid ptosis 1 week after receiving injections of botulinum toxin type A for treatment of glabellar rhytides. The patient is prescribed apraclonidine ophthalmic solution for the left eye. Complete resolution is noted 2 days later. Which of the following is the most important mechanism by which apraclonidine resulted in improvement of this patient's eyelid ptosis? A ) Displacement of botulinum toxin type A from its presynaptic receptors B ) Inhibition of alpha-adrenergic receptors in the levator palpebrae superioris muscle C ) Inhibition of alpha-adrenergic receptors in the superior tarsal (Müller) muscle D ) Stimulation of alpha-adrenergic receptors in the levator palpebrae superioris muscle E ) Stimulation of alpha-adrenergic receptors in the superior tarsal (Müller) muscle

The correct response is Option E. Since all components of the lip require revision, this is best achieved by recreation of the defect, and therefore the most appropriate code is 40720. Minor revisions of vermillion only or of half the lip would not address correction of all the components requiring reconstruction. Only recreation of the defect and repair will allow for closure of nasolabial fistulas, whistle deformity, and repair of the orbicularis oris across the lip.

2013 A 5-year-old boy is brought to the office because of a whistle deformity. History includes repair of a bilateral cleft lip at 6 months of age. On examination, the orbicularis oris is not in continuity across the lip. Dry, crusting mucosa on the vermillion of the whistle deformity and nasolabial fistulas are noted. A cleft lip revision is planned to repair the muscles, close the nasolabial fistulas, and correct the whistle deformity. Which of the following Current Procedural Terminology (CPT) codes is most appropriate for this procedure? A ) 40650 (Repair lip, full-thickness; vermillion only) B ) 40652 (Repair lip, up to half of vertical height) C ) 40654 (Repair lip, over one-half vertical height, or complex) D ) 40701 (Primary bilateral lip repair, one-stage procedure) E ) 40720 (Secondary lip repair, by recreation of the defect and reclosure)

The correct response is Option A. Sphincter pharyngoplasty for correction of velopharyngeal insufficiency is performed by elevation of myomucosal flaps from the posterior tonsillar pillar, which involves the palatopharyngeus muscle. The palatopharyngeus muscle is supplied by the cranial part of accessory (XI) nerve through the pharyngeal branch of vagus (X) nerve via the pharyngeal plexus. The medial pterygoid nerve innervates the tensor veli palatini. The greater and lesser palatine nerves are branches of the pterygopalatine ganglion which provide sensory innervation to the palate. The hypoglossal (XII) nerve innervates the lingual muscles, with the exception of the palatoglossus, which is supplied by pharyngeal branch of the vagus (X) nerve, via the pharyngeal plexus. A photograph is shown.

2013 A 6-year-old boy with velopharyngeal insufficiency is brought to the office for sphincter pharyngoplasty. History includes repair of a cleft palate as an infant. Innervation of the muscle used to create the sphincter arises from which of the following? A ) Cranial part of the accessory (XI) nerve B ) Glossopharyngeal (IX) nerve C ) Greater and lesser palatine branches of the pterygopalatine ganglion D ) Hypoglossal (XII) nerve E ) Medial pterygoid nerve

The correct response is Option C. The patient described has severe radionecrosis of the skin and the underlying bone from the previous radiation therapy. The skin is thinned, and spontaneous necrosis with scabbing can be seen in the photograph and will require extensive debridement including necrotic skin and underlying bone. As a result, closure with scalp rotation flaps will be inadequate because of the extensive skin damage, size of the defect, and exposed underlying dura. Similarly, bone burring will not be possible even if the underlying calvaria was preserved due to the history of radiation. Negative pressure wound therapy is not an option due to exposure of the underlying dura and brain matter. Similarly, acellular dermis will not be effective due to the damaged underlying tissues and chronic contamination. Radical debridement and coverage with a free flap (e.g., latissimus flap) will offer the best option for reconstruction in this patient because this procedure will deliver healthy vascularized tissues to cover the exposed cranial structures.

2013 A 62-year-old woman comes to the office because of skin necrosis and scabbing following a minor injury 6 weeks ago. A photograph is shown. History includes glioblastoma that was treated with craniotomy and radiation therapy 20 years ago. Debridement is performed, exposing the calvaria and dura. Which of the following is the most appropriate treatment for this patient? A ) Bone burring with skin grafting B ) Coverage with acellular dermis matrix C ) Coverage with a free flap D ) Coverage with scalp rotation flaps E ) Negative pressure wound therapy

The correct response is Option B. The circumflex scapular artery is the blood supply to the scapular bone or osteocutaneous free flap. The pedicle can be extended by including the subscapular artery and vein proximally. This flap can be designed as a chimeric flap to include the latissimus dorsi and serratus anterior muscles based on the thoracodorsal artery, another branch of the subscapular system, to reconstruct complex defects that involve multiple tissue types. The scapular flap can also be based on the angular branch of the thoracodorsal artery. One disadvantage of the scapular flap is that it is on the back, usually precluding simultaneous oncologic resection and flap harvest to save operative time. Composite bony reconstruction is indicated for the expected mandibular defect. Soft-tissue reconstruction alone for anterior mandibular defects is associated with significant cosmetic deformity as well as impaired masticatory, speech, and even swallowing function. The fibula osteocutaneous free flap, based on the peroneal artery, not the anterior tibial artery, is favored by many surgeons based on its generous bone length and good quality bone stock. However, the flap is contraindicated when blood supply to the distal lower extremity is compromised, such as in advanced peripheral vascular disease (also known as peripheral arterial disease, or PAD). The thoracoacromial artery is the blood supply for the pectoralis major muscle or myocutaneous pedicled flap, which is a soft-tissue flap that would not satisfactorily restore this patient's appearance and function. An osteomyocutaneous variant of the pectoralis major flap that incorporates the fifth rib has been described, but would not be a first-line option due to limited reach of the flap and the tendency to tether the reconstructed jaw to the neck as scar contracture occurs. The superficial circumflex iliac artery is the blood supply to the groin free flap, which is a soft-tissue flap without a bony component and, therefore, not appropriate for reconstructing the anticipated defect. The supraclavicular artery is the blood supply to the supraclavicular artery island pedicled flap, which is a soft-tissue flap that would be also inadequate to reconstruct this large bony defect.

2013 A 64-year-old man with peripheral vascular disease is evaluated because of an oral squamous cell carcinoma that is invading the mandible. On examination, pedal pulses are not palpable. Surgical resection and postoperative radiation therapy are planned. The resulting defect is expected to extend from the left mandibular parasymphysis to the right mandibular mid body. A flap supplied by which of the following arteries is most appropriate for reconstruction? A ) Anterior tibial B ) Circumflex scapular C ) Superficial circumflex iliac D ) Supraclavicular E ) Thoracoacromial

The correct response is Option B. The patient describes velopharyngeal insufficiency post-soft palate resection. Prosthetic obturation is the traditional means of reconstructing palatal defects. After 12 months, observation has already been performed without resolution of symptoms. Skin grafting of soft palatal defects is of little use because it does not adequately reestablish the bulk necessary for through-and-through palatal defects, therefore leading to contracture and palatal dysfunction. Tracheotomy would exacerbate speech and swallowing difficulties and therefore is not appropriate. Injecting the inferior turbinates with corticosteroids would not functionally correct the velopharyngeal insufficiency and is therefore not appropriate.

2013 A 65-year-old man is evaluated because of hypernasal speech and nasal regurgitation 12 months after he underwent resection of a soft palate tumor. Examination shows patent nasal passages and no soft palate. Which of the following is the most appropriate treatment? A ) Injection of corticosteroids to the inferior turbinates B ) Placement of an obturator prosthesis C ) Skin grafting D ) Tracheotomy E ) Observation only

The correct response is Option D. This is an extensive subtotal maxillectomy (Type IIIA Cordeiro classification) that requires reconstruction of a number of anatomical structures including the hard palate, the orbital floor, and the nasal lining. In addition, vascularized tissues are required to separate the maxillary sinus content from the orbit. In this case, the rectus flap is the best option because it can provide skin to re-line the maxillary sinus and repair the palatal defect. In addition, the rectus flap has sufficient bulk to obliterate the maxillary sinus and provide vascularized tissue to support the orbital floor reconstruction that is required. The omentum flap usually has enough bulk to obliterate the sinus but will not enable reconstruction of the palatal defect. The obturator will reconstruct the palate but will not provide vascularized tissue to line the orbital floor reconstruction and separate this region from the maxillary sinus. The temporalis flap can be used to cover the orbital floor reconstruction but does not reconstruct the palatal defect. The facial artery musculomucosal flap is a flap based on the facial artery and would not be useful in this reconstruction.

2013 A 65-year-old woman is evaluated immediately after she underwent subtotal maxillectomy for an adenoid cystic carcinoma. The resulting defect includes the right hemi-palate, maxillary sinus, orbital floor, and nasal lining. A photograph is shown. Which of the following is the most appropriate option for reconstruction? A ) Facial artery musculomucosal flap B ) Obturator C ) Omentum flap D ) Rectus flap E ) Temporalis flap

The correct response is Option B. Distraction osteogenesis allows anterior movements of the midface that cannot be achieved with conventional Le Fort III and bone grafting. The advancement obtained with single-stage midfacial advancement is limited by soft-tissue tension to 10 to 12 mm, and bone grafting is required to buttress the advanced segments. These procedures have long operative times and major blood loss, and some reports suggest long-term relapse presumably due to bone graft resorption. In contrast, the gradual soft-tissue creep achieved with distraction allows osseous advancement in excess of 30 mm. In addition, distraction osteogenesis obviates the need for bone grafting and relapse is minimal. The effect of successful midfacial advancement is a decrease in SNB, an increase in ANB, decrease in negative overjet, and an increase in upper airway volume. The latter change often leads to an improvement in obstructive sleep apnea. Exorbitism is reduced by the increase in orbital volume.

2013 In patients with syndromic craniosynostosis, successful mid facial distraction is most likely to result in which of the following outcomes? A ) Decreased ANB angle B ) Decreased exorbitism C ) Decreased upper airway volume D ) Increased negative overjet E ) Increased SNB angle

The correct response is Option A. When dealing with a near or total circumferential defect of the laryngopharyngeal unit, function must be considered as well as incidence of strictures and fistulas. Many of these patients receive a tracheoesophageal puncture prosthesis (TEP) for speech and many are quite intelligible after rigorous therapy. Speech with fasciocutaneous flaps, such as the anterolateral thigh flap and the radial forearm, is consistently better than with jejunal free flaps. In one study, a direct comparison between these flaps was performed, and 78% of patients used tracheoesophageal speech for conversation when reconstructed with an anterolateral thigh free flap compared with only 25% following a jejunal free flap. The distensibility and mucous production in reconstructions using a jejunal free flap appears to be responsible for the low-pitched, "wet" speech. There is some evidence that intensive speech rehabilitative programs can produce excellent results with jejunal flaps. A pedicled muscle flap is typically used to cover laryngeal or laryngopharyngeal closures, especially in radiation salvage cases. A silastic stent is usually used as salivary diversion while the reconstruction is healing. During its occupancy, no TEP speech can be generated. A gastric pull-up is usually used when a large portion of the cervical esophagus is involved with tumor, and thus a laryngopharyngoesophagectomy is performed so that the anastomosis of the swallowing system is cephalad to the thorax.

2013 A 65-year-old woman is scheduled to undergo reconstruction of a total laryngopharyngectomy defect. Use of which of the following is most likely to promote intelligible postoperative tracheoesophageal speech? A ) Anterolateral thigh flap B ) Gastric pull-up C ) Jejunal flap D ) Pedicled muscle flap E ) Silastic stent

The correct response is Option E. Kaposiform endothelioma is a malignant vascular tumor which is often associated with Kasabach-Merritt phenomenon. It is generally seen in infancy. It can be seen in the trunk and extremities. The skin overlying the tumor is deep red-purple, tense, and shiny, which is pathognomonic for kaposiform endothelioma. These patients typically have profound thrombocytopenia. As this is a malignant tumor, the primary mode of therapy is chemotherapy, sometimes combined with surgical resection. Vincristine is considered first-line therapy and has supplanted interferon alfa-2a due to its efficacy and decreased risk of major side effects. Interferon alfa-2a has a high risk of spastic diplegia, which is irreversible. Corticosteroids are not considered a first-line therapy for kaposiform endothelioma. Propranolol and imiquimod are not yet widely studied in kaposiform endothelioma.

2013 A 7-month-old female infant is brought to the office because of a large, rapidly growing vascular tumor of the mandible. Physical examination shows a reddish purple lesion with a centrifugally advancing rim of ecchymosis. Initial platelet count is below 10,000/mm3. Administration of which of the following is the most appropriate initial treatment? A ) Imiquimod B ) Interferon alfa-2a C ) Prednisone D ) Propranolol E ) Vincristine

The correct response is Option C. The Karapandzic technique involves performing circumoral incisions, mobilizing the orbicularis oris muscle, and preserving the nerves as well as the vascular supply to the lips from the superior and inferior labial arteries. The advantage of Karapandzic flaps is that they maintain a continuous circle of functioning orbicularis muscle, which helps to restore oral competence. Microstomia can occur, but is usually less of a problem in older patients who have greater tissue laxity. The optimal method for lip reconstruction depends on the size of the defect, the location of the defect, and the laxity of the remaining tissues. Primary closure can lead to excellent cosmetic and functional results but is restricted to wounds less than one third to one half of the lip width. The Estlander flap is a full-thickness lip transposition flap that borrows tissue from the opposite lip and can be used to reconstruct defects up to two thirds of the lip width. However, the Estlander flap is specifically designed to restore defects of the oral commissure. The Abbé flap is also a transposition flap that "steals" tissue from the opposite lip and is used for central defects, but must remain pedicled by a bridge of lip for several weeks, severely restricting mouth opening during that time. The radial forearm fasciocutaneous free flap is used for total lower lip defects, particularly in situations where sufficient cheek tissue cannot be recruited to close the defect (which is known as the Webster-Bernard technique). Suspension of the folded radial forearm flap over a palmaris longus tendon graft secured to the maxillary bone helps prevent flap ptosis that can result in loss of oral competence, a major problem that can occur with this technique. The facial artery musculomucosal (FAMM) flap is a pedicled flap consisting of buccal mucosa and a portion of the buccinator muscle that can be useful for reconstructing the vermillion lip, but not the cutaneous portion of the lip. Also, because the FAMM flap is based on the facial artery, it may not be reliable following a neck dissection in which the facial artery has been ligated.

2013 A 75-year-old man comes to the office because of a squamous cell carcinoma of the lower lip. A wide local excision, removing two thirds of the lower lip extending inferiorly to the chin crease but sparing both oral commissures and bilateral selective neck dissections, is performed. Which of the following is the most appropriate method of reconstruction? A ) Estlander flap B ) Facial artery musculomucosal flap C ) Karapandzic flap D ) Primary closure E ) Radial forearm fasciocutaneous free flap

The correct response is Option E. Treatment of airway compromise associated with Robin sequence depends on the severity and etiology of the airway obstruction. Obstruction is often caused by glossoptosis (tongue retropositioning into the oral pharynx), but this may not be the only cause of respiratory distress in these infants. Additional anatomic and physiologic causes of poor respiration (e.g., central apnea, laryngeal anomalies) have been documented in Robin sequence patients and can limit treatment options. Mandibular distraction and tongue-lip adhesion are both effective addressing the glossoptosis and upper airway obstruction, but will have no effect on the subglottic laryngomalacia and webbing. Similarly, prone positioning and nasal airway do not address the lower airway causes of airway obstruction in this child. Only tracheostomy will bypass the laryngeal anomalies and ensure adequate airway patency.

2013 A male newborn is emergently intubated in the prenatal intensive care unit because of severe respiratory distress. Examination shows marked mandibular micrognathia and a cleft palate. Nasopharyngoscopy shows glossoptosis, laryngomalacia, and laryngeal webbing. Which of the following is the most appropriate treatment for the airway compromise? A ) Extubation and prone positioning B ) Insertion of a nasal airway C ) Mandibular distraction D ) Tongue-lip adhesion E ) Tracheostomy

The correct response is Option A. Growing skull fractures occur in children. If a child sustains a fracture that appears linear but has an underlying dural injury, then the fracture may fail to heal. As the skull continues to grow, so does the cranial defect. Trauma alone, open fontanelles, genetic bone disorders, and craniosynostosis are not associated with growing skull fractures. Children under 2 years of age have a tremendous ability to regenerate bone and heal fractures or even large defects. Having normal, healthy dura is very important to bone healing.

2013 An 18-month-old girl is brought to the office because of a soft spot with pulsations on the right parietal region of her skull. At age 6 months, the patient sustained a linear skull fracture due to head trauma, which was managed nonoperatively. CT scan shows a growing skull fracture. Which of the following is the most likely cause of this patient's growing fracture? A ) Dural injury at the time of the fracture B ) Genetic bone healing disorder C ) Isolated right unicoronal craniosynostosis D ) Patent anterior fontanelle E ) Trauma-related infection

The correct response is Option E. Hemifacial microsomia (HFM) is the second most common congenital anomaly, with a reported incidence of approximately one in 5,600 live births. It is thought to be a result of vascular injury to the first and second branchial arches during the 30th to 45th day of pregnancy. The presentation of HFM is highly variable and ranges from mild facial asymmetry and microtia to a severe asymmetry of the orbit and mid and lower faces. Vertebral, cardiac, and renal malformations can also occur with HFM. A number of studies document velopharyngeal insufficiency (VPI) in patients with HFM. This VPI results from a unilateral hypodynamic palate. With this lack of movement, the nasopharyngeal port cannot be closed fully, and the speech is hypernasal. Some patients are able to compensate for the asymmetry of the palatal movement and have normal speech. Approximately 15% of patients with HFM have evidence of VPI based on speech evaluation and nasoendoscopy.

2013 Patients with hemifacial microsomia have an increased incidence of which of the following? A ) Craniosynostosis B ) Glossoptosis C ) Hemihypertrophy D ) Microstomia E ) Velopharyngeal insufficiency

The correct response is Option C. The radial forearm fasciocutaneous free flap is the most appropriate reconstructive choice among those listed. It is generally a thin, pliable flap with a long pedicle and vessels of adequate caliber for straightforward microvascular anastomosis. The physical characteristics of this flap are well-suited to preserving the mobility of the remaining hemitongue, resulting in reasonable speech and swallowing function following surgery in most cases. The primary goals of reconstruction following hemiglossectomy include watertight wound closure such that oral secretions do not communicate with the neck contents and result in a fistula, and restoration of speech and swallowing function by preserving the mobility of the remaining native tongue. Primary closure would result in severe tongue tethering and impaired speech and swallowing. Although likely to contract less than a split-thickness skin graft, a full-thickness skin graft may also restrict mobility of the tongue and is unlikely to achieve a watertight wound closure. The submental artery island flap is based on a branch of the facial artery that can be used to close defects up to approximately 7 × 18 cm, depending on neck skin laxity. This flap is generally unreliable following neck dissection in which the facial artery and its branches may be ligated. The rectus abdominis musculocutaneous free flap is a highly reliable free flap but is often too bulky to permit optimal mobility of the remaining tongue, especially in patients with truncal obesity.

2013 An obese 65-year-old man who undergoes resection of an oral tongue nodule has a 5 × 9-cm defect of the hemitongue and floor of the mouth. Which of the following is the most appropriate method of reconstruction? A ) Full-thickness skin grafting B ) Primary closure C ) Radial forearm fasciocutaneous flap D ) Rectus abdominis musculocutaneous flap E ) Submental artery island flap

The correct response is Option C. In the absence of other pulmonary disease, the stridor is most likely being caused by the hemangioma which is present in the beard distribution. Hemangiomas can be present in the airway as well as the surrounding soft tissue. In some centers, propranolol has supplanted corticosteroids as the first-line medical therapy for complicated hemangiomas. Currently, there is no universally accepted protocol for propranolol administration. Propranolol is initially given intravenously and then converted to oral dosing. Blood pressure and heart rate monitoring must be done on an outpatient basis, generally by a pediatric cardiologist. Treatment is typically 2 to 6 months. Systemic corticosteroids are still considered effective therapy. Corticosteroids generally require prolonged therapy to prevent rebound effect. It can take upwards of 2 years for treated patients to catch up in growth after prolonged corticosteroid therapy in infancy. Treatment is typically upwards of 9 to 12 months. Vincristine is a first-line treatment for kaposiform hemangioendothelioma. It has supplanted interferon alfa-2a, which is avoided due to the risk of spastic diplegia. Topical imiquimod is still considered an experimental therapy.

2013 An otherwise healthy 3-month-old female infant is admitted to the pediatric intensive care unit because of progressive difficulty breathing for the past 3 weeks. Examination shows mild stridor. Cultures are negative for acute viral or pulmonary illness. A photograph is shown. Administration of which of the following is the most appropriate treatment? A ) Imiquimod B ) Interferon alfa-2a C ) Propranolol D ) Vincristine

The correct response is Option B. The Current Procedural Terminology (CPT) code set is maintained by the American Medical Association through the CPT Editorial Panel. The CPT code set describes medical, surgical, and diagnostic services and is designed to communicate uniform information about medical services provided and procedures performed among physicians, coders, patients, accreditation organizations, and payers for administrative, financial, and analytical purposes. CPT codes group together portions of an operation or procedure. Reporting or billing for each individual part of a procedure is legal and known as "unbundling" the procedure. The free flap breast reconstruction code includes harvesting of the flap, microvascular transfer, closure of the donor site, and inset/shaping of the flap into a breast. The CPT code book specifically states that use of the operating microscope cannot be added. It does not include repair of an incidentally found hernia, which may be coded additionally.

2013 During breast reconstruction with a free transverse rectus abdominis musculocutaneous (TRAM) flap, the axillary vessels are dissected, and the anastomosis is performed using an operating microscope. During flap elevation, an umbilical hernia is encountered and repaired. The TRAM flap is then contoured and inset to form the new breast. The abdomen is then closed. In addition to 19364 (breast reconstruction with free flap), which of the following is the most appropriate Current Procedural Terminology (CPT) coding for this procedure? A ) 15847 (abdominoplasty) B ) 49585 (repair of umbilical hernia, reducible) C ) 49585 (repair of umbilical hernia, reducible), 15847 (abdominoplasty), and 69990 (use of operating microscope) D ) 69990 (use of operating microscope) E ) No additional coding is necessary

The correct response is Option B. The stage of mixed dentition is defined as the age range in which there are both deciduous (primary) and permanent (secondary) teeth erupted in the oral cavity at the same time. Normally, the mandibular and maxillary teeth erupt in a slightly different pattern. This usually occurs at age 6 to 7 years and is completed by age 11 to 12 years. In the maxilla, the order of eruption is as follows: first molar, central incisor, lateral incisor, first premolar, second premolar, canine, second molar, and third molar. In the mandible, the order is slightly different and is as follows: first molar, central incisor, lateral incisor, canine, first premolar, second premolar, and second molar. The permanent first molars erupt between ages 6 and 7 years, the central and lateral incisors erupt between ages 6 and 8 years, and the first premolars erupt between ages 8 and 9 years. The first tooth to erupt is the permanent mandibular first molar, which erupts first in a position posterior to the deciduous second molar. There are no premolars in deciduous teeth.

2013 During the period of mixed dentition, which of the following is the first permanent tooth to erupt? A ) Mandibular canine B ) Mandibular first molar C ) Mandibular first premolar D ) Maxillary central incisor E ) Maxillary lateral incisor

The correct response is Option B. In a recent review of the National Trauma Data Bank of over 1.3 million trauma patients, an analysis of isolated facial fractures and neurologic injury was undertaken. An isolated mandible fracture had a 5.1% relationship with a cervical spine injury. These data showed a higher than previously published rate of cervical spine and head injures associated with isolated facial fractures.

2013 In a trauma patient with an isolated mandible fracture, which of the following is the likelihood that this patient will have a concomitant cervical spine injury? A ) 1% B ) 5% C ) 15% D ) 25% E ) 50%

The correct response is Option A. Cryptotia is a congenital anomaly in which the upper part of the retroauricular sulcus is absent or buried under the temporal skin. Various surgical techniques have been reported for correction of cryptotia, starting with a V-Y plasty in 1933. Conventional methods using local flap, skin grafting, tissue expander, Z-plasty, and any combined approaches correct the skin deficiency of the upper auricle. However, cosmesis can still be unsatisfying because of a visible periauricular scar, color mismatch, or a contracture deformity. Cryptotia may be treated early nonsurgically with splinting of the ear or with surgical release at a later age. Microtia is a hypoplastic condition of the ear which includes a spectrum from complete absence of the ear (anotia) to a smaller than normal ear with normal morphology. Microtia is seen in patients with the hemifacial microsomia. In fact, patients with isolated microtia are considered to have a mild form of hemifacial microsomia. Pixie ear deformity is a complication of rhytidectomy. Prominent ear has a widening of the conchal-scaphal angle, an increased auriculocephalic distance, and loss of the antihelical fold. Stahl ear, also known as Spock ear, has a third crus, a flat helix, and a malformed scaphoid fossa.

2013 Which of the following best describes the ear anomaly seen in the photograph? A ) Cryptotia B ) Microtia C ) Pixie ear D ) Prominent ear E ) Stahl ear

The correct response is Option D. Exorbitism describes bulging of the globe due to decreased volume of the bony orbit, whereas exophthalmos describes protrusion of the globe due to increased orbital soft-tissue contents. There is decreased orbital bony volume due to craniosynostosis in Apert syndrome. Exotropia and esotropia describe strabismus, in which the eyes are deviated laterally (outward) or medially (inward), respectively. Enophthalmos is a condition where the globe is recessed in the orbit. This can be due to increased bony volume or malposition of the bony orbit after trauma and/or under-correction or repositioning of the bony skeleton.

2013 Which of the following best describes the orbital deformities observed in a patient with Apert syndrome? A ) Enophthalmos B ) Esotropia C ) Exophthalmos D ) Exorbitism E ) Exotropia

The correct response is Option C. The innervation of the parotid gland comes from parasympathetic fibers of the glossopharyngeal nerve (cranial nerve IX). It also receives taste sensation (afferent) from the posterior one third of the tongue. The maxillary nerve of cranial nerve V (V2) is a sensory nerve and receives sensation from the mid face. Parasympathetic fibers (efferent) innervate the submandibular and sublingual glands via the chorda tympani. Afferent fibers, via the chorda tympani, send taste sensation of the anterior two thirds of the tongue. The vestibulocochlear nerve (cranial nerve VIII) supplies sound and equilibrium to the brain. The Arnold nerve, also called the auricular branch of the vagus nerve (cranial nerve X), innervates the external acoustic meatus. Stimulation of the Arnold nerve can lead to reflex coughing (Arnold reflex).

2013 Which of the following cranial nerves is responsible for parasympathetic innervation to the parotid gland? A ) Trigeminal (V) nerve B ) Vestibulocochlear (VIII) nerve C ) Glossopharyngeal (IX) nerve D ) Vagus (X) nerve E ) Hypoglossal (XII) nerve

The correct response is Option B. There have been numerous publications on subtopics in clinical photography. Clinical photographs should be taken with the same camera lens, lens setting, lighting, film, and patient position to ensure reproducibility and to enable valid pre- and postoperative comparisons. Specific anatomical landmarks are used as guides for positioning each pose. These landmarks should be used each time to ensure the exact duplication of poses. A grid pattern can also be used. Two lenses are recommended. A single lens with a focal length of 50 to 60 mm with macro capability is required as a minimum, although the longer focal length lens of 90 to 105 mm can aid in facial photography close-ups. At least two light sources are required to avoid flatness and shadowing. An even, uncluttered background is best, with a color such as sky blue that is most visually pleasing with skin tones. This can be painted on a wall, paper, or retractable fabric. Three-dimensional capability is useful for volumetric evaluation and changes, but is not yet standard of care in plastic surgery photography.

2013 Which of the following is a critical feature required to obtain the optimal reproducible digital photographs for valid pre- and postoperative comparison? A ) Lens with a focal length of 90 to 105 mm B ) Matching anatomical landmarks C ) Single flash D ) Three-dimensional capability E ) White background

The correct response is Option D. The periodontal ligament is responsible for keeping the tooth anchored. Enamel is the outer protective layer of the tooth. Dentine, enamel, cementum, and pulp are the four major components of the tooth but none are responsible for anchoring the tooth.

2013 Which of the following structures (A-E) is responsible for anchoring the tooth in its socket? A) B) C) D) E)

The correct response is Option B. Maffucci syndrome is associated with venous malformations and multiple enchondromas. It can be associated with malignant chondrosarcomas, and intracranial tumors occur in 20% of patients. Kasabach-Merritt syndrome is associated with a hemangioma or with diffuse hemangiomatosis. The hallmark is profound thrombocytopenia. The child can present with petechiae, ecchymosis, and bleeding. Sturge-Weber syndrome is associated with facial capillary malformations (port-wine stain) distributed in a trigeminal nerve pattern, most often the first and second divisions of the nerve. Vascular malformations on the ipsilateral side can be found deep to the leptomeninges, and seizure disorders are common. Parkes-Weber syndrome is a variant of Klippel-Trenaunay syndrome (patchy port-wine stains of the lower extremity with lymphatic/venous malformations and hypertrophy). However, in Parkes-Weber syndrome, there are arteriovenous fistulae present. Osler-Weber-Rendu syndrome is also called hereditary hemorrhagic telangiectasia syndrome. It is inherited in an autosomal dominant fashion. The telangiectasia is located on the face, tongue, lips, nasal and oral mucosa, conjunctiva, and hands/nails. The lesions often emerge later in life.

2013 Which of the following syndromes is most likely in a patient with venous malformations? A ) Kasabach-Merritt B ) Maffucci C ) Osler-Weber-Rendu D ) Parkes-Weber E ) Sturge-Weber

The correct response is Option C. Patients with 22811.2 deletion may have cardiac abnormalities, renal issues, immune deficiencies, speech and feeding delays, mental health issues, developmental delay, cleft palate, and calcium regulation disturbances. Postoperatively, patients with 22811.2 deletion may be at higher risk than non-deletion patients for hypocalcemia and should have postoperative calcium concentration checked in the first 6 hours postoperatively to identify and correct any abnormalities. Failure to identify postoperative hypocalcemia may lead to increased morbidity and mortality. Fluorescent in situ hybridization is a genetic test that is unnecessary in this patient who already has the diagnosis of 22811.2 deletion. The sodium, potassium, and phosphate values are not routinely obtained early postoperatively.

2014 A 10-month-old male infant with 22q11.2 deletion and preoperative basic metabolic profile within the reference range is scheduled for palatoplasty to treat a soft palate cleft. After surgery, which of the following laboratory studies is most appropriate within the first 6 hours? A ) Fluorescent in situ hybridization B ) Measurement of serum phosphate concentration C ) Measurement of serum calcium concentration D ) Measurement of serum potassium concentration E ) Measurement of serum sodium concentration

The correct response is Option C. The patient described has McCune-Albright syndrome. It is a triad of polyostotic fibrous dysplasia, precocious puberty, and café-au-lait spots. Surgical intervention is not indicated in asymptomatic lesions. The general indications for surgery are aesthetic imbalance, facial disfigurement, distortion of functional occlusion, orbital dystopia, ocular proptosis, impingement on neural foramina, impingement on the optic nerve, and intractable pain. It is debatable whether contour reduction or resection and replacement of the afflicted bone is preferable. Recent literature seems to favor the latter. Biopsy is generally not needed for diagnosis, as imaging studies are generally specific for fibrous dysplasia. Equivocal imaging may warrant biopsy to confirm the diagnosis. In general, medical treatment has had little impact on fibrous dysplasia. Early attempts at treatment included chemotherapy, glucocorticoids, calcitonin, and radiation therapy, which were all unsuccessful. Radiation therapy should never be used, as it clearly promotes sarcomatous degeneration of fibrous dysplasia. Pamidronate, a bone resorption-inhibiting bisphosphonate, has been shown in multiple small studies to decrease pain associated with fibrous dysplasia and decrease bone turnover. Nonsteroidal anti-inflammatory drugs can be effective, as can narcotic analgesics in pain control. Referral to a pain specialist may be necessary.

2014 A 12-year-old girl is brought to the office because of an enlarging mass of bone in the maxilla, precocious puberty, and café-au-lait spots. No aesthetic distortion or functional impact from the lesion is noted. She has intermittent bone pain in the upper extremities. Which of the following treatments is most appropriate in this patient? A ) Calcitonin B ) Doxycycline C ) Pamidronate D ) Prednisone E ) Radiation therapy

The correct response is Option E. Nasopharyngeal angiofibromas, also known as juvenile nasopharyngeal angiofibromas, are benign but locally invasive vascular tumors that occur almost exclusively in adolescent males. Onset is most common in the second decade of life, and rarely occurs after age 25 years. Symptoms include unilateral or bilateral nasal obstruction, frequent epistaxis or blood-tinged nasal discharge, and conductive hearing loss from Eustachian-tube obstruction. In advanced stages, the angiofibroma can deform the nose, face, and orbits, as well as erode into the cranial cavity and put pressure on the optic chiasm, resulting in diplopia. Treatment is usually surgical with radiation and reserved for extensive cases such as those with intracranial extension. Preoperative embolization as well as hormone therapy with estrogens, may limit blood loss. Nasopharyngeal angiofibromas are highly vascular, and office biopsies should be avoided. Inverted papilloma is a benign, locally aggressive neoplasm that arises in the nasal cavity and is associated with squamous cell carcinoma in approximately 5% of patients. The age of onset is usually between 40 and 60 years. Surgery is the primary treatment of inverted papilloma. Encephaloceles are neural tube defects that result in sac-like protrusions of the meninges (meningocele) or brain and meninges (meningoencephalocele) in various locations along the cranium, including intranasally. They tend to be bluish, soft, compressible masses that transilluminate. Biopsy may result in a cerebrospinal fluid leak. Hemangiomas are benign vascular lesions that are present at birth and characterized by a rapid growth phase around the age of 1 to 6 months followed by gradual involution over 1 to 12 years. A hemangioma would not be expected to first occur in adolescence. Dermoid cysts are derived from ectodermal and mesodermal tissue and may contain skin, hair follicles, sebaceous glands, and sweat glands. Dermoids are usually firm and noncompressible and most frequently occur as a slow-growing cystic mass over the dorsum of the nose, but may also be entirely intranasal. Dermoid cysts may also have a dural component and should not be biopsied until intracranial communication can be ruled out by x-ray studies. Encephaloceles, hemangiomas, and dermoid cysts are congenital nasal masses that occur in infancy rather than adolescence.

2014 A 16-year-old boy comes to the office because of a progressive 6-month history of unilateral nasal obstruction and frequent epistaxis. Anterior rhinoscopy shows a soft, smooth, purplish lobulated mass filling the left nasal cavity. An attempted office biopsy results in profuse bleeding. Which of the following is the most likely diagnosis? A ) Dermoid cyst B ) Encephalocele C ) Hemangioma D ) Inverted papilloma E ) Nasopharyngeal angiofibroma

The correct response is Option C. Cleft palates can affect tooth development, leading to a variety of dental abnormalities. In general, the upper lateral incisor tooth bud is most commonly susceptible to injury in the area of the cleft in both the deciduous and permanent teeth. Other teeth in the area of the cleft, such as the canines, may be affected as well. The premolar, molar, and central incisor tooth buds are typically too far from the cleft area to be affected.

2014 A 2-month-old male infant is evaluated for complete unilateral cleft of the lip and palate. Development of which of the following tooth buds is most likely to be impaired in this patient? A ) Central incisor B ) First molar C ) Lateral incisor D ) Premolar E ) Third molar

The correct response is Option B. The risk of having a child with a cleft of the lip and palate is multifactorial. In familial cases, the risk is dependent on the family history and sibling involvement. In this case, neither the family history nor sibling history is available, so risk calculation is dependent upon the child's personal history only. For males with an oral cleft, the prevalence of clefts in their first offspring is 4.7%, and for females it is 3.6%. If this child were to have a child with a cleft, then the risk of subsequent children to also have a cleft would be 17%. If this child also had lip pits, then the risk of having a child with a cleft would be 50% (van der Woude syndrome).

2014 A 2-year-old boy who was recently adopted is brought to the office for evaluation and treatment of cleft of the lip and palate. Physical examination shows involvement of the lip, alveolus, and entire palate. A photograph is shown. He is otherwise healthy with no other congenital anomalies. Which of the following is this patient's risk of having a child with cleft of the lip? A ) 1% B ) 5% C ) 10% D ) 15% E ) 50%

The correct response is Option C. Given the patient's history of diabetes and necrotizing non-purulent infection after minimal trauma, he is likely to have mucormycosis, a life-threatening fungal infection caused by organisms from the class Zygomycetes. On microscopy, tissue samples from patients with mucormycosis demonstrate right-angle nonseptate branching hyphae. Grape-like clusters of gram-positive bacteria is not appropriate. This option describes the characteristic appearance of a staphylococcal infection. Given the patient's lack of cellulitis or purulent infection, it is an unlikely mechanism for this necrotizing ulceration. Septate nonbranching hyphae and yeast forms is not appropriate. This option describes the characteristic appearance of a candidal infection. Given the patient's lack of marked erythema and excoriation, and location of the infection on the face, rather than in skin folds, the likelihood of Candida as the primary pathogen is extremely low. Chain-like collections of gram-positive bacteria is not appropriate. This option describes the characteristic appearance of a streptococcal infection. Although streptococcal infections are common in the head and neck region, the patient's history and appearance of the lesion do not support Streptococcus as the causative organism. Tiny yeast forms with occasional unequal bud formation is not appropriate. This option describes the characteristic appearance of Histoplasma capsulatum, an opportunistic fungus, which may cause marked pulmonary infections in immunocompromised patients. Often emergent debridement is required, and that decision will need to be made on Gram stain, not on final culture.

2014 A 21-year-old man with type 1 diabetes mellitus comes to the emergency department because of a large necrotizing, non-purulent infection after minimal trauma to the right cheek. Radical surgical debridement of the ulcer is performed, and the tissue is sent for histologic and microbiologic evaluation. Which of the following organisms are most likely to be found on light microscopy? A ) Chain-like collections of gram-positive bacteria B ) Grape-like clusters of gram-positive bacteria C ) Right angle nonseptate branching hyphae D ) Septate nonbranching hyphae and yeast forms E ) Tiny yeast forms with occasional unequal bud formation

The correct response is Option A. A Marcus Gunn pupil is a relative afferent pupillary defect caused by a lesion of the optic nerve (between the retina and the optic chiasm) or severe retinal disease. It is observed during the swinging-flashlight test whereupon the patient's pupils constrict less (therefore appearing to dilate) when a bright light is swung in front of the unaffected eye to the affected eye. The affected eye still senses the light and produces pupillary sphincter constriction to some degree, albeit reduced. Conditions that do not cause a Marcus Gunn pupil include cataracts, vitreous hemorrhage, injury to the oculomotor nerve (cranial nerve III), or symmetrical acute glaucoma.

2014 A 21-year-old woman comes to the emergency department because she has had ocular pain and decreased vision since she was struck in the face with a bottle 2 hours ago. On examination in a dark room, both pupils constrict when a light is shone in the right or left sides directly. When the light is shifted from the right to the left, the pupils dilate. Which of the following is the most likely pathologic condition in this patient? A ) Central retinal artery occlusion B ) Oculomotor (III) nerve injury C ) Preexisting cataract D ) Symmetrical acute glaucoma E ) Vitreous hemorrhage

The correct response is Option A. Le Fort I osteotomy with downward repositioning effectively lengthens the maxilla in cases of isolated vertical maxillary hypoplasia. The maxilla is repositioned vertically in its entirety or rotated downward, depending on whether or not the hypoplasia extends to the posterior maxilla. The goal is to close the anterior open bite and to restore facial height, allowing 3 to 4 mm of upper incisor to show with lips in repose. Perinasal osteotomy is a procedure designed to lengthen the skeletal framework of the nose. It lengthens and increases nasal projection. It is therefore a suitable procedure for patients with nasomaxillary hypoplasia and a foreshortened nose, but with normal dental occlusion and facial height. It does not correct maxillary height or change the dental relationships. Naso-orbito-maxillary osteotomy is a step beyond perinasal osteotomy, in that it corrects both the foreshortened and retruded nasal framework and maxillary hypoplasia horizontally and vertically. The entire osteotomized segment includes the central section of the maxilla from nasion to teeth, and from one internal orbital rim to the other. It can therefore close an anterior open bite when vertical maxillary insufficiency is a component of the deformity in addition to a retruded nasomaxillary complex. However, it would most likely shift the occlusion into class II if there were not also a horizontal deficiency of the maxilla. Therefore, it is not an appropriate procedure for the patient in the vignette because it would alter the naso-orbital region unnecessarily, and possibly cause a new deformity or abnormal relationship in this otherwise isolated vertical maxillary deficiency. The indications for a or a naso-orbito-maxillary osteotomy would overlap those for a Le Fort II osteotomy. Le Fort II osteotomy is indicated for nasomaxillary hypoplasia with a recessed maxilla and class III malocclusion. This is frequently noted in patients with a history of cleft lip and palate. The same discussion used for the naso-orbito-maxillary osteotomy would apply here as well. Sagittal split osteotomy is a procedure that modifies the mandible, permitting setback or advancement of the mandibular dentition when the cause of the malocclusion is mandibular hypoplasia or overdevelopment. It has no effect on the maxilla.

2014 A 22-year-old woman comes to the office for evaluation of an abnormal bite. On physical examination, she has an anterior open bite, and the upper teeth are not exposed with the lips in repose. Cephalometric analysis shows a nasion (N) to anterior nasal spine (ANS) distance of 45 mm (N 52-57 mm), an ANS to menton (Me) distance of 63 mm (N 63-68 mm), and an N-ANS:ANS-Me ratio of 1:1.4 (N 1:1.2). All other measurements are within the reference ranges. Which of the following is the most appropriate surgical procedure for correction of this patient's deformity? A ) Le Fort I maxillary osteotomy with downward repositioning B ) Le Fort II osteotomy with maxillary advancement C ) Naso-orbito-maxillary osteotomy D ) Perinasal osteotomy E ) Sagittal split osteotomy with mandibular setback

The correct response is Option A. The most appropriate answer is dentigerous cyst. This type of cyst develops in the context of an unerupted tooth, which can be seen below the cyst. The cyst is lined with benign nonkeratinizing epithelium and is caused by degeneration of the enamel reticulum of the unerupted tooth. It is the second most common type of jaw cyst. Two thirds occur in the mandible. Gingival cysts appear most commonly on alveolar ridges of infants but can also rarely appear in adults. Their origin relates to rests of dental lamina, and, unlike dentigerous cysts, these contain keratin. Clinically, they are soft and fluctuant, and range in size between 1 and 15 mm. Primordial cysts and odontogenic keratocysts are equivalent. They develop from rests of dental lamina and basal cell hamartomas. Therefore, unlike dentigerous cysts, these are lined with a keratinizing stratified squamous epithelium that is sometimes dysplastic. Their size range is 1 to 9 cm, and pain is a common symptom. Gorlin syndrome includes the association of multiple odontogenic keratocysts with multiple basal cell carcinomas, nasal deformity, skeletal abnormalities, calcification of the falx cerebri, and palmar or plantar pits. Calcifying odontogenic (Gorlin) cysts are distinct from those above because they may be part cystic and part neoplastic. The histology shows features similar to the calcifying epithelioma of Malherbe (epithelium undergoing keratinization and calcification), and ameloblastic proliferations. Radiographically, they contain various amounts of radiopaque (calcified) material and are usually located anterior to the first molars ranging in size from 1 to 8 cm. Radicular cysts are the most common type of jaw cysts and develop at the apex of a nonviable erupted tooth, from epithelial rests of Malassez in the periodontal ligament. As these cysts are inflammatory in nature rather than developmental, they are usually preceded by a periapical granuloma. Histology shows a fibrous shell lined with nonkeratinizing stratified squamous epithelium infiltrated with chronic inflammatory cells. Due to their painless quality, they most commonly occur as an incidental finding of routine x-ray studies of the maxilla. These cysts are radiolucent, and differ from dentigerous cysts because they are located at the apex of an erupted tooth rather than at the crown of an unerupted tooth.

2014 A 23-year-old man comes to the office because of a 1-year history of painless swelling, asymmetry, and loss of interdental relationships on the right side of the jaw. Physical examination shows crowding of the right-sided first and second molars and premolar dentition. The third molar has not erupted; in its place there is a palpable firm enlargement of the mandible. Panoramic x-ray study (Panorex) shows a 3-cm radiolucent unilocular cyst. Percutaneous biopsy of the cyst shows nonkeratinizing stratified squamous epithelium. Which of the following is the most likely diagnosis in this patient? A ) Dentigerous cyst B ) Gingival cyst C ) Gorlin cyst D ) Primordial cyst E ) Radicular cyst

The correct response is Option A. The cribriform plate is the horizontal component of the ethmoid bone which supports the olfactory bulb and creates a passageway for the olfactory nerves. The cribriform plates (right and left) have many foramina that act as passageways for olfactory nerves and are in intimate contact with the meninges. In addition to anosmia (loss of smell), injury to the cribriform plate may cause tearing of the meninges with cerebrospinal fluid leakage. The crista galli is a midline prominence of the ethmoid, immediately above the cribriform plates, which serves as a point of attachment for intracranial soft tissue. Other components of the ethmoid bone are the vertical portion and the lateral masses. The vertical portion is called the perpendicular plate, which forms part of the nasal septum. The lateral masses of the ethmoid bone contain a plate of bone called the lamina papyracea, which forms part of the medial orbital wall and the ethmoid air cells. While anosmia and taste abnormalities can occur following a variety of facial fracture patterns, including frontal bone, naso-orbital-ethmoid, nasal, Le Fort, and zygoma, the close anatomic relationship of the ethmoid cribriform plate to the olfactory nerve makes it most likely.

2014 A 23-year-old man sustains multiple fractures to the middle and upper face during a motor vehicle collision. The patient reports disturbances of smell immediately afterwards. Injury to which of the following bones is the most likely reason for this patient's anosmia? A ) Ethmoid B ) Frontal C ) Nasal D ) Sphenoid E ) Vomer

The correct response is Option D. Load-bearing osteosynthesis of the mandible may be accomplished with a reconstruction plate and locking screws. There are two basic types of mandibular fracture fixation: load-bearing osteosynthesis and load-sharing osteosynthesis. In load-bearing osteosynthesis, the plate (or external fixator) assumes all the forces of mandibular function at the fracture site. Common clinical indications include comminuted fractures, fractures with segmental defects, and those in the atrophic edentulous mandible. In load-sharing osteosynthesis, stability at the fracture site is created by the frictional resistance between the end of the bone and the hardware used for fixation. This requires adequate bony buttressing across the fracture line. Lag screws, Champy plate (at the oblique ridge), compression plates, and simple screws are examples of load-sharing osteosynthesis, which may have different levels of force distribution between the hardware and the bone.

2014 A 25-year-old man comes to the office because of jaw pain after sustaining a punch to the face 2 days ago. Maxillofacial CT scan shows a displaced comminuted fracture of the left mandibular angle. Open reduction and internal fixation is planned. Which of the following methods of fixation is considered load-bearing osteosynthesis? A ) Champy plate (oblique ridge) B ) Compression plates C ) Lag screws D ) Locking reconstruction plate E ) Simple screws

The correct response is Option D. The patient described has marked loss of facial soft-tissue volume related to the initial tumor resection and the adverse effects of postoperative radiation treatment. The best method for restoring soft-tissue volume is a scapular free flap. This method of reconstruction has advantages over the others listed. The free scapular flap does not rely on the damaged and scarred soft-tissue envelope for vascular support and, thus, it will retain its volume. In contrast, fat grafting, dermal fat graft, and layered acellular dermis all undergo some resorption, especially in this poorly vascularized recipient site. The scapular flap is of sufficient thickness to correct even a volume defect of this size. Although the other soft-tissue reconstructive options can improve contour, the volume required in this patient cannot be achieved with these modalities alone. The use of an alloplastic reconstruction alone can improve mid-facial volume, but will not address the lower third deficit. In addition, there is a moderate risk of extrusion and/or infection with this technique alone.

2014 A 28-year-old man is evaluated because of the facial deformity shown in the photograph. Three years ago, he underwent resection of an infratemporal malignancy and intraoperative alloplastic reconstruction of bony defects. Postoperatively, he underwent extensive radiation therapy. Which of the following is the most appropriate method for restoring facial volume in this patient? A ) Custom-fabricated alloplastic implantation B ) Dermal fat grafting C ) Implantation of layered acellular dermis D ) Parascapular free flap reconstruction E ) Serial fat grafting

The correct response is Option E. The patient in the photograph has the classic features of sagittal suture synostosis. The primary clinical features of scaphocephaly are lengthening of the cranial vault in the anterior-posterior dimension, an anteriorly displaced cranial vertex, bullet-shaped occiput, biparietal and/or temporal narrowing and frontal bossing. All of these features are present in this patient. Because of the patient's late clinical presentation for treatment, the most appropriate surgical procedure would be a total cranial vault reconstruction of which there are many variations and techniques. Because of the patient's age, he is not a candidate for either endoscopic-assisted wide strip craniotomy or spring-mediated cranioplasty. Ideal candidates for either procedure are ideally under age 6 months and 9 months, respectively, for an adequate clinical result. The results of both techniques have been shown to be comparable to traditional open remodeling procedures and are generally less invasive surgical procedures. Although the monobloc distraction may allow the opportunity to remodel the forehead, there is no clinical indication for midface distraction in the patient described. A monobloc distraction alone will not correct the other abnormal features of scaphocephaly. It is not an indicated procedure for the correction of isolated sagittal suture synostosis. Bilateral fronto-orbital advancement may allow the opportunity to remodel the forehead, but it will not address the other cranial vault abnormalities. There is no indication for remodeling the supraorbital bar in this case.

2014 A 3-year-old boy is brought to the office because of abnormal head shape since birth. Photographs are shown. The patient is developmentally appropriate and has no other medical problems. Which of the following is the most appropriate surgical procedure for correction of this deformity? A ) Bilateral fronto-orbital advancement B ) Endoscopic craniotomy and helmet therapy C ) Monobloc distraction D ) Spring-mediated cranioplasty E ) Total vault reconstruction

The correct response is Option E. The injury period is short enough that the potential for the initial neurorrhaphy to work is still likely. Clinical Tinel sign would be helpful to assess this further. Injection of botulinum toxin type A at this point would confuse the picture because it would prevent clinical monitoring of nerve recovery. If the patient does not recover nerve function within the next 6 months, then she is still a candidate for facial reanimation because the injury is not long-standing. A static procedure is not indicated unless the patient has a long-standing injury or is not a candidate for facial reanimation.

2014 A 30-year-old woman comes to the office for evaluation of an asymmetric smile. Two months ago, she underwent primary neurorrhaphy of a facial nerve laceration 2 cm lateral to the oral commissure. Physical examination shows no elevation of the upper lip with smiling on the side of the injury. Which of the following is the most appropriate management? A ) Cross-facial nerve grafting B ) Facial sling suspension C ) Free muscle transfer D ) Injection of botulinum toxin type A E ) Observation

The correct response is Option D. Fractures of the orbital roof that enter the middle cranial fossa may allow communication between the cavernous sinus and the carotid artery. Other findings include associated bruit and ipsilateral blindness, which would not be appreciated in the obtunded patient. The remaining fractures are important but are not as critical as a carotid-cavernous fistula.

2014 A 38-year-old truck driver is examined in the emergency department following a motor vehicle collision. He is sedated and intubated on a spine board. On examination, ecchymoses over the cheeks, swelling over the left jaw, and pulsation of the left globe are noted. Which of the following is the potential fracture of most concern? A ) Mandibular body B ) Medial maxillary buttress C ) Nasal bone D ) Orbital roof E ) Zygomatic-malar complex

The correct response is Option D. The patient described likely has a stage III tumor (T3 N0 M0). Standard management algorithms developed by the National Comprehensive Cancer Network recommend adjuvant radiation treatment when the persistence of positive margins relates to microscopic disease and not gross disease. If there is gross disease, either by physical examination or follow-up imaging, and it is resectable, then surgical resection of the residual disease should be done initially, followed by adjuvant radiation. Chemotherapy for major salivary gland tumors is appropriate as a first-line therapy concomitant with radiation only in cases of squamous cell carcinoma. In patients with mucoepidermoid, adenoid cystic, and adenocarcinomas, the role ofvchemotherapy is mainly palliative and reserved for advanced situations of recurrent or distant systemic disease. The absence of standard chemotherapy protocols for these situations attests to the degree of response that can be expected. Immunotherapy has no significant role in the treatment of major salivary gland malignancies. Neck dissection is indicated for malignant salivary gland tumors with clinically positive nodes detected either on physical examination or with preoperative imaging workup. This applies to parotid tumors of either the superficial or the deep lobe. Typical imaging to identify nodal disease would include CT scan or MRI or both. Neck dissections performed electively are rarely indicated, and only in very high-risk situations that are based on factors other than clinical and histologic features of the primary tumor. Radiation treatment is an effective treatment for negative necks with high risk of nodal disease, and is preferred over elective neck dissections. Surgical resection of persistent disease is indicated when a previously treated parotid mass was incompletely resected, and the remaining tumor is gross and resectable, rather than just microscopic. If not resectable, then the patient should have definitive radiation treatment.

2014 A 39-year-old woman undergoes a total parotidectomy with facial nerve preservation for mucoepidermoid carcinoma of the parotid gland. The final pathology report indicates microscopic disease at the deep margin, and follow-up imaging shows no gross residual disease. No detectable nodal or other metastases are noted. Which of the following is the most appropriate next step in management? A ) Chemotherapy B ) Immunotherapy C ) Neck dissection D ) Radiation therapy E ) Reexcision of the deep margin

The correct response is Option E. The patient described has bilateral macrostomia, also known as Tessier No. 7 cleft, the most common facial cleft in the Tessier classification system. This resulted from the failure of fusion between the maxillary and mandibular processes. Repair of the macrostomia can be undertaken in the first months of life. Duplicated maxilla has been reported in as high as 39% of patients with macrostomia. It is defined as having multiple supernumerary teeth and marked overlap of the maxillary arches. Other craniofacial findings such as mild mandibular/condylar anomalies and alveolar clefting have also been reported. The anatomy can be defined by three-dimensional CT scan and panoramic x-ray study (Panorex). Therefore, it is important to continue to observe these children with dental and orthodontic workups as they grow, even after the repair of the macrostomia. Glossoptosis and cleft palate are associated with Pierre Robin sequence. Renal anomalies may be associated with congenital anomalies of the ears, such as in branchiootorenal syndrome. Posterior fossa abnormalities and intracranial arterial anomalies are associated with PHACE syndrome. (P, posterior fossa; H, hemangioma; A, arterial anomalies; C, cardiac defects; E, eye anomalies). Coloboma of the eyelids and lacrimal gland anomalies are associated with Tessier No. 3 clefts (oro-nasal-ocular clefts). Macrostomia is most commonly associated with hemifacial microsomia.

2014 A 4-month-old female infant is brought to the office for evaluation. A photograph is shown. Which of the following additional abnormalities are most likely associated with this patient's condition? A ) Glossoptosis and cleft palate B ) Hydronephrosis of the kidneys and hearing loss C ) Lacrimal duct obstruction and coloboma of the eyelids D ) Posterior fossa abnormalities and stenotic cerebral arteries E ) Supernumerary teeth and duplicate maxilla

The correct response is Option C. The mutation for van der Woude syndrome has been mapped to the interferon regulatory factor 6 (IRF6) gene in chromosome 1. The inheritance is autosomal dominant with variable penetrance. Other associated findings include hypodontia (as high as 86%), high arched palate, lip pits, syngnathia, and ankyloglossia. Agenesis of the corpus callosum and preauricular skin tags are not associated with van der Woude syndrome. Glossoptosis is associated with Pierre Robin sequence. Medialized internal carotids can be seen in patients with velocardiofacial syndrome.

2014 A 4-week-old male newborn is evaluated for complete left unilateral cleft of the lip and palate. Genetics workup shows IRF6 gene mutation. Which of the following additional physical findings is most likely in this patient? A ) Agenesis of the corpus callosum B ) Glossoptosis C ) Hypodontia D ) Medialized internal carotid arteries E ) Preauricular accessory tag

The correct response is Option A. Blepharophimosis syndrome is the only diagnosis listed that is associated with congenital ptosis. Blepharophimosis syndrome is associated with a tetrad of findings including ptosis, telecanthus, epicanthus inversus, and decreased horizontal lid fissure. In type I blepharophimosis, patients have epicanthus inversus and ptosis. In type II, findings include telecanthus, ptosis, ectropion of the lower lids, absent epicanthal folds, and insufficient skin in all lids. Type III is notable for telecanthus, ptosis, hypertelorism, slanting palpebral fissures, and insufficient eyelid skin. Correction involves a variety of techniques including, but not limited to, Z-plasty, transnasal wiring of the medial canthal tendon, and ptosis correction with frontalis suspension. Other abnormalities of the blepharophimosis syndrome include flattening of the nasal dorsum, hypoplasia of the superior orbital rim, as well as forehead and ear deformities. In epiblepharon, the eyelashes are vertical as a result of excess pretarsal muscle and skin overriding the margin of the eyelid, often affecting the lower lids. This causes corneal irritation. If the condition does not resolve spontaneously in the first few years of life, correction involves shortening of the anterior lamella through excision of a horizontal piece of skin and orbicular muscle. Epiblepharon may also be caused by trauma, burns, or fractures. Euryblepharon refers to widening of the palpebral fissure both laterally and vertically caused by a shortage of eyelid tissue. Treatment involves corneal protection and may require surgical correction with standard techniques used for ectropion repair. Cryptophthalmos is a failure in embryonic development of the lid fold. The eye is buried in the developing cover of the epithelium and does not differentiate normally. It may be associated with other congenital abnormalities such as syndactyly, cardiac, facial, and ear defects. Treacher Collins syndrome is a maxillary-zygomatic cleft with a coloboma of the lower eyelid and absent eyelashes.

2014 A 4-year-old boy is brought to the office for treatment and evaluation of lid ptosis. On examination, bilateral lagophthalmos, poor levator excursion, and severe ptosis are noted. Which of the following is the most likely diagnosis? A ) Blepharophimosis syndrome B ) Congenital epiblepharon C ) Congenital euryblepharon D ) Fraser cryptophthalmos syndrome E ) Treacher Collins syndrome

The correct response is Option E. The patient described has velopharyngeal incompetency (VPI). The inability to adequately close the palate against the pharyngeal walls leads to nasal air escape during speech. This is most common with fricatives such as "s" and "z." As the degree of incompetence increases, speech errors with plosive sounds become apparent, such as "d" and "p" and "b." At the age of 4 years, intervention to correct VPI is appropriate. Speech therapy alone is unlikely to improve hypernasal speech production. A delay in treatment can lead to the development of compensatory misarticulation and worsening speech errors that will be difficult to correct in the future. The goals of surgery are to eliminate the symptoms of hypernasality and eliminate audible nasal emissions without causing complete obstruction of the velopharyngeal (VP) port, allowing for nasal breathing and nasal resonance. Multiple procedures have been described. Studies indicate that the success of repair depends on selecting the appropriate procedure based on the anatomy and the movement of the VP port. Sphincter pharyngoplasty involves reduction of the lateral and posterior aspects of the VP ports while maintaining the centric opening. The palatopharyngeus muscle is incised, and a flap is constructed from the posterior tonsillar pillar. These bilateral superiorly based musculomucosal flaps are juxtaposed in the midline of the posterior pharyngeal wall. This procedure is advantageous in that it potentially recreates a functional sphincter, and the incidence of postoperative nasal obstruction is less than that with the pharyngeal flap. The nasendoscopic examination demonstrates a classic palate closure pattern where the central gap is minimal, and a much larger gap occurs at the lateral ports. Thus, surgery to close the central gap, such as augmentation of Passavant ridge or a posterior pharyngeal flap, will have a lower success rate. The prosthetic speech bulb is most useful in patients with little or no VP motion. VP movement is essential to surgical success for the VP flap procedure or sphincteroplasty. Patients with little VP movement are good candidates for prosthetic management. A VP speech prosthesis can elevate the velum (lift), fill the residual velopharyngeal gap (obturator), or both (lift-orator).

2014 A 4-year-old girl is referred by her speech therapist because she has persistent nasal air escape with phonation. She underwent isolated repair of the cleft palate in infancy. Physical examination shows a long, mobile palate. No fistula is noted. Nasendoscopy shows good coronal closure with poor lateral pharyngeal wall movement. Which of the following is the most appropriate management? A ) Augmentation of Passavant ridge B ) Continued speech therapy C ) Implantation of a palatal lift prosthesis D ) Posterior pharyngeal flap E ) Sphincter pharyngoplasty

The correct response is Option A. Ameloblastomas are benign, locally invasive, odontogenic tumors accounting for 1% of tumors of the jaw and 10% of odontogenic tumors. Approximately 80% occur in the mandible and 20% occur in the maxilla. The peak incidence is in the third and fourth decades but may also arise in children and adolescents. Ameloblastomas may be radiographically found to be unilocular or, more commonly, multilocular with a "soap bubble" or "honeycomb" appearance. Treatment may be with enucleation and curettage or more radical resection. In rare cases, metastatic ameloblastoma and ameloblastic carcinomas have been reported. Fibrous dysplasia is a benign hamartomatous lesion that has a diffuse, "ground-glass" appearance on x-ray studies. It is usually treated conservatively with shaving and re-contouring of the bone. Squamous cell carcinoma is usually associated with a painful mucosal lesion. Radiographically, bony invasion may be noted in locally advanced cases. Osteosarcomas are aggressive malignancies of mesenchymal origin that exhibit osteoblastic differentiation. They are the most common primary bony cancer. Their radiographic appearance is variable and may include nonspecific destruction of the bone similar to a carcinoma, mottled ossification, similar to fibrous dysplasia, but without well-defined borders, or lamellar ossification (sheets of neo-osteogenesis). Nasopharyngeal angiofibromas are benign but locally invasive vascular tumors that occur almost exclusively in male adolescents. Their symptoms include nasal obstruction but can eventually cause facial asymmetry and eye displacement, as they grow from the region of the sphenopalatine foramen first into the nasopharynx and choanae then into the paranasal sinuses, pterygopalatine and infratemporal fossae, orbit, and even the intracranial cavity. Radiographically, they are nonencapsulated, lobular soft-tissue masses that demonstrate intense uptake of intravenous contrast due to their highly vascular nature. Extensive bony destruction is usually not a feature, but bone may be remodeled or resorbed.

2014 A 40-year-old woman comes to the office because of a 5-year history of firm, painless swelling of the upper jaw that has increased progressively in size. CT scan is performed, and the lesion is shown. Resection is performed. Pathologic examination shows odontogenic epithelial islands bordered by palisading columnar cells. No invasion into the surrounding tissues is noted. Which of the following is the most likely diagnosis? A ) Ameloblastoma B ) Fibrous dysplasia C ) Nasopharyngeal angiofibroma D ) Osteosarcoma E ) Squamous cell carcinoma

The correct response is Option A. Salivary gland tumors are relatively rare and make up about 3 to 4% of all head and neck neoplasms. The majority of salivary gland tumors (approximately 80%) originate in the parotid gland. Approximately 80% of parotid gland tumors are benign. Malignant tumors are associated with facial paralysis and pain, although they may also be asymptomatic. Malignant tumors may also metastasize to the regional lymph nodes and to distant sites. Pleomorphic adenoma, also known as benign mixed tumor, is the most common benign tumor of the parotid gland. This tumor is histologically characterized by epithelial and connective tissue elements, with stellate and spindle cells interspersed with a mixoid background. Warthin tumor (papillary cystadenoma lymphomatosum) is the next most common tumor of the parotid gland and is also benign. Warthin tumors predominantly occur in males and are bilateral in 10% of patients. Histologically, they are characterized by papillary cysts and mucoid fluid as well as nodules of lymphoid tissue. Mucoepidermoid carcinoma is the most common malignancy of the parotid gland and the second most common malignancy of the submandibular and minor salivary glands. Mucoepidermoid carcinomas contain two major elements: mucus-producing cells, and epithelial cells of the epidermoid variety. Low-grade tumors are associated with a predominance of mucus-secreting cells lining cysts and intervening nests of well-differentiated epidermoid cells. High-grade tumors show few or no mucus-producing cells and the epidermoid cells are poorly differentiated. Intermediate-grade tumors are defined by less cyst formation than low-grade tumors with nests of epidermoid and less differentiated intermediate cells. The biologic behavior of mucoepidermoid carcinomas correlates with their histologic grade. Adenoid cystic carcinoma is the second most common tumor of the salivary glands and the most common malignant tumor of the submandibular, sublingual, and minor salivary glands. It is slightly more common in female patients and typically affects patients between the ages of 30 and 70 years with a peak incidence of 40 to 59 years. There are three histologic subtypes: cribriform, tubular, and solid. The cribriform pattern has a classic "Swiss cheese" appearance with cells arranged in nests separated by round or oval spaces. The tubular pattern has a glandular architecture, while the solid (or basaloid) pattern has sheets of cells with little or no luminal spaces. Adenoid cystic carcinoma usually exhibits a protracted course characterized by indolent growth and a propensity for perineural invasion, reported to occur in 20 to 80% of patients. Distant metastases, most frequently to the lung, are not uncommon. Lymphangiosarcoma is a rare vascular tumor, which may be associated with prolonged lymphedema. These tumors are more commonly found in the extremities and under light microscopy appear as vascular channels with anaplastic endothelial cells.

2014 A 45-year-old woman comes to the office because of a painful 4-cm left parotid mass. Physical examination shows weakness of the left facial muscles. CT scan of the chest shows multiple lung nodules consistent with metastases. Parotidectomy is performed, and pathologic examination of the gland shows a cribriform ("Swiss cheese") pattern of cells with perineural invasion. Which of the following is the most likely diagnosis? A ) Adenoid cystic carcinoma B ) Lymphangiosarcoma C ) Mucoepidermoid carcinoma D ) Pleomorphic adenoma E ) Warthin tumor

The correct response is Option E. The patient described has new-onset facial paralysis brought on by Lyme disease infection. Neurologic manifestations (early disseminated infection) can show symptoms as early as a few days to a few weeks after initial bite. Treatment should be directed to the underlying disease, and doxycycline is the antibiotic of choice unless there are contraindications. There is no role for antivirals. Surgical treatment is contraindicated at this time unless the patient's facial palsy becomes permanent. Although botulinum toxin type A is efficacious in treating contralateral facial hyperkinesia, the patient is in the initial stages of Lyme disease, and treatment should be aimed at the primary disease at this time. Corticosteroids are helpful in reducing inflammation and edema which are thought to contribute to the neurologic manifestations in Bell palsy but not for neurologic manifestations of Lyme disease (Lyme neuroborreliosis).

2014 A 45-year-old woman is referred by her primary care physician because of left facial paralysis. She was hiking in the woods 2 weeks ago and pulled a tick off her leg at the end of the weekend trip. Three days ago, she had onset of a rash and fever. Since awakening this morning, she has been unable to move the left side of her face and has had painful spasms on the contralateral (right) side of her face when she tries to smile or talk. On physical examination, she is unable to move the left side of the face. Which of the following is the most appropriate management of this patient's condition? A ) Acyclovir therapy B ) Botulinum toxin type A injection C ) Contralateral facial nerve grafting and free gracilis flap D ) Corticosteroid therapy E ) Doxycycline therapy

The correct response is Option A. The fibula flap is the most appropriate option in this case because a long section of bone requiring multiple osteotomies is needed. The fibula flap can provide 18 to 20 cm of bone and has both an endosteal and periosteal blood supply enabling shaping of the bone with multiple osteotomies. In addition, a skin paddle can be harvested with the flap to reconstruct the floor of mouth defect. The scapula and radial forearm flaps also provide bone and soft tissues; however, these flaps will not provide a long enough bone segment and cannot be reliably osteotomized in multiple locations. The rectus and pectoralis flaps are soft-tissue flaps, and their use in this case would result in marked deformity because the anterior arch has been resected.

2014 A 47-year-old man is referred for examination of a composite defect of the mandible that extends from the right mandibular angle to the left mandibular angle. Which of the following flaps is most appropriate for reconstruction in this patient? A ) Fibula B ) Pectoralis C ) Radial forearm D ) Rectus E ) Scapula

The correct response is Option B. The anterior tonsillar pillar (palatoglossal arch) and tonsil are the most common site for primary neoplasms of the oropharynx. A 1-cm tumor (T1) at this location has a 71% incidence of cervical lymph node metastases. Oropharyngeal tumors arising at the base of the tongue have a similar incidence of lymphatic metastases, whereas oropharyngeal wall and soft palate T1 tumors only metastasize in 8 to 25% of cases. The most direct path of lymphatic drainage from the oropharynx is to level II (jugulodigastric) lymph nodes, which can be examined clinically. From level II the progression is sequential to levels III, IV, and V. It is rare to encounter a "skipped" level. The other less frequent lymphatic drainage pathways detectable only on imaging studies are to retropharyngeal and parapharyngeal nodes. Midline tumors can drain to bilateral lymphatic systems.

2014 A 55-year-old man is referred because of a 1-year history of ear and throat pain. Physical examination shows a 1-cm exophytic tumor of the anterior tonsillar pillar within the oropharynx. Biopsy of the tumor shows squamous cell carcinoma. Which of the following cervical lymphatic levels is most likely to be first involved in this patient? A ) I B ) II C ) III D ) IV E ) V

The correct response is Option D. The best option for reconstruction in this patient requiring circumferential pharyngeal reconstruction is the anterolateral thigh flap. This fasciocutaneous flap is supplied by perforators from the descending branch of the lateral femoral circumflex vessels, which are a branch of the profunda femoris vessels. The descending branch runs between the vastus lateralis and rectus femoris muscles, not the vastus medialis and rectus femoris. The radial forearm flap is based on the septum between the flexor carpi radialis and brachioradialis muscles in the arm. Although it can be used to reconstruct partial, noncircumferential pharyngectomy defects, it is not ideal for a long, circumferential defect in a previously radiated neck. The pedicle runs between the flexor carpi radialis and brachioradialis, not the palmaris longus. The circumflex scapular artery emerges from the triangular space in the back, which is defined by the teres minor, teres major, and the long head of the triceps. It is the pedicle to the parascapular and scapular flaps. Option C defines the quadrangular space that transmits the axillary nerve and posterior humeral circumflex artery.

2014 A 58-year-old man undergoes total laryngopharyngectomy for recurrent squamous cell carcinoma. The pedicle to the most appropriate flap for reconstruction of the resulting total circumferential pharyngectomy defect extending from the base of the tongue to the cervical esophagus is located between which of the following muscles? A ) Flexor carpi radialis and palmaris longus B ) Teres minor, teres major, and long head of the triceps C ) Teres minor, teres major, long head of the triceps, and humerus D ) Vastus lateralis and rectus femoris E ) Vastus medialis and rectus femoris

The correct response is Option E. The sphenopalatine nerve arises from the incisive foramen and provides sensation to the anterior hard palate. Blockade of this nerve is essential for adequate blockade of the palatal mucosa for laceration repair. The anterior superior alveolar nerve arises from the second branch of the trigeminal nerve before it exits the infraorbital foramen. The nerve supplies the maxillary anterior teeth and is part of the superior dental plexus of nerves that also includes the middle superior alveolar and the posterior superior alveolar nerves. The infraorbital nerve provides sensation to the ipsilateral lateral nose, upper lip, and cheek. The lesser palatine descends through the greater palatine foramen and provides innervation to the soft palate and uvula.

2014 A 6-year-old boy is brought to the emergency department because of a laceration of the hard palate. Repair of the laceration with local anesthesia for greater palatine nerve block is planned. As the anterior portion is sutured in place, the patient feels pain. Which of the following additional nerve blocks is most appropriate? A ) Anterior superior alveolar B ) Infraorbital C ) Lesser palatine D ) Middle superior alveolar E ) Sphenopalatine

The correct response is Option A. The deep circumflex iliac artery is the major blood supply to the iliac crest free flap. It gives rise to periosteal branches and nutrient endosteal branches that supply the iliac crest bone posterior to the anterior superior iliac spine. It also gives rise to an ascending branch that supplies the internal oblique muscle and several musculocutaneous perforators that supply the overlying skin, allowing a myo-osseous or osteocutaneous free flap to be harvested, respectively. Use of the iliac crest osteocutaneous free flap has been described by several authors for maxillary as well as mandibular reconstruction, and the bone itself provides ample stock for accommodating osseointegrated implants for dental restoration. The peroneal artery is the blood supply to the fibula free flap. Use of this flap is contraindicated when the peroneal artery contributes markedly to the blood supply of the distal lower extremity. The descending genicular artery is a branch of the superficial femoral artery and is the blood supply to the medial femoral condyle osseous free flap. Alternately, the medial superior genicular artery, another branch of the superficial femoral artery, can be used to supply this flap, but the pedicle is shorter. The descending branch of the lateral circumflex femoral artery is the blood supply to the anterolateral thigh free flap, which is a cutaneous perforator flap. The deep inferior epigastric artery is the blood supply to the rectus abdominis musculocutaneous free flap or the deep inferior epigastric perforator flap.

2014 A 60-year-old man is evaluated for a 6-cm ameloblastoma of the right maxilla. Reconstruction using an osteocutaneous iliac crest free flap is planned. Which of the following arteries supplies arterial blood to this flap? A ) Deep circumflex iliac B ) Deep inferior epigastric C ) Descending genicular D ) Lateral circumflex femoral E ) Peroneal

The correct response is Option E. When performing head and neck microsurgery, a strong background in the vascular anatomy of that region is imperative. As chemotherapy regimens have become commonplace for laryngopharyngeal cancers, so have the challenges of failures which generally require surgery. These cases have much higher complication rates, including fistulas, strictures, and carotid injury, among others. This case demonstrates another complexity that is increasingly observed, the "vascular or vessel-depleted neck." The facial and occipital arteries are branches of the external system and would not be available. The subclavian is generally not a viable option due to its size, location, and potential complications through dissection. The internal mammary system has potential, but requires dissection through the ribs and has morbidities and the potential need of vein grafts. Generally speaking, even radical neck dissections do not sacrifice the transverse cervical vessels as they are usually used as the caudal margin. A number of reports have detailed the usefulness of these vessels as recipients in cases like the one described.

2014 A 65-year-old man undergoes surgery for management of a subtotal massive squamous cell carcinoma of the posterior larynx. History includes chemoradiation and subsequent bilateral selective neck dissections for persistent disease 9 months ago. An anterolateral thigh free flap is chosen for reconstruction, but the operative notes state that both external carotid systems were sacrificed. Which of the following recipient vessels is most appropriate in this patient? A ) Facial B ) Internal mammary C ) Occipital D ) Subclavian E ) Transverse cervical

The correct response is Option C. The most appropriate current procedural terminology (CPT) code for this procedure is 14041, adjacent tissue transfer or rearrangement of cheek defect. In this case, the rhomboid flap is most accurately considered an adjacent tissue transfer. 11643, excision of a malignant lesion is not separately reportable with codes 14000-14302, and separate reporting would be considered unbundling. If the lesion were excised and closed by wide undermining, only 11643 and 13132, complex closure and excision of malignant lesion, would be used. In this case, the closure consisted of more than simple undermining and thereby closure with an advancement flap is a more appropriate choice. Excision of benign or malignant lesion, excisional preparation of a wound bed, or debridement of an open fracture or open dislocation are not included in complex repair codes. The choices of 11643 (excision of malignant lesion) or 11100 (biopsy of skin) both under-code based on the extent of the procedure in the described scenario.

2014 A 67-year-old woman comes to the office because of a 1-year history of a 2-cm basal cell carcinoma of the cheek. The lesion is excised with a 5-mm margin leaving a 3.0 × 3.0-cm defect. The wound is closed with a rhomboid flap. Which of the following is the most appropriate current procedural terminology (CPT) code for this procedure? CPT code Description 11100: biopsy of skin (including simple closure); single lesion 11643: excision, malignant lesion including margins face, 2.1 to 3.0 cm diameter 13132: repair, complex, cheek 2.6 to 7.5 cm 14040: adjacent tissue transfer or rearrangement cheeks defect 10 cm2 or less 14041: adjacent tissue transfer or rearrangement cheeks defect 10.1 cm2 to 30.0 cm2 A ) 11100 B ) 11643 C ) 14041 D ) 11643 and 13132 E ) 14040 and 11643

The correct response is Option A. The patient described has focal hemangioma. Medical therapy is the first line of treatment in this case when there are minimal ophthalmologic findings. There is a subset of periorbital hemangiomas for which surgical resection is indicated: well-localized hemangioma amenable to resection, greater than 2 diopters of astigmatism, visual obstruction, or nonresponsive to medical therapy. In a study by Arneja and Mulliken, the best ophthalmologic improvement occurred when the hemangioma was resected at less than age 3 months. Laser therapy can treat discoloration only and will not decrease the bulk of the lesion. Surgical debulking is indicated if the hemangioma does not clinically respond (softer, lightening in color, no improvement in/worsening of astigmatism).

2014 A 7-month-old male infant is referred because of the lesion shown in the photograph. Ophthalmologic examination shows mild astigmatism, for which he was being treated with corrective lenses. No amblyopia is noted. Which of the following is the most appropriate next step in management? A ) Administration of propranolol 2 mg/kg/day in divided doses B ) Magnetic resonance angiography of the brain C ) Pulsed-dye laser (595 nm) therapy D ) Surgical resection E ) Observation only

The correct response is Option E. Facial reanimation procedures using free tissue transfer have largely supplanted static procedures for pediatric facial paralysis. Although cross-facial nerve grafting (CFNG) remains an excellent option, many surgeons now prefer using the motor branch to the masseter (trigeminal nerve), since it can be done in a single stage, yields excellent muscle reinnervation, and produces muscle contraction/commissure displacement that typically exceeds that of CFNG. Although this motor branch provides innervation to the masseter, there are no reports of crossbite after its use in facial reanimation. CFNG provides relatively consistent smile symmetry and spontaneity since the stimulus for muscle contracture on both sides of the face comes from the same facial nerve source. In contrast, smile spontaneity and symmetry are much more variable when the motor masseteric branch is used and requires some cortical adjustment and/or behavioral education to develop; younger patients respond much more reliably and naturally than older patients.

2014 A 7-year-old girl with congenital palsy of the left facial nerve is scheduled to undergo facial reanimation using a free gracilis muscle neurotized by the motor branch to the masseter muscle. Compared with cross-facial nerve grafting from the contralateral facial nerve, use of this procedure is most likely to result in which of the following? A ) Decreased excursion of the reanimated oral commissure B ) Development of a crossbite C ) Increased risk of long-term muscle atrophy D ) Need for additional surgical procedures E ) Unpredictable smile symmetry

The correct response is Option C. Palatal obturators can adequately restore missing maxillary dentition as well as prevent oronasal leakage of air, liquids, and foods. They have the advantage of being removable, which permits visualization of the maxillary cavity for tumor surveillance. Prosthetic retention can be difficult or impossible in sizable defects, particularly when there are few teeth to stabilize the prosthesis. In this patient who has sufficient remaining maxillary teeth and the majority of the alveolar arch, the prosthesis is expected to have good stability, and would be the appropriate choice in a patient who wishes to avoid further invasive procedures. The temporalis muscle flap can be transposed into the oral cavity and can be used for closing defects of the palate. However, this flap alone would not provide replacement of the missing dentition and is still more invasive than a palatal obturator. Additionally, the temporalis muscle flap results in marked temporal hollowing at the donor site. The rectus abdominis musculocutaneous free flap can close the palatal defect and restore shape to the cheek in patients with a unilateral maxillectomy. In combination with a dental prosthesis, the rectus abdominis musculocutaneous free flap can restore the patient's appearance and function. However, the rectus abdominis musculocutaneous free flap is also an invasive procedure, and it can sometimes be challenging to inset the flap such that there is enough room in the mouth for a prosthesis. In a patient who has had a maxillectomy,there is generally inadequate remaining bone stock to place osseointegrated implants for prosthetic retention. The patient's existing dentition should be adequate to support a prosthesis. The fibula osteocutaneous free flap and other osteocutaneous flaps can be used to close the palatal defect to prevent nasal regurgitation. The fibula osteocutaneous free flap can also accept osseointegrated implants for dental restoration due to the good quality of bone stock associated with this flap. However, fibula free flap and osseointegrated implant reconstruction is a very long and extensive procedure and can require more than one surgery to fully restore this patient, particularly if osseointegrated implants are not placed during the same procedure as the free flap reconstruction.

2014 A 70-year-old man is evaluated following tumor resection. Physical examination shows a 4 × 4-cm defect of the right maxilla that includes all of the teeth posterior to the right canine (two premolars and three molars) but spares the right orbital floor. He did not undergo radiation therapy. He wishes to restore mastication, speech, and swallowing by the simplest means that will still be efficacious. Which of the following is the most appropriate method of reconstruction? A ) Fibula osteocutaneous free flap with osseointegrated implants B ) Osseointegrated implant-retained prosthesis C ) Prosthetic obturator D ) Rectus abdominis musculocutaneous free flap with a conventionally retained dental prosthesis E ) Temporalis muscle pedicled flap

The correct response is Option C. Karapandzic flaps are appropriate for reconstruction of defects involving one to two thirds of the lower lip, such as the one in this patient. The Karapandzic technique involves performing circumoral incisions and mobilizing the orbicularis oris muscle, while preserving its innervations and vascular supply. The main advantage of this technique is that a continuous sphincter of functional orbicularis muscle is created, helping to restore oral competence. The Estlander flap is a full-thickness, cross-lip transposition flap designed to reconstruct lateral defects of the lower lip (one to two thirds) requiring recreation of the oral commissure. Melolabial flaps can be used to reconstruct large full-thickness lower lip defects. However, they require grafting of the deep surface of the flap, have a less reliable random blood supply, and do not provide a functional muscular oral sphincter. The submental artery island flap is based on the submental branch of the facial artery. A paddle of skin, subcutaneous tissue, and fascia harvested from the submental area can be used for coverage of lower face and preauricular defects, as well as inferior and lateral neck wounds. Its use for reconstruction of partial lower lip full-thickness defects has not been established. Facial artery myomucosal flaps consist of oral mucosa, submucosa, a small amount of buccinator muscle, and a more deeply lying facial artery and venous plexus. They are ideal for reconstructing the inner, most lip mucosa because they consist of similar tissue, with the same color, texture, and moisture. They can also be used for reconstruction of the dry vermilion, although some drying-out and scabbing of the mucosa will occur. They are not indicated for large, full-thickness lower lip defects.

2014 A 72-year-old man undergoes wide local excision of a squamous cell carcinoma of the lower lip. Margins are free of involvement. A photograph of the resulting defect is shown. Reconstruction using which of the following flaps is most appropriate in this patient? A ) Estlander B ) Facial artery myomucosal C ) Karapandzic D ) Melolabial E ) Submental artery island

The correct response is Option E. Syndromic ear anomalies are associated with an increased risk of renal anomalies in syndromes such as brachio-oto-renal syndrome, Townes-Brocks syndrome, etc. Cardiac abnormalities are not associated with isolated microtia, but they are associated with extended spectrum hemifacial microsomia (oculoauriculovertebral dysplasia). Magnetic resonance angiography of the brain to rule out internal cerebral artery anomalies is indicated in children suspected of PHACE syndrome (P, posterior fossa; H, hemangioma; A, arterial anomalies; C, cardiac defects; E, eye anomalies) when clinical symptoms include a large segmental hemangioma. Tethered cord is not associated with ear anomalies. It is a concern with myelomeningoceles and spina bifida, and with Chiari malformations. Limb-length discrepancies are not associated with microtia.

2014 A female newborn is evaluated after an uneventful delivery because of microtia of the left ear. The face appears otherwise symmetrical. Which of the following studies is most appropriate to obtain? A ) CT scan to rule out tethered cord B ) Echocardiography to rule out ventricular septal defect C ) Extremity x-ray studies to rule out limb-length discrepancies D ) Magnetic resonance angiography of the brain to rule out ipsilateral cerebral artery anomalies E ) Ultrasonography of the kidney to screen for structural anomalies

The correct response is Option B. This child has aplasia cutis congenita, or cutis aplasia, of the scalp. First described in 1767 by Cordon, cutis aplasia is the congenital absence of all skin layers including the epidermis, dermis, and subcutaneous fat. This process most commonly affects only focal areas of tissue but involvement can be extensive. The majority of cases involve the scalp, but this process can occur in any cutaneous area of the body. Cutis aplasia can occur in isolation or as part of a syndrome, the most common being Adams-Oliver syndrome. Cutis aplasia of the scalp can range from small areas of involvement that often heal in utero and appear at birth as a "congenital scar" to massive defects that are devoid of scalp and cranium. Most small- or intermediate-sized full-thickness defects heal quickly (as in the patient described) if kept moist and the resultant scar can be excised secondarily. Bone healing is often complete in small lesions, and residual defects can be reconstructed when the child is older if needed. Large areas are more problematic and extensive scalp defects that threaten dural integrity may require early operative intervention. Cutis aplasia involving large areas of the scalp has a reported mortality ranging from 20 to 55%, typically as a result of sagittal sinus hemorrhage or associated congenital defects. In such cases, coverage of the dura can be life-saving. Described methods of soft-tissue coverage include skin graft, cultured allograft, acellular dermis, and immediate or delayed reconstruction with a flap. Tissue expansion of the scalp in a newborn presents many challenges and is not recommended.

2014 A male newborn is evaluated because of the scalp anomaly shown in the photograph. Which of the following is the most appropriate initial management of the affected area? A ) Application of a skin substitute B ) Local wound care with antibiotic ointment C ) Primary closure D ) Skin grafting E ) Tissue expansion

The correct response is Option B. The newborn described has CHARGE (coloboma of the eye, heart defects, atresia of the nasal choanae, retardation of growth and/or development, genital and/or urinary abnormalities, and ear abnormalities and deafness) syndrome. Bicoronalcraniosynostosis is associated with Crouzon, Apert, and Pfeiffer syndromes, which do not include all of the other defects. Radial hypoplasia is associated with VACTERL (vertebral defects, anal atresia, cardiac defects, tracheo-esophageal fistula, renal anomalies, and limb anomalies) syndrome. Syndactyly can be seen in many conditions including Down syndrome, Apert syndrome, and Carpenter syndrome. Webbing of the neck can be commonly seen in Noonan syndrome, Klippel-Feil syndrome, and Turner syndrome.

2014 A newborn in the neonatal intensive care unit is small for gestational age and has choanal atresia, cryptorchidism, a ventricular septal defect, and abnormal external ear framework. Prenatal findings included growth retardation with poor fetal movement. Which of the following additional characteristics is most likely in this newborn? A ) Bicoronal craniosynostosis B ) Coloboma C ) Radial hypoplasia D ) Syndactyly E ) Webbing of the neck

The correct response is Option C. Oculoauricular dysplasia (Goldenhar syndrome) may have many different possible manifestations, but is typically characterized by ear anomalies, epibulbar dermoids, facial and mandibular hypoplasia, and vertebral anomalies. Cleft palate, although it may be encountered in patients with Goldenhar syndrome, is not generally regarded as a classic finding and is less common than microtia in affected patients. Lower lip pits are a finding suggestive of van der Woude syndrome, the most common orofacial clefting syndrome, rather than Goldenhar syndrome. Syndactyly is not generally regarded as a component of Goldenhar syndrome. Telecanthus is not a classic finding in patients with Goldenhar syndrome.

2014 An 8-hour-old male newborn is examined in the neonatal intensive care unit for epibulbar dermoids. X-ray studies show fused cervical vertebrae. Which of the following additional features is most consistent with this diagnosis? A ) Cleft palate B ) Lower lip pits C ) Microtia D ) Syndactyly E ) Telecanthus

The correct response is Option C. Craniosynostosis refers to the premature fusion of one or more cranial sutures that make up the cranial vault and cranial base. Once this fusion occurs prematurely, the growth of the skull is altered and the development of the head takes on a characteristic morphologic shape that is determined by the fusing suture. Trigonocephaly is classically characterized by a typically wedge-shaped skull when viewed from above; it originates from a premature stenosis of the metopic suture followed by a bilateral growth restriction of the forehead. This results in bitemporal narrowing and hypotelorism. Plagiocephaly or unilateral coronal synostosis is characterized by the flattening of the forehead and frontoparietal region ipsilateral to the fused suture. As a result of this fusion, a compensatory bulge occurs in the opposite frontoparietal skull. The temporal fossa on the side of the fusion is convex and the ear becomes anteriorly displaced. The petrous portion of the temporal bone that contains the glenoid fossa is also displaced forward and the articulation with the mandible is displaced forward as a result. The nasal radix is also deviated toward the fused side and the tip of the nose is turned to the opposite side. Ridging of the sagittal suture forms a narrow biparietal skull. Scaphocephaly shows compensatory growth in the frontal region or frontal bossing and/or occipital coning. There is associated enlargement of the head circumference. Sagittal synostosis remains the most frequent of the nonsyndromic craniosynostosis. Unilateral lambdoid synostosis has ridging of the lambdoid suture, ipsilateral parieto-occipital flattening, prominence of the mastoid air cells, posterior displacement of the ear on the side of the occipital flattening, and scoliosis of the base of the skull, resulting in curvature of the cervical spine.

2014 An 8-month-old female infant is brought to the office by her parents. Physical examination shows a wedge-shaped skull with a keel formation on the forehead, close-set eyes, and hollowness of the temporal fossa on both sides of the head. Premature cranial suture ossification at which of the following sites is the most likely cause of this patient's condition? A ) Bicoronal B ) Lambdoid C ) Metopic D ) Sagittal E ) Unicoronal

The correct response is Option A. The most appropriate method of reconstruction for this patient with a moderate-sized full-thickness skin and soft-tissue defect is a bilobed flap. This technique will cover the defect with existing nasal skin providing the best color match and tissue thickness. Although bilobed flaps do have a fair amount of scarring, these incisions typically heal well. A split-thickness skin graft is not an ideal choice for nasal tip reconstruction, as this option is typically too thin to match the surrounding skin resulting in a depressed scar. Furthermore, the color match is usually not optimal. A forehead flap would be a useful technique for larger defects; however, this operation would require two trips to the operating room and is excessive for a moderate-sized defect such as described. The V-Y advancement flap is not a good choice for nasal tip defects, as it is difficult to reach the defect from the surrounding tissues and advancement results in marked distortion. A nasolabial flap is a good choice for defects of the ala but requires two operations for the nasal tip (flap transfer followed by sectioning and inset) and is therefore suboptimal compared with the bilobed flap.

2014 An otherwise healthy 50-year-old woman is referred 1 hour after Mohs micrographic surgery. The margins are clear. Physical examination shows a 1.4-cm full-thickness skin and soft-tissue defect of the nasal tip. Which of the following is the most appropriate method of reconstruction in this patient? A ) Bilobed flap B ) Split-thickness skin graft C ) Nasolabial flap D ) Paramedian forehead flap E ) V-Y advancement flap

The correct response is Option E. Cleft lip and cleft palate are common birth defects that result from a variety of genetic and environmental factors. On average, they occur in 1.7 of every 1000 live-born babies. The development of the lip and palate in utero involves a complex series of steps that involve cell migration, proliferation, and apoptosis. During the fourth week of gestational development, neural crest cells migrate to the developing craniofacial region of the embryo, where they help in the formation of the frontonasal prominence, the mandibular processes, and the maxillary prominences. Nasal placodes divide the lower portion of the frontonasal prominence into the paired medial and lateral nasal processes. During weeks 6 to 8 of gestation, fusion of the medial nasal prominences with each other and with the maxillary processes forms the upper lip and primary palate. Therefore, Option E is the most appropriate answer regarding the etiology of the baby's cleft of the lip. Option D is not appropriate because it states that the failure of fusion occurs during weeks 9 to 11, which is too late in embryological development. Options A and B are not appropriate because fusion of the frontonasal prominence with the maxillary prominences forms the primary palate. Option C describes the embryologic formation of an oblique cleft, and the time frame is not appropriate.

2014 Failure of fusion of which of the following results in the formation of a cleft of the lip? A ) Frontonasal and maxillary prominences during the first 4 to 5 weeks of gestation B ) Frontonasal and maxillary prominences during the first 9 to 11 weeks of gestation C ) Lateral nasal and maxillary prominences during the first 2 to 4 weeks of gestation D ) Medial nasal and maxillary prominences during the first 9 to 11 weeks of gestation E ) Medial nasal and maxillary prominences during the first 6 to 8 weeks of gestation

The correct response is Option A. A zygoma fracture involves displacement of the zygoma that articulates with the frontal bone, maxilla, and sphenoid. In order to stabilize the fracture after adequate reduction, the zygomaticofrontal, zygomaticomaxillary buttress, and infraorbital rim need to be stabilized. If there is a large (>2 cm2) defect in the orbital floor after reduction, reconstruction of the orbital floor is also necessary to prevent enophthalmos. Although the nasomaxillary buttress is one of the vertical buttresses of the face, the zygoma does not articulate with the nasal bones.

2014 Proper reduction of an isolated zygoma fracture requires reduction and realignment of which of the following? A ) Zygomaticofrontal suture, zygomaticomaxillary buttress, and infraorbital rim B ) Zygomaticofrontal suture, zygomaticomaxillary buttress, and orbital floor C ) Zygomaticofrontal suture, zygomaticonasal suture, and infraorbital rim D ) Zygomaticomaxillary buttress, infraorbital rim, and nasomaxillary buttress E ) Zygomaticomaxillary buttress, orbital floor, and alveolus

The correct response is Option D. Neural crest cells derive from the ectoderm layer of the developing embryo, specifically the neuroectoderm of the forebrain, midbrain, and hindbrain. The neural crest contributes significantly to the craniofacial region, and also to the conotruncal endocardial cushions that are responsible for dividing the outflow tract of the heart into separate pulmonary and aortic components. Therefore, defects in neural crest cell development will frequently result in malformations of both the craniofacial area and cardiac septum. Examples of this association are: Treacher Collins syndrome, Pierre Robin sequence, 22q11.2 deletion syndrome, and oculoauriculovertebral syndrome. The cardiogenic mesoderm derives from splanchnic (visceral) mesoderm, and contributes the precursor cells that differentiate into the endocardium and myocardium. It does not contribute to the head and neck. Ectodermal placodes are separate from the neuroectoderm and consist of areas of thickened ectoderm that guide neural crest cells in forming the cranial sensory ganglia. Lateral plate mesoderm is a derivative of the mesoderm layer of the embryo and contributes to the laryngeal cartilages and associated connective tissue of the head and neck. Paraxial mesoderm produces somites that form the skull, meninges, voluntary craniofacial musculature, and dermis and connective tissue of the dorsal aspect of the head.

2014 The association between craniofacial defects and cardiac malformations in patients with velocardiofacial syndrome results from a disruption in the cellular development of which of the following? A ) Cardiogenic mesoderm B ) Ectodermal placodes C ) Lateral plate mesoderm D ) Neural crest E ) Somites

The correct response is Option D. Poly-L-lactic acid, marketed as Sculptra, has been utilized for the treatment of HIV retroviral drug-related lipoatrophy since 1999. The images shown illustrate such a patient. Since its initial use, the indications for utilization have broadened and include most etiologies of lipoatrophy with its second most common use for age-related changes. In patients with age-related facial changes, it has been useful for mid face rejuvenation and temporal hollowing. Similar to age-related changes is weight loss that, when mild, will also respond to poly-L-lactic acid but when severe, will require a rhytidectomy. Lipoatrophy secondary to discoid lupus has been reported to respond best to fat grafting. The least likely to respond to poly-L-lactic acid is progressive hemifacial atrophy due to the severity of the soft-tissue deformity and will usually require soft-tissue augmentation with a free flap in severe cases and fat grafting in mild cases.

2014 Treatment with poly-L-lactic acid (Sculptra) is most likely to correct the soft-tissue facial deformity associated with which of the following conditions? A ) Discoid lupus B ) Progressive hemifacial atrophy C ) Scleroderma D ) Secondary effects of HIV treatment

The correct response is Option D. Furlow double-opposing Z-plasty is an effective method of treating velopharyngeal insufficiency associated with submucous cleft palate or following conventional push-back palatoplasty procedures. Several studies suggest that the size of the preoperative velopharyngeal gap, as determined by preoperative nasendoscopy, is the most important determinant of velar competence after Furlow palatoplasty. Thus, patients with a smaller preoperative maximal closure gap were more likely to have a competent velopharyngeal sphincter postoperatively. The procedure has also been reported to be less effective in older children, in overt (versus submucous) clefts, and in patients with certain syndromes such as velocardiofacial syndrome. Nevertheless, these variables are not as important as the preoperative gap. The gender of the patient and compliance with speech therapy do not influence outcomes.

2014 Velar competence after treatment of velopharyngeal insufficiency with Furlow double-opposing Z-plasty is most strongly correlated with which of the following? A ) Age at the time of procedure B ) Compliance with speech therapy C ) Patient gender D ) Preoperative closure gap E ) Type of cleft

The correct response is Option D. The two cephalometric planes used most frequently in lateral cephalograms to describe and evaluate the cranial base are the Sella-nasion plane and the Frankfort horizontal plane. The Frankfort horizontal plane is defined by a line from the superior edge of the external auditory meatus (porion) to the inferior orbital meatus (orbitale). The SNA and SNB are angles used to describe the position of the maxilla and mandible, respectively. The SN refers to a line from the sella turcica to the nasion, while point A is on the maxilla and point B is on the mandible. The pogonion refers to the chin point.

2014 Which of the following cephalometric landmarks is included in the Frankfort horizontal plane? A ) Nasion B ) Pogonion C ) Point B D ) Porion E ) Sella turcica

The correct response is Option D. Innervation of the parotid gland comes from parasympathetic fibers that travel with the glossopharyngeal nerve (cranial nerve IX). It also receives taste sensation (afferent) from the posterior one-third of the tongue. The maxillary nerve of cranial nerve V (V2) is a sensory nerve and receives sensation from the mid face. Parasympathetic fibers (efferent) innervate the submandibular and sublingual glands via the chorda tympani. Afferent fibers, via the chorda tympani, send taste sensation of the anterior two-thirds of the tongue. The vestibulocochlear nerve (cranial nerve VIII) supplies sound and equilibrium to the brain. The auricular branch of the vagus nerve (cranial nerve X), innervates the external acoustic meatus. Stimulation of the vagus nerve can lead to reflex coughing (Arnold reflex).

2014 Which of the following cranial nerves provides parasympathetic innervention of the parotid gland? A ) V B ) VII C ) VIII D ) IX E ) X

The correct response is Option E. The most common cause of temporomandibular joint (TMJ) ankylosis is trauma. It usually occurs after untreated or inadequately treated mandibular fractures. Damage to the articular surface of the TMJ is the most common factor seen. In children, this can lead to growth disturbances ultimately requiring orthognathic surgery. Otherwise, joint replacement and repair may be indicated in adults. In the antibiotic era, infection is a rare cause. Congenital anomalies, bruxism, and radiation are less common.

2014 Which of the following is the most common cause of temporomandibular joint ankylosis? A ) Bruxism B ) Congenital anomaly C ) Infection D ) Radiation E ) Trauma

The correct response is Option C. The first branchial arch, also known as the mandibular arch, has the trigeminal nerve (ophthalmic, maxillary, and mandibular branches) as its neurologic component. The muscles of mastication (i.e., temporalis, masseter, medial, and lateral pterygoids); mylohyoid; anterior digastric; tensor tympanic; and the tensor veli palatini are the muscle components. The cartilaginous bar gives rise to the premaxilla; maxilla, zygomatic bone; part of the temporal bone; incus; malleus; anterior malleolar ligament; and the sphenomandibular ligament. The pharyngeal pouch and groove develop the tubotympanic recess (tympanic cavity, mastoid antrum and pharyngotympanic tube, internal acoustic meatus, tympanic membrane, adenoids). The vascular element largely disappears, but gives rise to the maxillary and external carotid arteries. The second branchial arch, also known as the hyoid arch, accounts for 95% of all branchial arch anomalies. The cranial nerve is the facial nerve. It supplies the muscles of facial expression; buccinators; stapedius; stylohyoid; posterior digastric; auricular and platysma muscles. The skeletal contributions from Reichert's cartilage include the stapes, styloid process, stylohyoid ligament, and hyoid (lesser cornu and upper part of body). The pharyngeal pouch and groove shape the crypts of the palatine tonsil and the cervical sinus. The vascular component again primarily disappears but forms the stapedial and hyoid arteries. Third branchial cleft anomalies are rare. The glossopharyngeal nerve sends motor innervation to only the stylopharyngeus. The cartilaginous bar forms the hyoid (greater cornu and lower part of body). The pharyngeal pouch and groove give rise to the inferior parathyroids, thymus, and cervical sinus. The vascular elements contribute to the internal carotid and common carotid. The fourth branchial arch is supplied by the vagus nerve (superior laryngeal, inferior laryngeal). Musculature innervated includes the cricothyroid and all intrinsic muscles of the soft palate, including the levator veli palatini. The thyroid and epiglottic cartilage develop from the cartilaginous bar. The pharyngeal pouch and groove form the superior parathyroids, and the thyroid parafollicular cells. The right fourth aortic arch forms the subclavian artery, while the left fourth aortic arch forms the aortic arch. The sixth branchial arch also is supplied by the vagus nerve (recurrent laryngeal nerve). This area supplies all intrinsic muscles of the larynx (except the ciricothyroid-fourth arch). This includes the thyroarytenoid muscle, which makes up the primary mass of the vocal fold. It consists of two parts, the ventricularis and vocalis. Skeletal derivations form the cricoid, arytenoid, corniculate, and cuneiform cartilages. The right sixth aortic arch gives rise to the right pulmonary artery and the left sixth aortic arch forms the left pulmonary artery and the ductus arteriosus.

2014 Which of the following muscles is associated with the hyoid or second branchial arch? A ) Lateral pterygoid B ) Levator veli palatini C ) Posterior digastric D ) Stylopharyngeus E ) Thyroarytenoid

The correct response is Option C. Nasopharyngeal cancers are most often associated with Epstein-Barr virus (EBV) infections and arise from the mucous epithelium of the nasopharynx and are relatively rare in the United States. However, these tumors are endemic in Africa and East Asia, accounting for as many as 18% of head and neck cancers in China. Nasopharyngeal tumors are classified as either squamous cell cancers, keratinizing undifferentiated carcinoma, or non-keratinizing undifferentiated carcinoma. EBV infection is most strongly associated with the non-keratinizing undifferentiated subtype and is thought to increase malignant transformation. Nasopharyngeal cancers are most commonly treated with chemotherapy and radiation, with surgery reserved for recurrent or unusual cancers. Reconstruction of skull base defects is most commonly performed using microsurgical transfer of soft-tissue flaps. Alcohol and tobacco are the most common risk factors for head and neck cancers in general, and laryngeal cancers in particular, with cigarette smoking increasing the lifetime risk 5- to 25-fold. Other risk factors for head and neck cancers in general include cigar smoking, environmental exposures, dietary factors (red meat, betel nuts), and human papillomavirus (HPV) infection. HPV infections are most commonly associated with oropharyngeal cancers (tongue, tonsil). Significant risk factors for maxillary sinus cancers include cigarette smoking and environmental factors such as exposure to wood dust.

2014 Which of the following types of head and neck tumors are most often associated with Epstein-Barr virus infection? A ) Larynx B ) Maxillary sinus C ) Nasopharynx D ) Tongue E ) Tonsil

The correct response is Option D. Frey syndrome occurs with injury and abnormal regeneration of the auriculotemporal nerve, but is a late complication (median time at presentation: 11 months). Postoperative facial nerve dysfunction can occur in up to 60% of patients, but the majority (90%) resolve without need for operative intervention. Tinnitus is not a recognized complication of superficial parotidectomy, and hematoma is an early complication (<24 hours). Patients with sialadenitis alone have increased risk for developing salivary fistulae.

2017 Which of the following findings is most likely in patients who undergo superficial parotidectomy for treatment of sialadenitis? A) Frey syndrome at 3 months postoperatively B) Hematoma after the first 24 hours postoperatively C) Permanent postoperative facial nerve dysfunction D) Salivary fistulae E) Tinnitus

The correct response is Option A. Syndromic craniosynostosis often presents with findings in the hands and feet, sometimes referred to as acrocephalosyndactyly. Apert syndrome is unique for having bilateral symmetric complex syndactylies involving nearly all digits, of both the hands and feet. Crouzon syndrome usually has normal hands and feet. Jackson-Weiss can have foot anomalies, such as short metatarsals. Pfeiffer syndrome usually has broad thumbs and toes. Saethre-Chotzen syndrome may have incomplete single syndactylies, but otherwise does not typically have extremity findings.

2017 A 16-month-old male infant is brought to the physician because of congenital anomalies of both feet. The dorsal and plantar aspects are depicted in the photographs shown. Which of the following is the most likely diagnosis? A) Apert syndrome B) Crouzon syndrome C) Jackson-Weiss syndrome D) Pfeiffer syndrome E) Saethre-Chotzen syndrome

The correct response is Option B. This patient has a fracture of the tooth crown that extends through the dental enamel into the deeper parts of the tooth. This is evidenced by the sensitivity to touch and cold, a finding not characteristic of a fracture limited to the enamel. The yellow color to the base of the fracture indicates exposed dentin, which resides just under the hard outer enamel layer of the tooth. If the fracture had extended deeper into the pulp cavity, the area where the vessels and nerves reside, the base of the fracture would appear as a blood-filled cavity. These injuries often challenge the viability of the tooth and often require drilling and packing of the pulp space (root canal). The fracture described is of the crown and there is no indication that it involves the root of the tooth or the surrounding structures. Cementum is a bone-like covering of the tooth root and would not be affected by this injury. The Ellis classification provides a useful system of categorizing these injuries. There are 9 categories: Ellis I: enamel fracture. The tooth is non tender and treatment is smoothing of the rough surfaces and, possibly, application of a filling or amalgam. Ellis II: fracture of the enamel and dentin. Tooth is tender to air, cold, and probing and the base of the defect often appears yellow. Ellis III: involves the enamel, dentin, and the pulp space. The tooth is sensitive as in Ellis II, but the base of the defect appears red or bloody. Ellis IV: a nonviable tooth. Ellis V: luxation of the tooth. Ellis VI: tooth avulsion. Ellis VII: displacement without fracture. Ellis VIII: fracture of entire crown. Ellis IX: fracture of deciduous teeth.

2017 A 10-year-old boy is brought to the physician after sustaining a nondisplaced fracture of the mandibular body in a fall. Soft diet is recommended. Two days later, he is brought back to the office and reports pain in the right mandibular lateral incisor when drinking cold liquid. The base of the defect appears yellow and is tender when probed. Examination shows a lingual fracture of the tooth crown. On the basis of these findings, which of the following is the deepest layer of exposed tooth? A) Cementum B) Dentin C) Enamel D) Pulp cavity E) Root canal

The correct response is Option B. The relationship between morphea en coup de sabre and Parry-Romberg syndrome is unclear but there is some overlap. Morphea is characterized by the vertical furrow, atrophy of the tongue and upper lip, absent or flattened zygoma, orbital rim, and a hypoplastic maxilla and mandible on the affected side. A lateral open bite may be seen due to the maxillary and mandibular hypoplasia. Sensation, function of muscles of facial expression, and mastication are normal.

2017 A 12-year-old boy is evaluated for a vertical furrow near the midline of his face from the hairline to the eyebrows. Each of the listed clinical findings can be seen in hemifacial atrophy EXCEPT A) Atrophy of the tongue B) Change in facial sensation C) Malar hypoplasia D) Malocclusion E) Mandible hypoplasia

The correct response is Option A. In patients with hereditary hemorrhagic telangiectasia (HHT), also known as Osler-Weber-Rendu disease, characteristic abnormal arteriovenous shunting is noted in the mucosae of the naso- and oropharynx, and pulmonary, GI/hepatic, and CNS systems. They are at risk for bleeding and anemia, and even stroke (CNS manifestation). Several genes have been associated with HHT, including the endoglin gene, ENG. ACVRL1 and SMAD4 may also be associated. RASA1 mutation has been associated with capillary malformations with or without AVMs. Oval, macular port-wine staining on the skin is a common finding. PTEN mutation has been associated with Bannayan-Riley-Ruvalcava syndrome, an autosomal dominant condition that presents with macrocephaly, genital lentiginosis (speckled penis), and GI polyps. A subset of patients may develop arteriovenous anomalies (arteriovenous malformations and arteriovenous fistulae). KRIT1 mutation is an autosomal dominant condition associated with cavernous malformation in the brain (venous malformation, no fast-flow component). Affected patients are at risk for cerebral hemorrhage. Cutaneous manifestations are hyperkeratotic vascular malformations (slow-flow malformations). CLOVES (congenital lipomatous overgrowth, vascular malformations, epidermal nevi, spinal/skeletal anomalies/scoliosis syndrome) syndrome can occur as a result of a mutation in PIK3CA. It is an overgrowth syndrome where vascular malformations can also occur. Patients with CLOVES syndrome should have a spinal MRI to screen for CNS AVM. They may also have other slow-flow vascular malformations.

2017 A 14-year-old boy is brought to the office for evaluation because of recurrent, severe nosebleeds that require visits to the emergency department. Dermatologic examination shows no skin discolorations. Neurologic examination shows no abnormalities. Family history includes frequent nosebleeds. A diagnosis of hereditary hemorrhagic telangiectasias (HHT) is suspected. A mutation of which of the following genes is most likely in this patient? A) ENG B) KRIT1 C) PIK3CA D) PTEN E) RASA1

The correct response is Option B. Although minor salivary gland tumors are much less common than major salivary gland tumors, minor salivary gland tumors are much more likely to be malignant. Additionally, pediatric salivary cancers represent only about 5% of all salivary cancers, but are also more likely to be malignant (almost 50% were malignant in Armed Forces Institute of Pathology series of 168 pediatric salivary gland tumors). Finally, the palate is the most common source of minor salivary gland tumors, which are more likely to be malignant and higher stage when detected. The clinical presentation of paresthesias of adenoid cystic carcinoma (ACC) suggests perineural invasion. One series (University of Maryland) of 243 minor salivary gland tumors found 78% of them were malignant, and of those malignant tumors, 15% were ACC. Given the perineural invasion symptoms, imaging, in particular MRI, can detect perineural invasion and help plan the degree of surgery. In this scenario, clinical exam pointed to perineural invasion, which should be imaged to plan for surgery. Chemotherapy is not used in the treatment of this disease. Radiation therapy alone is not usually performed, as this is considered a surgical disease. However, it can be used as adjuvant therapy in addition to surgery. Excisional biopsy usually recommends 1- to 2-cm margins. Patients with high-stage, perineural invasion, lymphadenopathy, or other signs of extensive disease may receive surgery with adjuvant radiation therapy. Regardless, ordering a CT scan and MRI is a reasonable initial approach before surgical treatment.

2017 A 14-year-old girl is brought to the office because of a 1-month history of a painful, growing lesion in the hard palate with "electric-shock sensations" on palpation. Results of incisional biopsy show adenoid cystic carcinoma of the minor salivary glands. Which of the following is the most appropriate next step in management of this patient? A) Chemotherapy B) CT scan and MRI C) Excision with 1-cm margins D) Excision with 2-cm margins E) Radiation therapy

The correct response is Option E. This patient has a metopic ridge and would not be correctly classified as having the phenotype associated with pathologic metopic closure—trigonocephaly. Consequently, observation is the only correct answer. Thickening of the metopic suture is a normal variant and should not be interpreted as abnormal unless it is accompanied by frontal narrowing and retrusion of the superior-lateral orbital rims. These findings are not present in this patient. The metopic suture closes normally within the first year of life, so the presence of a fused metopic suture on CT scan in a child this age is not necessarily abnormal. The degree of frontal narrowing required to classify a patient as having pathologic cranial shape is a matter of much debate and is not settled. Some authors point to the importance of additional clinical (hypotelorism, biparietal widening) or radiographic (endocortical thickening or omega sign on CT) findings to secure the diagnosis of metopic craniosynostosis. The other responses are surgical interventions and should be invoked only in the context of pathologic metopic closure and resultant trigonocephaly. Endoscopic suturectomy and postoperative helmet therapy, spring-mediated frontal distraction, and fronto-orbital advancement are all viable options to correct the frontal narrowing associated with trigonocephaly. Total calvarial remodeling is rarely if ever required to correct trigonocephaly.

2017 A 2-year-old boy is referred for evaluation of an abnormal frontal prominence that his parents report has become more noticeable during the past year. A photograph is shown. He is otherwise healthy and is meeting all developmental milestones. A CT scan obtained at the request of his pediatrician shows closure of the metopic suture with ectocortical thickening; no other abnormalities are noted. Which of the following is the most appropriate next step in management? A) Endoscopic suturectomy and postoperative helmet therapy B) Fronto-orbital advancement C) Spring-mediated frontal distraction D) Total calvarial remodeling E) Observation

The correct response is Option C. One of the most important causes of failure of closed reduction to address the nasal fracture is simultaneous nasoseptal fracture. Murray, et al. found 30 to 40% residual nasal deformity after closed reduction. The cadaver study showed failure consistently associated with a C-shaped nasoseptal deviation and fracture when the external nose deviated at least 1/2 of the nasal bridge width. The theory is that the interlocking of the septal fracture creates tension causing the nasal bone to displace. Untreated septal hematoma results in thickening of the cartilage and nasal obstruction, but not with inadequate reduction. Nasal casting for 7 to 10 days is sufficient to allow the reduction to set. Nonunion is a rare cause of inadequate reduction, usually in comminuted or open nasal fractures. Turbinate hypertrophy can cause nasal obstruction but does not interfere with nasal bone reduction.

2017 A 22-year-old man comes to the office because of a history of nasal trauma with resultant nasal deformity, C-shaped septal fracture, and nasal obstruction. Two weeks after injury, he undergoes closed reduction of the nasal fractures, but significant nasal obstruction persists. Which of the following is the most likely reason for his residual nasal deformity and nasal obstruction? A) Inadequate time of nasal casting B) Nonunion of the nasal bones C) Presence of a septal fracture D) Turbinate hypertrophy E) Unidentified septal hematoma

The correct response is Option B. The patient described has typical radiographic and physical exam findings of a right subcondylar mandibular fracture. Anteromedial displacement of the condylar segment out of the glenoid fossa occurs secondary to pull from the lateral pterygoid muscle, which normally functions in anterior translation of the condyle across the articular eminence of the temporal bone during wide mouth opening. This leads to loss of height of the mandibular ramus and a premature occlusion on the fracture side. This causes the typical findings of condylar/subcondylar fractures: ipsilateral crossbite and contralateral open bite. The masseter and medial pterygoid form the pterygomasseteric sling, which attaches from the skull base and zygoma to the inferior mandibular border, and is responsible for fracture displacement after angle and body fractures. The mylohyoid and genioglossus muscles run along the floor of the mouth and can contribute to fracture displacement in the body and parasymphyseal region.

2017 A 27-year-old man is admitted to the emergency department after being injured in an altercation. Physical examination shows objective malocclusion with a left-sided crossbite and right-sided open bite. A CT scan is shown. Which of the following muscles is most likely responsible for these radiographic and physical examination findings? A) Genioglossus B) Lateral pterygoid C) Masseter D) Medial pterygoid E) Mylohyoid

The correct response is Option E. A medial canthal degloving injury may or may not be associated with a naso-orbito-ethmoid fracture. In this scenario, CT scan showed no fractures. These injuries can present as vertical lacerations from the forehead to the cheek, crossing the medial canthus. They usually present with the triad of telecanthus (from degloving of the medial canthal tendon, usually the posterior limb that is the weakest limb), ptosis (from avulsion or injury to the upper eyelid), and epiphora (from lacrimal or canalicular injuries). One suggested algorithm is to stage the initial repair of the telecanthus and lacrimal/canalicular repair, and then allow 3 to 6 months of healing before a second stage of ptosis repair. The other choices are not as consistent with medial canthal degloving: miosis, ptosis, and anhidrosis are incorrect, as these are the triad in Horner syndrome. Exophthalmos, ptosis, and ophthalmoplegia may be seen in superior orbital fissure syndrome. Lagophthalmos, ptosis, and eyes not closing at night may be more associated with congenital ptosis. Strabismus and ptosis, which worsen at night, are suggestive of myasthenia gravis.

2017 A 30-year-old man comes to the physician 2 months after sustaining a deep laceration from forehead to cheek with medial canthal degloving in a motor vehicle collision. His initial workup included a CT scan that showed no facial fractures; the laceration was primarily repaired in the emergency department. Today, his vision is the same as his pre-injury vision. Which of the following is the triad of clinical sequelae that this patient is most likely to report today? A) Exophthalmos, ptosis, and ophthalmoplegia B) Lagophthalmos, ptosis, and eyes not closing at night C) Miosis, ptosis, and anhidrosis D) Strabismus and ptosis that worsen at night E) Telecanthus, ptosis, and epiphora

The correct response is Option E. The primary goal of nasoalveolar molding (NAM) is to reposition and approximate the alveolar segments and reshape the nasal cartilage. Financial analysis shows that NAM costs significantly less than lip adhesion, but this a labor-intensive process for the family. It is not designed to address palatal collapse. There is anecdotal information on impact of feeding, but this is not the principal goal as customized bottles work well in these children. There is no impact on midface growth in either place based on studies with more than 18 years, follow-up. Finally, the biggest benefit of NAM in the bilateral cases is the correction in columella height-even more than in unilateral cases.

2017 Which of the following best describes the principal goal of using the nasoalveolar molding (NAM) appliance? A) Decrease of the financial and care burden on the family B) Improvement in feeding C) Improvement in midface growth in the sagittal and vertical planes D) Palatal expansion E) Repositioning and approximation of the alveolar segments and nasal cartilage

The correct response is Option C. Chemodenervation is a common treatment for ocular-oral synkinesis and perhaps the most effective. The chemodenervation medications have unknown effects for pregnant or nursing women. Botulinum toxin type A is a class C drug. Permanent surgical selective denervation is not recommended as it could impact eye protection. Surgery may also have risks for the fetus. This patient would benefit from a physical therapy referral to work on facial neuromuscular re-education, including biofeedback using mirrors and electromyography. Additional strategies such as use of sunglasses and other strategies to avoid squinting can be helpful. These are likely the treatment modalities that this patient is most interested in. Gold weight insertion would not be appropriate in a patient with symptoms of squinting.

2017 A 30-year-old primigravid woman at 24 weeks' gestation, who has a history of Bell palsy, has synkinesis and squinting of the left eye when smiling. She wants to know her treatment options, but is not interested in options that may put her pregnancy at risk or impact her goal of breast-feeding for 1 year after delivery. Which of the following treatment options is most appropriate for this patient? A) Chemodenervation to the left orbicularis oculi B) Chemodenervation to the left orbicularis oris C) Facial neuromuscular retraining D) Gold weight to the left upper eyelid E) Selective neurolysis to the temporal branch of the facial nerve facial nerve

The correct response is Option A. One of the determinants of beauty is facial shape. For a woman, a heart-shaped face is considered youthful and attractive. With a history of bruxism, masseter hypertrophy changes the face into a more square, masculine shape. Botulinum injections can reduce the facial bulk, redefine the lower facial width, and make the face more youthful and heart-shaped. The other surgical options are associated with significant morbidity and may not give the desired shape. Orthodontic care is used for patients whose malocclusion has altered the facial shape.

2017 A 32-year-old woman with bilateral masseter hypertrophy comes to the physician because she wants to have a more heart-shaped face. She has normal occlusion otherwise. Which of the following is the most appropriate initial treatment option for this patient? A) Injection of botulinum toxin type A 30 to 50 U into the area of the masseter bulk B) Intraoral debulking of the deep masseteric muscles C) Sagittal split of the mandible with retrusion D) Selective denervation of the masseter muscles E) Use of an orthodontic dental appliance to realign the dentition

The correct response is Option B. Cerebellopontine angle (CPA) tumor resection often requires intracranial facial nerve sacrifice. For best recovery of spontaneous ipsilateral facial nerve function, cross-facial nerve grafting (CFNG) should be performed from the contralateral facial nerve branches to the ipsilateral facial nerve branches. CFNG often results in delayed and weakened reinnervation of the ipsilateral facial musculature. For this reason, hypoglossal to facial nerve transfer and cross-facial nerve grafting (babysitter nerve transfers) are often employed to minimize muscle atrophy and enhance the strength of the ipsilateral facial musculature. Both hypoglossal nerve and nerve to masseter have been used. The other choices would not augment the ipsilateral facial musculature and would likely result in more pronounced facial asymmetry.

2017 A 34-year-old woman is scheduled to undergo resection of a left-sided cerebellopontine angle tumor with sacrifice of the facial nerve trunk. Staged cross-facial nerve grafting is planned. Which of the following treatment options is most likely to minimize the risk for permanent facial asymmetry at the time of tumor resection? A) Free gracilis muscle transfer to the left facial nerve stump B) Hypoglossal to facial nerve transfer and cross-facial nerve grafting C) Insertion of a 1.2-g gold weight in the left upper eyelid D) Percutaneous distal facial nerve branch stimulation E) Placement of an extended temporalis fascia static sling

The correct response is Option B. Hyperkinesis is generally considered to be the hyperactivity of the contralateral, unaffected side. Mirror biofeedback therapy has been shown to significantly improve facial symmetry when used in conjunction with botulinum toxin injections in the treatment of facial hyperkinesis. Beta-adrenergic blocker therapy is not indicated for facial hyperkinesis following facial nerve reconstruction. While selective facial myotomy has been used for improvement in synkinesis, selective contralateral facial neurotomy is not generally indicated for correction of hyperkinesis. Radiofrequency ablation and cryotherapy have only recently begun to be investigated as an option for improvement of synkinesis, but have not been generally accepted as treatments for hyperkinesis.

2017 A 35-year-old man returns for postoperative evaluation 12 months after undergoing facial nerve reconstruction with free gracilis transfer. Physical examination shows significant hyperkinesis of the contralateral side. In addition to injections of botulinum toxin type A, which of the following measures has been shown to improve facial symmetry? A) Cryotherapy of facial musculature B) Mirror biofeedback therapy C) Oral beta-adrenergic blocker therapy D) Radiofrequency ablation E) Selective contralateral facial neurotomy

The correct response is Option C. Use of the masseter nerve as the motor source for a gracilis free tissue transfer to restore smile is a single-stage procedure with many advantages over the more traditional use of cross-facial nerve grafting when the proximal stump of the facial nerve is not available for use. Its popularity has increased recently, particularly because it is a single-stage surgery and morbidity is minimal. Use of the masseter nerve, however, requires the patient to clench the jaw to smile, and is much less spontaneous than with cross-facial nerve grafting. Less excursion of the gracilis muscle graft is incorrect. The masseter nerve is an excellent motor nerve and allows for powerful contraction of the transferred muscle. Significant, permanent weakness in chewing function is incorrect because harvest of the masseter nerve is partial, and generally results in minimal donor site morbidity. Craniofacial osteotomy is incorrect because while the masseter nerve is in proximity to the zygomatic arch, an osteotomy is not frequently needed to reach the nerve. Decreased smile symmetry is incorrect because smile symmetry is comparable with both techniques.

2017 A 35-year-old woman is evaluated for long-standing facial nerve palsy. The proximal stump of the facial nerve is not available for use in reconstruction. In addition to gracilis muscle transfer, the surgeon is considering cross-facial nerve graft or using the masseter nerve. Which of the following factors regarding masseter nerve use is often cited as a disadvantage to cross-facial nerve grafting? A) Decreased excursion of the gracilis muscle B) Decreased smile symmetry C) Less spontaneity in smiling D) Requirement of a craniofacial osteotomy for harvest E) Significant, permanent weakness in chewing function

The correct response is Option A. When a facial nerve has been divided or resected, the best outcomes for regaining function are usually obtained from direct repair, or cable nerve grafting when too great a distance for direct repair separates the nerve ends. While autologous nerve grafts from "expendable" donor nerves, such as the great auricular nerve or sural nerve, have long been the gold standard, nerve repair using biologic or synthetic nerve conduits has also produced reasonable results, in some series equivalent to cable nerve grafts. Conduit nerve repair has the advantage of having no donor site morbidity. However, the length of the gap between the proximal and distal cut nerve ends is usually limited to less than 3 cm for the best chances of nerve recovery. When direct repair or cable nerve grafting is not possible—for example, when the nerve has been resected very proximally up to the intracranial portion of the nerve—cross-facial nerve grafting between redundant branches of the normal contralateral nerve and the distal facial nerve stumps of the paralyzed side can produce reasonable results with spontaneous symmetrical facial movement. Performing a nerve transfer from a donor nerve, such as the masseteric (V), spinal accessory (XI), or hypoglossal (XII) nerve can provide facial tone and symmetry at rest, and, in some cases, volitional movement with training. A temporary nerve transfer to these nerves is sometimes performed as a "babysitter" procedure while awaiting axonal growth through cross-facial nerve grafts. When nerve repair or nerve transfers from the contralateral face or donor nerves are not feasible, such as after motor endplate degeneration has occurred in the facial muscles, innervated free muscle flap transfers can restore facial movement to the lower face. Muscles commonly used for facial reanimation include the gracilis, pectoralis minor, serratus anterior, and latissimus dorsi, because of their thinness, good excursion, and low donor site morbidity. In addition to a microvascular anastomosis, an epineural nerve repair is performed either to a cross-facial nerve graft or to a donor cranial nerve such as the masseteric nerve.

2017 A 35-year-old woman undergoes surgical resection of a left parotid gland malignancy. The facial nerve was resected with the tumor, leaving a 5- to 7-cm gap between the proximal nerve stump and the distal nerve branches. Which of the following is the most appropriate treatment? A) Cable nerve grafting B) Cross-facial nerve grafting C) Hypoglossal nerve to facial nerve transfer D) Innervated gracilis muscle free flap reconstruction E) Nerve repair with a conduit

The correct response is Option D. This patient's congenital nevus involves 40% of his scalp and has areas of severe dysplasia. Although the lifetime risk of malignant transformation in giant nevi as a whole is approximately 4%, the presence of biopsy-proven severe dysplasia at this young age mandates a more aggressive approach to management than observation. Complete removal of the nevus with reconstruction of the scalp is recommended to prevent malignant conversion. Although serial excision is useful and often preferred for moderate sized lesions, this nevus is far too big to completely excise and close in that fashion. Moreover, the intervals between stages would require more time than tissue expansion, and with the degree of dysplasia present in this patient, this is risky. Similarly, it is dubious if a nevus this size could be successfully excised and closed in one stage using only rotational flaps. In addition, this would alter the direction of the hair. The use of a free flap is far too extreme in the absence of a frank malignancy (which this is not) and would leave this child with a large bald area. The best option is tissue expansion, followed by complete excision and closure, which was successfully employed in this case.

2017 A 4-year-old boy undergoes evaluation of a large congenital melanocytic nevus of the scalp affecting 40% of the surface area. Results of a recent biopsy in the center of the lesion showed diffuse areas of severe dysplasia. Which of the following is the most appropriate management? A) Serial excision, linear closure B) Single-stage excision, latissimus dorsi musculocutaneous free flap, skin grafting C) Single-stage excision, rotational flap closure D) Tissue expansion, excision, closure E) Observation

The correct response is Option D. Most parotid tumors are benign, but they can grow to a large size and produce significant symptoms of discomfort and distortion of anatomy. The differential diagnosis is aided by the use of imaging, CT, or MRI to confirm the location of the mass. Ultrasound-guided fine-needle aspiration is a useful next step in diagnosis of the majority (92%, Sharma, et al). MRI would be superfluous in the present case since imaging is sufficient. Benign neoplasms include pleomorphic adenoma, mucocele, branchial cleft cysts, and lymph nodes. Malignancies include adenocarcinoma.

2017 A 45-year-old man is evaluated because of a 5-cm mass at the angle of the mandible. A CT scan shows an intraparotid mass. Ultrasound-guided fine-needle aspiration shows benign findings. Which of the following is the most appropriate next step in management? A) Chemotherapy and superficial parotidectomy B) Follow-up evaluation in 3 months C) MRI D) Superficial parotidectomy only E) Total parotidectomy

The correct response is Option B. Hyperdontia is an anomaly of number of teeth and occurs most frequently in the maxilla (90%). It occurs during the initiation or proliferation stage. Morphodifferentiation issues lead to an anomaly of enamel/dentin/cementin composition. It is more common in males (2:1). It is 5 times more common in permanent dentition than in primary dentition. Finally, ectodermal dysplasia is associated with hypodontia.

2017 A 5-year-old child is brought to the office for evaluation of several supernumerary teeth. Which of the following is most characteristic regarding this anomaly? A) More common in the primary dentition than in the secondary dentition B) More frequent in the maxilla than in the mandible C) More prevalent in females than in males D) Occurs most often from disruption during the morphodifferentiation stage of tooth development E) Typically associated with ectodermal dysplasia

The correct response is Option E. This patient has cryptotia. The superior helical rim and scapha are not absent but lie buried under the supra-auricular skin. Although some authors have advocated expanding the superior helical framework, this is rarely indicated. In this patient with very mild auricular shortening, supplementing or reconstructing the cartilage framework with rib cartilage, polyethylene, or silicone is unnecessary. Instead, the ear framework simply released its posterior aspect and the resultant defect lined with full-thickness skin graft or any number of skin flaps raised from the postauricular region. The use of acellular dermal matrix is not an accepted method of correcting cryptotia.

2017 A 5-year-old girl is brought to the physician because her parents are interested in correction of the unilateral ear anomaly shown in the photographs. Which of the following materials is most likely to be used in the procedure to correct this anomaly? A) Acellular dermal matrix B) Autogenous rib cartilage C) Porous polyethylene D) Silicone E) Skin

The correct response is Option C. Vertical maxillary excess (VME), or long face syndrome, occurs when there is excessive (imbalanced) anterior facial height in the lower half of the face. The midface is relatively protrusive. Excessive eruption of the posterior dentition in the maxilla can cause clockwise rotation of the mandible. There is lip incompetence, excessive gingival show, and an effort to close the lips can result in mentalis strain. It is associated with an anterior open bite.

2017 Which of the following findings is most common in patients with vertical maxillary excess? A) Counterclockwise rotation of the mandible B) Excessive height in the upper half of the face C) Mentalis strain D) Posterior open bite E) Retrusive midface

The correct response is Option A. The osteocutaneous radial forearm free flap (OCRFFF) is based on the radial artery, one of the major sources of blood to the hand. An abnormal Allen test is a sign of insufficient ulnar artery blood flow and would be a major contraindication to utilizing this flap. While the quantity of bone available for transfer from the radius is thought of as a limitation of this flap, many authors have reported safely harvesting up to 10 cm or more of bone length. The thickness of the bone is also a limitation and it is recommended that no more than one-third to one-half of the bone thickness be harvested to avoid an iatrogenic radial fracture, even when the remaining bone is prophylactically plated and/or bone grafted. Because of this, osseointegrated implants for dental restoration can rarely be performed. The bone component of the OCRFFF is well vascularized and associated with high rates of union and can tolerate osteotomies needed for anterior mandible restoration as well as be used for closing irradiated wounds. An advantage of this flap, in addition to providing a thin, pliable skin paddle, is that it has a long pedicle length that can often reach the inspilateral transverse cervical blood vessels or contralateral neck blood vessels without the need for interposition vein grafting.

2017 A 50-year-old man is scheduled to undergo mandibulectomy with floor of mouth resection for cancer. An osteocutaneous radial forearm free flap is being considered. Which of the following is the strongest contraindication to performing this flap? A) Abnormal Allen test B) Anterior mandibular tumor location C) Defect length of 8 cm D) History of prior radiation E) Lack of availability of ipsilateral neck muscles as recipients

The correct response is Option B. When dental implants are placed in thicker cortical bone, better initial stability is achieved at the bone implant interface, reducing micromotion during the process of osseointegration. Cortical bone thickness has also been shown to correlate with the amount of torque required to remove a dental implant. No significant correlation has been shown between removal torque and total bone thickness. The presence of osteogenic cells in cancellous bone is responsible for a biologic response, although it is not as important in improving initial stability as thick cortical bone. Periosteal thickness is not relevant to dental restoration.

2017 A 55-year-old man undergoes resection of locally destructive floor-of-mouth cancer including a tooth-bearing segment of the mandible. Immediate reconstruction is performed using a fibula flap. Dental restoration with implants is planned. Which of the following properties of the fibula flap is most likely to ensure the greatest initial stability of the dental implants? A) Cancellous bone thickness B) Cortical bone thickness C) Periosteal thickness D) Total bone thickness E) Total bone width

The correct response is Option B. There are many methods to reconstruct this nasal tip defect. Denuded cartilage needs a flap for coverage. As this patient's cartilages are intact, they do not need to be replaced. Smaller defects can be covered with a locally available flap. In this case neither a forehead nor a nasolabial flap is necessary, and each would result in more severe donor site morbidity. Bilobed flaps are ideal for distal nasal reconstruction, while the glabella flap is ideal for proximal reconstruction. A dorsal nasal flap, if large enough, may also be an option for reconstruction of the nasal tip.

2017 A 55-year-old woman comes to the office with a 10 x 6-mm full-thickness defect after undergoing Mohs micrographic surgery to remove a basal cell carcinoma on the nasal tip not involving the alar margin. The denuded lower lateral cartilages with no perichondrium are exposed. Which of the following is the most appropriate reconstruction option? A) Auricular composite graft B) Bilobed flap C) Forehead flap D) Glabella flap E) Nasolabial flap

The correct response is Option C. Hyperlacrimation, or Bogorad syndrome, is a known complication after Bell palsy or other injury and insults to the facial nerve. Similar to Frey syndrome, the predominant theory for this form of gustatory hyperlacrimation is due to aberrant facial nerve regeneration. Epiphora in general can also occur due to poor "pumping mechanisms" in the eyelids as well as prolonged ectropion and conjunctival show after facial nerve injury. However, hyperlacrimation during gustatory activity is a specific and definable pathology. Treatment for this syndrome includes subtotal lacrimal gland resection, botulinum toxin type A, and various forms of enlarging the lacrimal tract. Synkinesis is a common event after facial nerve regeneration, when the nerve improperly fires and there is lack of typical mimetic muscle coordination. The anterior portion of the tongue taste buds are innervated by facial nerve fibers from the chorda tympani to the lingual nerve, but the base of the tongue is innervated by cranial nerves IX and X. Anosmia is loss of smell that occurs through cranial nerve I injury or obstruction and can lead to taste disturbances. Migraine headaches can be associated with a variety of syndromes and need to be differentiated from other forms of headaches. Ramsay Hunt syndrome can lead to facial nerve dysfunction and facial pain, but this pain is not associated with facial nerve regeneration or migraine headaches.

2017 A 57-year-old man undergoes superficial parotidectomy. Facial nerve neuropraxia results in gustatory sweating and which of the following additional symptoms? A) Anosmia B) Base of tongue dysgeusia C) Hyperlacrimation D) Migraine headache E) Synkinesis

The correct response is Option B. The deep circumflex iliac artery arises from the external iliac artery and is the blood supply to the iliac crest osteocutaneous flap. This flap can be harvested either as a bone-only or an osteocutaneous free flap. It is often used in hemimandibular reconstruction because the natural curvature of the iliac crest closely resembles the shape of the hemimandible. A portion of the internal oblique muscle, based on the ascending branch of the deep circumflex iliac artery, can also be included with this flap. The descending branch of the geniculate artery is the blood supply to the medial femoral condyle flap. The ascending branch of the lateral circumflex femoral artery is the blood supply to the tensor fascia lata flap. The peroneal artery is the blood supply to the fibula flap. The deep inferior epigastric artery is the blood supply to the rectus abdominis myocutaneous flap.

2017 A 58-year-old man undergoes a left hemimandibulectomy. Reconstruction with an osteocutaneous free flap harvested from the ipsilateral pelvis is planned. The vascular pedicle supplying this flap is based on which of the following arteries? A) Ascending branch of the lateral circumflex femoral artery B) Deep circumflex iliac artery C) Deep inferior epigastric artery D) Descending branch of the geniculate artery E) Peroneal artery

The correct response is Option B. Most patients who undergo hemiglossectomy can expect reasonable speech and swallowing function when reconstructed with a thin, pliable free flap, such as the radial forearm fasciocutaneous free flap, that facilitates unrestricted residual tongue movement. Although his swallowing may likely improve as he recovers from surgery and tissue edema resolves, this patient will need a feeding tube to maintain his nutrition at this time. Additionally, it can be difficult for many patients who have undergone substantial tongue resections to meet their caloric needs even if they pass their initial swallowing study during radiation therapy and short-term feeding tube placement may be indicated. Tracheoesophageal puncture with placement of a one-way valve speech prosthesis is used to restore speech function in patients who have received a laryngectomy and does not apply to this patient. Revision of the free flap is not indicated in the early postoperative period as it is unlikely to significantly improve swallowing and may delay adjuvant treatment. A laryngectomy for aspiration would only be indicated as a last resort in a patient with chronic, long-term aspiration of oral secretions resulting in recurrent pneumonia, most commonly following more extensive tongue resections, such as a total glossectomy, including removal of the tongue base. Postoperative radiation therapy should be administered within 4 to 6 weeks of surgery for maximal effectiveness and, therefore, delaying for 10 more weeks may adversely affect this patient's survival.

2017 A 59-year-old man with tongue cancer undergoes a hemiglossectomy, neck dissection, and reconstruction with a radial forearm fasciocutaneous free flap. On postoperative day 10, he fails a swallowing study for all food consistencies. Postoperative radiation therapy is scheduled to begin in 2 weeks. What is the appropriate next step in management? A) Laryngectomy for aspiration B) Percutaneous endoscopic gastrostomy tube placement C) Revision of the free flap D) Tracheoesophageal puncture E) Observation with delay of radiation therapy for 10 weeks

The correct response is Option B. GLUT-1 is a specific marker for infantile hemangioma and is often used by pathologists to confirm the diagnosis. VEGF and TGF-B are incorrect, because while these markers may often be present in hemangiomas, they are not as specific as GLUT-1 in confirming the diagnosis of hemangioma. Staining for FGFR-3 is not routinely used in confirming the diagnosis of hemangioma. FGFR-3 mutations are implicated in a number of syndromes, including Muenke syndrome, epidermal nevus, and achondroplasia. Staining for TNF-1 is also not routinely used in confirming the diagnosis of hemangioma.

2017 A 6-month-old infant is brought to the clinic for evaluation of an expanding vascular cutaneous lesion that partially obstructs the visual axis. The presence of which of the following histologic markers is most likely to confirm the diagnosis of infantile hemangioma in this patient? A) Fibroblast growth factor receptor 3 (FGFR-3) B) Glucose transporter 1 (GLUT-1) C) Transforming growth factor beta (TGF-B) D) Tumor necrosis factor 1 (TNF-1) E) Vascular endothelial growth factor (VEGF)

The correct response is Option D. During embryologic development, the thyroid gland descends from the foramen caecum to the midline of the neck. The tract in descent is typically absorbed but sometimes remains. This can be secondarily infected from infections of the head and neck. Rarely, the thyroid gland does not fully descend into its position in the midline neck. Diagnosis of this mass over other types of midline mass include elevation of the mass on tongue protrusion. This is because of the remaining attachment to the base of the tongue. Lymph nodes can present in the midline, but they are infrequent at the level of the hyoid and typically do not drain percutaneously. A type II branchial cleft cyst presents laterally, not in the midline. Infected sebaceous cysts can drain in any hair-bearing area, but do not move with protrusion of the tongue.

2017 A 6-year-old boy is brought to the office because of a draining sinus in the midline of the neck. His mother reports that the drainage developed after he had an upper respiratory infection a few weeks ago. Physical examination shows a palpable mass in the mid third of the neck that moves upward when the patient protrudes his tongue. Which of the following is the most likely diagnosis? A) Infected sebaceous cyst B) Infected thyroid gland C) Lymph node D) Thyroglossal duct cyst E) Type II branchial cleft cyst

The correct response is Option A. Head and neck melanomas in general were thought to be distinct from other anatomic sites. Clearly there can be reconstructive and functional issues that are unique, such as in an ear melanoma. In general the first Multicenter Selective Lymphadenectomy Trial (MSLT-1) concluded that for intermediate-thickness melanoma (1-4 mm), the status of the sentinel node was the most powerful predictor of outcome. In an interim report in 2006, well before the final report in 2014, the specifics on what percentage of patients had melanoma of the head and neck in the MSLT-1 trial were not described, but what was reported was a lower rate of identification of a sentinel node in the neck versus lesions that mapped to the groin or axilla. This led many to conclude that the utility of the sentinel node biopsy in the head and neck was in question. More recent data from multiple high-volume institutions have concluded that identification of the sentinel node for head and neck melanoma is as accurate as other sites in the body, including similar false-negative rates and impact on prognostication. Mitotic rate does not affect staging above 1-mm thick lesions, and although ulceration can, it does not influence the rationale to perform a sentinel node biopsy in intermediate thickness tumors. In thinner melanomas, a positive deep margin may be an indication for a sentinel node biopsy, but not for a tumor greater than 1-mm thick. The fact that lymphoscintigraphy may map the sentinel node to the parotid region means that the surgeon should be prepared to do a parotidectomy with facial nerve preservation, although recently less-invasive techniques have been described.

2017 A 62-year-old man has a lesion of the left pinna. Examination of a specimen obtained on biopsy shows a 2.01-mm-thick melanoma with no ulceration, 11 mitoses, and a positive deep margin. The surgeon's decision to perform sentinel node biopsy is most heavily influenced by which of the following factors? A) Breslow thickness B) Head and neck location C) Lack of ulceration D) Number of mitoses E) Positive deep margin

The correct response is Option E. The primary goal of cleft palate repair is normal speech. Velopharyngeal competence, the ability to completely close the velopharyngeal sphincter, is required for the normal production of all but the nasal consonants (in English: /m/, /n/, and /ng/). Velopharyngeal insufficiency is defined as the inability to completely close the velopharyngeal sphincter. The primary effects of velopharyngeal insufficiency are nasal air escape and hypernasality. Video fluoroscopy and nasal endoscopy can detect the sagittal deficiency closure pattern occurring in patients with velopharyngeal insufficiency after cleft palate surgery. Speech articulation errors (i.e., distortions, substitutions, and omissions) are secondary effects of velopharyngeal insufficiency. The result is decreased intelligibility of speech. The velopharyngeal port is bordered anteriorly by the velum, bilaterally by the lateral pharyngeal walls, and posteriorly by the posterior pharyngeal wall. Velopharyngeal insufficiency can be diagnosed by both subjective and objective means. The speech evaluation by a trained pathologist with perceptual evaluation of speech by an experienced speech language pathologist is the standard. Multiview videofluoroscopy and nasendoscopy both provide visual information (i.e., closure pattern and closure rating) that is valuable for surgical planning. However, the need for radiation has caused most cleft centers to migrate to direct nasal endoscopy. The MRI is emerging technology but would not be the first line choice in the diagnostic workup. Rhinometry is an objective measurement of nasal air emission during speech and is not routinely used for surgical decision-making. Overall, the exam under anesthesia would likely not be required but instead a complete exam and nasal endoscopy would likely be able to be performed comfortably in the office. Experience with nasal endoscopy has grown in most comprehensive cleft centers and has become an invaluable tool for surgical planning.

2017 A 7-year-old girl with a history of cleft palate repair is brought to the office for evaluation of velopharyngeal insufficiency. After speech evaluation, which of the following is the most appropriate initial tool for diagnosis and management of this patient's condition? A) Cine MRI B) CT scan C) Physical examination under anesthesia D) Rhinometry E) Video nasal endoscopy

The correct response is Option E. The submandibular approach to the mandibular body/ramus is an important surgical approach to address a number of facial fractures. An understanding of the anatomy of the submandibular region is crucial to performing this approach with minimal complications. The marginal mandibular nerve leaves branches off the facial nerve trunk during its intraparotid course. One to three branches usually exit the inferior border of the parotid gland before changing course to travel along the mandibular border toward the symphysis. In almost all cases, the marginal mandibular nerve travels superficial to or within the investing layer of the deep cervical fascia as it courses anteriorly. In several cadaveric studies, the nerve consistently coursed within 1 cm below the mandibular border posterior to the facial vessels, and above the mandibular border anterior to the facial vessels. The submandibular approach to facial fractures involves incising the skin 2 cm below the mandibular border, dividing the platysma and dissecting deep to the investing fascia, just above the submandibular gland. Once in this plane, the course of dissection is carried superiorly toward the mandibular border.

2017 A 63-year-old woman is admitted to the hospital with a fracture to the left mandibular angle that she sustained in a motor vehicle collision. Open reduction and internal fixation is planned via a submandibular (Risdon) incision. The region between which of the following planes of dissection is most appropriate for approaching the fracture and avoiding injury to the marginal mandibular branch of the facial nerve? A) Carotid sheath and the posterior belly of the digastric muscle B) Platysma and the superficial (investing) layer of the deep cervical fascia C) Skin and the platysma D) Submandibular gland and the carotid sheath E) Superficial (investing) layer of the deep cervical fascia and the submandibular gland

The correct response is Option A. Bisphosphonate-related osteonecrosis of the jaw is a condition found in patients who have received intravenous and oral forms of bisphosphonate therapy for various bone-related conditions such as osteoporosis. The patient may develop exposed, nonvital bone involving the maxillofacial structures. Osteonecrosis may occur following minor trauma with decreased capacity for bone healing due to the effects of bisphosphonate therapy. Treatment may involve antimicrobial rinses, systemic antibiotics, systemic or topical antifungals, and discontinuation of bisphosphonate therapy. Raloxifene is in a class of drugs called estrogen agonists/antagonists that have been developed to provide the beneficial effects of estrogens without their potential disadvantages. It is neither an estrogen nor a hormone. Raloxifene used to be called a selective estrogen receptor modulator (SERM). Calcitonin is a synthetic hormone for the treatment of osteoporosis. The naturally occurring hormone is involved in calcium regulation and bone metabolism. Teriparatide, a type of parathyroid hormone, is approved for the treatment of osteoporosis in postmenopausal women and in men who are at high risk for fracture. Estrogen therapy with or without progesterone is approved for the prevention of osteoporosis in postmenopausal women. Estrogen reduces bone loss, increases bone density in both the spine and hip, and reduces the risk for hip, spine, and other fractures in postmenopausal women.

2017 A 64-year-old post-menopausal woman is referred for evaluation and treatment of mandibular osteonecrosis. Discontinuation of which of the following medications should be considered? A) Alendronate B) Calcitonin C) Estrogen D) Raloxifene E) Teriparatide

The correct response is Option E. Salivary gland tumors are relatively rare and make up about 3 to 4% of all head and neck neoplasms. The majority of salivary gland tumors (approximately 80%) originate in the parotid gland. Approximately 80% of parotid gland tumors are benign. Facial paralysis may be associated with malignant tumors and are a sign of neural invasion. Warthin tumor (papillary cystadenoma lymphomatosum) is the second most common tumor of the parotid gland and is benign. Warthin tumors predominantly occur in men of 50 to 70 years of age, most frequently smokers, and are the most common bilateral salivary gland tumors. The histologic appearance of this tumor is very characteristic and is characterized by papillary cysts and mucoid fluid as well as nodules of lymphoid tissue. Pleomorphic adenoma, also known as benign mixed tumor, is the most common tumor of the parotid gland. This tumor is histologically characterized by epithelial and connective tissue elements, with stellate and spindle cells interspersed within a myxoid background. Adenoid cystic carcinoma is the second-most common malignancy of the salivary glands after mucoepidermoid carcinoma and exhibits a propensity for perineural invasion. There are three histologic subtypes: cribriform, tubular, and solid. The cribriform pattern has a classic "swiss cheese" appearance with cells arranged in nests separated by round or oval spaces. The tubular pattern has a glandular architecture, while the solid (or basaloid) pattern has sheets of cells with little or no luminal spaces. Hemangiomas are the most common salivary gland tumors found in children and usually involve the parotid gland. Histologically, the tumors are composed of capillaries lined by proliferative endothelial cells. Squamous cell carcinoma is a malignant tumor that rarely involves the parotid gland, in comparison to the skin and aerodigestive tract. It is histologically identical to squamous cell cancers arising from other sites with epithelial cells that form sheets or compact masses, which invade adjacent connective tissue. Round nodules of keratinized squamous cells known as "keratinous pearls" are the hallmark of well-differentiated squamous cell carcinoma.

2017 A 65-year-old man comes to the office because of slow-growing, painless masses within each of the parotid glands. He has smoked 10 cigarettes daily for the past 25 years. Superficial parotidectomies are performed, and pathologic examination shows papillary cysts and mucoid fluid as well as nodules of lymphoid tissue in both tumor specimens. Which of the following is the most likely diagnosis? A) Adenoid cystic carcinoma B) Hemangioma C) Pleomorphic adenoma D) Squamous cell carcinoma E) Warthin tumor

The correct response is Option A. The patient in this question has had multiple palate surgeries and severe midface hypoplasia. One of the sequelae of severe midface hypoplasia is obstructive sleep apnea, which is confirmed by the patient's abnormal polysomnogram. The best treatment for this patient is a trial of CPAP. Although midface advancement surgery (either conventionally or with distraction) is often used to address obstructive sleep apnea, this patient is a poor candidate for the surgical options provided. The patient is in mixed dentition, and a Le Fort I level surgery would risk permanent injury to his unerupted adult teeth. Although a Le Fort III osteotomy would avoid injury to tooth roots, it would also advance his infraorbital rims, which are not affected in patients with cleft lips. A tracheostomy will bypass his midface level obstruction; however, it is associated with significant cost, burden of care, and a 1% annual mortality risk and should be avoided if less invasive options exist.

2017 A 7-year-old boy with a history of bilateral cleft lip and palate has undergone multiple procedures including lip and nose repair, palate repair, and closure of an oronasal fistula. His parents note that during the past 6 months he has had nighttime snoring, frequent pauses in his breathing, and daytime somnolence. Physical examination shows mixed dentition with severe midface hypoplasia and Angle class III malocclusion with 12 mm of negative overjet. A polysomnogram demonstrates an obstructive apnea-hypopnea index (AHI) of 12.5 per hour. The patient is otherwise healthy. Which of the following is the best treatment option for this patient? A) Continuous positive airway pressure (CPAP) B) Le Fort I advancement and bilateral sagittal split setback C) Le Fort I osteotomy and application of bilateral internal maxillary distractors D) Le Fort III osteotomy and application of external halo distractor E) Tracheostomy

The correct response is Option D. Direct closure is being used more commonly in large Mohs facial defects, especially in the elderly, as it minimizes the disruption of skin tension lines. The contraindication in this case to closure alone is the nodal involvement and the extension into the parotid capsule. Given the parotid capsule extension, adequate margins at a minimum will require a superficial parotidectomy. Radiation alone plus flap closure would have a higher recurrence rate because residual disease is left in the capsule. The nodal enlargement requires further evaluation and given the options the best combination is superficial parotidectomy, neck dissection, and a local rotation flap.

2017 A 70-year-old man is referred for evaluation directly after undergoing excision of recurrent squamous cell carcinoma and Mohs micrographic surgery. A photograph is shown. Pathology shows some extension onto the parotid capsule. Clinical examination shows two enlarged postauricular lymph nodes. Which of the following is the most appropriate management of this patient? A) Cervical facial flap plus radiation B) Direct closure alone C) Superficial parotidectomy and direct closure D) Superficial parotidectomy, neck dissection, and coverage with a cheek rotation flap E) Superficial parotidectomy, neck dissection, and direct closure

The correct response is Option C. Merkel cell carcinoma, an aggressive neuroendocrine tumor, is most likely. It presents in older, immunocompromised women in sun-exposed areas. About 80% of Merkel cell carcinomas are secondary to polyomavirus infection. Treatment of the primary tumor should be wide local excision or Mohs micrographic surgery. For wide local excision of tumors smaller than 2 cm, the recommended surgical margin should be 1 cm. As there is a high rate of occult nodal metastasis, and nodal status is associated with mortality rates, biopsy of the sentinel node is recommended for all cases regardless of primary tumor size. Merkel cell carcinoma is a radiosensitive tumor, and postoperative adjuvant radiation therapy has been shown to decrease local recurrence. Chemotherapy is only currently indicated for palliation and distant metastasis.

2017 A 75-year-old woman is evaluated because of a new skin lesion on the right upper eyelid. Examination of the specimen obtained on biopsy shows a 1-cm Merkel cell carcinoma. In addition to regional node sampling, which of the following is the most appropriate excision and adjuvant management in this patient? A) 1-cm margins and chemotherapy B) 2-cm margins and chemotherapy C) 1-cm margins and postoperative radiation therapy D) 2-cm margins and postoperative radiation therapy E) 5-mm margins and postoperative radiation therapy

The correct response is Option B. A 9-year-old girl with a history of basal cell carcinoma as well as kyphoscoliosis and a cystic mass in the mandible should raise concerns for Gorlin syndrome (also known as basal cell nevoid syndrome). Gorlin syndrome is an autosomal dominant disorder characterized by multiple basal cell carcinomas, keratocystic odontogenic tumors mostly of the mandible, as well as skeletal abnormalities including kyphoscoliosis and bifid ribs. The histologic features of keratocystic odontogenic tumors include keratinized epithelium without characteristic epidermal architecture, such as rete ridges. The other answer options describe different head and neck tumors not associated with Gorlin syndrome. Palisading basaloid cells with large nuclei are characteristic of ameloblastoma; multinucleated giant cells are found in giant-cell granulomas; polymorphonuclear cell infiltrate would be seen with a foreign body reaction; and dysmorphic squamous epithelial cells with high mitoses are suggestive of squamous cell carcinoma of the head and neck. None of these answer choices are supported by the clinical description.

2017 A 9-year-old girl is brought to the office because of a 3-month history of a slowly enlarging right mandible angle mass. Medical history includes multiple basal cell carcinomas. A panoramic x-ray study (Panorex) is shown. A biopsy of the mass is performed. Which of the following histologic characteristics is most likely to be identified from the biopsy specimen? A) Dysmorphic squamous epithelial cells with multiple mitoses B) Keratinized epithelium without evidence of rete ridges C) Multinucleated giant cells D) Palisading basaloid cells with large nuclei E) Polymorphonuclear cell infiltrate

The correct response is Option D. Tessier No. 3 orofacial clefts are the most common type. When they affect the alveolus, they typically traverse between the lateral incisor and the canine and extend into the floor of the nose and through the nasolacrimal system and orbital floor, involving the medial canthal region. The other options are all incorrect, because they are not typically the route seen in the Tessier No. 3 orofacial cleft.

2017 A newborn male with a Tessier No. 3 orofacial cleft is evaluated in the NICU. Which of the following locations is the most common pathway of this cleft through the alveolar ridge? A) Between the canine and premolar B) Between the central and lateral incisors C) Between the central incisors D) Between the lateral incisor and canine E) Between the premolar and first molar

The correct response is Option D. Naso-orbital-ethmoid (NOE) fractures can be challenging fractures, and either through direct instrumentation with transcanthal wiring or from the fractures themselves, the lacrimal drainage system can be affected. Postoperative epiphora can be very common and is present in at least 50% of patients who have undergone open reduction and internal fixation (ORIF) of an NOE fracture. After 3 to 6 months approximately half of this epiphora resolves, with the other half of patients (25%) requiring consideration for other investigations to evaluate lacrimal drainage.

2017 A patient underwent open reduction and internal fixation of naso-orbital-ethmoid fractures 12 months ago, and epiphora was noted on follow-up examination. After 6 months of observation and persistent epiphora, which of the following is the most appropriate next step in management of the patient's nasolacrimal system? A) Conjunctivorhinostomy tube placement B) Continued observation C) Dacryocystorhinostomy D) Jones tests E) Lacrimal system flushing

The correct response is Option A. This radiograph is most consistent with a dentigerous cyst. Dentigerous cysts are the second most common and develop in the dental follicle of an unerupted tooth. On radiograph there is usually a lucency attached at an acute angle to the tooth. The mandibular and maxillary third molars are the most commonly affected. Odontogenic cysts are epithelial lined cysts that are defined by location and histologic characteristics. Periapical cysts are the most common and usually form from necrotic pulp after a tooth infection. They usually present as a radiologic lucency at the apex of the tooth. A gingival cyst is a superficial cyst in the gingiva. A primordial cyst develops instead of a tooth. This is a rare cyst. A residual cyst may result from a retained periapical cyst after teeth have been removed.

2017 An 18-year-old man comes to the office for evaluation because of swelling of his chin. A panoramic x-ray study (Panorex) is shown. Which of the following types of cyst is the most likely diagnosis? A) Dentigerous B) Gingival C) Periapical D) Primordial E) Residual

The correct response is Option C. Bioresorbable fixation is now widely employed in craniomaxillofacial surgery. While this technology has theoretical advantages in the treatment of craniosynostosis (e.g., obviates concerns of intracranial implant migration), the benefits of orthognathic surgery and the management of facial trauma are dubious. For mandibular fractures, several studies have found no statistical differences in overall or specific complication rates, including the need for plate removal, postoperative infection, malreduction/malocclusion, postoperative pain, or loss of fixation. Rigorous comparisons are lacking, but a recent comprehensive review of the reported studies demonstrated a trend toward increased complications using resorbable fixation to treat facial fractures. The only consistent difference is the cost of the implants, which is considerably higher for the resorbable systems.

2017 An 18-year-old man is evaluated for a mandibular fracture sustained in a motor vehicle collision. CT scan shows displaced left parasymphyseal and right mandibular body fractures. Resorbable plate fixation is planned. Compared with titanium implants, the use of resorbable fixation to treat this fracture is associated with which of the following? A) Decreased postoperative pain B) Decreased risk for malunion C) Increased overall cost D) Increased risk for infection E) Lower hardware profile

The correct response is Option A. The patient shown has a large soft tissue defect with exposed bone and hardware. There is friable granulation tissue, and the patient presents several weeks after injury. She has a complete facial palsy and her eye is closed at rest (good eye protection) without taping. The ultimate goals are to address both her wound and her facial palsy, with an aesthetic facial reconstruction. In this setting, a simple nerve transection that would benefit from a simple neurorrhaphy is unlikely. The tissues are very friable, and tissue planes are not easily identified due to inflammation, extensive damage, and subacute time period. Dissection and exploration would be difficult and could lead to further damage. Soft tissue coverage is a priority at this point in the patient's reconstruction. She has a large defect with exposed bone and hardware. Free tissue transfer could provide stable coverage. A skin graft will not provide durable coverage over hardware and exposed bone. Delayed facial reanimation after stable soft tissue coverage will allow for improved healing. Delaying addressing the facial nerve palsy a few weeks to months can still have good outcomes and there will be an improved healing environment.

2017 An otherwise healthy 20-year-old woman is evaluated 5 weeks after sustaining facial shear injury in a motor vehicle collision. She has a facial nerve palsy on the right and exposed mastoid, zygoma, and zygomatic arch. A photograph is shown. She has had previous debridement and titanium mesh cranioplasty for the traumatic cranial defect of the temporal bone. Multifocal extratemporal facial nerve injuries are suspected. Which of the following is the most appropriate management for the wound coverage and facial nerve palsy? A) Free tissue transfer for wound coverage with delayed facial reanimation B) Immediate cross-face nerve grafting with cervicofacial flap for soft tissue coverage C) Immediate exploration and primary repair of the facial nerve followed by skin grafting for coverage D) Split-thickness skin grafting for coverage with delayed facial reanimation E) Temporalis muscle sling with skin grafting for wound coverage

The correct response is Option D. Most isolated orbital fractures involve the orbital floor made up of the maxillary bone. The maxillary bone is quite thin behind the infraorbital rim and is perforated by the infraorbital nerve passing in a canal below it. Most pure blow-out fractures involve the orbital floor with the maxillary bone making the majority of the orbital floor. A retrospective study by Hwang et al. evaluated 391 patients with orbital bone fractures from a variety of accidents that were treated at the department of Plastic and Reconstructive Surgery, Inha University Hospital, Incheon, South Korea, between February 1996 and April 2008. The medical records of these patients were reviewed and analyzed to determine the clinical characteristics and treatment of the orbital bone fractures. The following results were obtained. The mean age of the patients was 31.1 years, and the age range was 4 to 78 years. The most common age group was the third decade of life (32.5%). There was a significant male predominance in all age groups, with a ratio of 4.43:1. The most common etiology was violent (assault) or nonviolent traumatic injury (57.5%) followed by traffic accidents (15.6%) and sports injuries (10.7%). The most common isolated orbital bone fracture site was the orbital floor (26.9%). The largest group of complex fractures included the inferior region of the orbital floor and zygomaticomaxilla (18.9%). Open reduction was performed in 63.2% of the cases, and the most common fracture reconstruction material was MEDPOR (56.4%) followed by a resorbable sheet (41.1%). The postoperative complication rate was 17.9%, and there were no statistically significant differences among the reconstruction materials with regard to complications. During follow-up, diplopia, hypoesthesia, and enophthalmos occurred as complications; however, there was no significant difference between porous polyethylene sheet (MEDPOR) and resorbable sheet groups. Long-term epidemiologic data regarding the natural history of orbital bone fractures are important for the evaluation of existing preventative measures and for the development of new methods of injury prevention and treatment.

2017 In adults, which of the following bones is most commonly fractured in isolated orbital floor fractures? A) Ethmoid B) Frontal C) Lacrimal D) Maxillary E) Zygomatic

The correct response is Option A. Proboscis lateralis, which is illustrated in the photograph, is thought to be a failure of fusion between the lateral and maxillary nasal processes. A cleft lip is the result of a failure of fusion of the maxillary prominence with the medial nasal prominence. A midline cleft or Tessier Zero cleft is a result of the failure of fusion of the medial nasal prominences. Choanal atresia is a result of a failure of the oronasal membrane to rupture. Finally, a mandibular cleft or Tessier # 30 cleft is a result of the failure of fusion of the mandibular prominences.

2017 The congenital anomaly shown in the photograph is thought to be caused by which of the following? A) Failure of fusion of the lateral and maxillary nasal processes B) Failure of fusion of the maxillary prominence with the medial nasal prominence C) Failure of fusion of the medial nasal prominence and the lateral nasal prominence D) Failure of fusion of the medial nasal prominences E) Failure of the oronasal membrane to rupture

The correct response is Option B. The pharyngeal, or branchial, arches are developmental structures derived from all three germ layers and also contain neural crest cells. These arches give rise to bony, cartilaginous, vascular, muscular, and neural structures of the head and neck. The second pharyngeal arch gives rise to the muscles of facial expression, posterior belly of the digastric muscle, and the stapedius muscle. These muscles are innervated by the facial nerve (VII), which also arises from this arch. The first pharyngeal arch gives rise to the muscles of mastication, anterior belly of the digastric, mylohyoid, tensor tympani, and tensor veli palatini muscles as well as the trigeminal nerve (V). The third pharyngeal arch gives rise to the stylopharyngeus muscle as well as the glossopharyngeal nerve (IX). The fourth pharyngeal arch gives rise to the cricothyroid muscle and all intrinsic muscles of the soft palate except the tensor veli palatini, as well as the superior laryngeal nerve (X). The sixth pharyngeal arch gives rise to the intrinsic muscles of the larynx except the cricothyroid muscle as well as the recurrent laryngeal nerve (X). The fifth pharyngeal arch does not give rise to structures in humans. Arch/Nerve Skeletal Ligaments Muscles Pouch First (V) 1.Malleus 2 Incus 1. Anterior ligament of malleus 2. Sphenomandibular ligament 1. Muscles of mastication 2. Tensor palatini 3. Tensor tympani 4. Mylohyoid 5. Ant. Belly digastric 1. Ext. auditory canal 2. Middle ear space Second (VII) 1. Stapes 2. Styloid 3. Hyoid (lesser horn, upper body Stylohyoid ligament 1. Facial expression 2. Stapedius 3. Stylohyoid 4. Post belly digastric Tonsillar fossae Third (IX) Hyoid greater horn, lower body None Stylopharyngeus 1. Inf parathyroid 2. Thymus Fourth (X) Larynx None 1. Laryngeal mm 2. Pharyngeal mm 3. Soft palate 1. Sup parathyroid 2. Thyroid c-cells Sixth (XI) None None 1. Sternocleidomastoid 2. Trapezius None

2017 The muscles of facial expression, the posterior belly of the digastric muscle, and the stapedius muscle are derived from which of the following pharyngeal arches? A) First B) Second C) Third D) Fourth E) Sixth

The correct response is Option E. Maxillary transverse deficiency (MTD) in the skeletally mature patient is best addressed with surgically assisted rapid palatal expansion (SARPE). In the young patient (before suture closure), orthopedic and orthodontic forces can be more easily used to correct the MTD. Reverse-pull headgear does not aid in expansion in the skeletally mature patient. One-piece Le Fort and bilateral sagittal split osteotomy (BSSO) procedures address anterior-posterior discrepancies rather than transverse deficiencies.

2017 Which of the following is the best method to treat maxillary transverse deficiency in a skeletally mature patient? A) Mandibular setback (bilateral sagittal split osteotomy) B) Maxillary advancement (Le Fort I advancement) C) Orthopedic and orthodontic expansion D) Reverse-pull headgear E) Surgically assisted rapid palatal expansion

The correct response is Option A. Cerebrospinal fluid leak is the most common complication of a temporal bone injury. The majority of these will resolve spontaneously within a week. If they persist longer, then there is higher risk for meningitis, but this is not common. Facial nerve injury is the second most common injury and prognosis is dependent on the severity and delay of onset. Incomplete nerve loss or delayed onset is associated with a better prognosis for recovery. Hearing loss is the third most common complication seen with this fracture. Temporomandibular joint ankylosis is an unlikely sequela of this type of injury.

2017 Which of the following is the most common complication of a fracture to the temporal bone? A) Cerebrospinal fluid leak B) Facial nerve injury C) Hearing loss D) Meningitis E) Temporomandibular joint ankylosis

The correct response is Option C. Two phase 3 randomized, multi-center, double-blind, placebo-controlled studies of identical design were conducted to evaluate botulinum toxin type A prior to FDA approval. The injection volume was 0.1 mL/injection site, for a dose/injection site in the active treatment groups of 4 units. Subjects were injected intramuscularly in five sites—1 in the procerus muscle and 2 in each corrugator supercilii muscle—for a total dose in the active treatment groups of 20 units. Botulinum toxin type A blocks neuomuscular transmission by binding to acceptor sites on motor nerve terminals, entering the nerve terminals, and inhibiting the release of acetylcholine. One unit corresponds to the calculated median intraperitoneal lethal dose (LD50) in mice. Each vial of botulinum toxin type A contains either 100 units of Clostridium botulinum type A neurotoxin complex, 0.5 mg of albumin (human), and 0.9 mg of sodium chloride, or 50 units of C. botulinum type A neurotoxin complex, 0.25 mg of albumin (human), and 0.45 mg of sodium chloride in a sterile, vacuum-dried form without a preservative.

2017 patient with facial hyperkinesia comes to the office for treatment with botulinum toxin type A for temporary improvement in the appearance of moderate to severe glabellar facial lines. How many units of botulinum toxin type A should be administered to this patient, according to the Food and Drug Administration? A) 1 B) 10 C) 20 D) 50 E) 100

The correct response is Option B. The findings on the panoramic x-ray study (Panorex) are most consistent with a dentigerous cyst, also referred to as a follicular cyst. These lesions are not tumors, but cysts that originate from the separation of the dental follicle from the crown of an unerupted tooth. Thus, dentigerous cysts arise during development and are odontogenic (from the tooth or its precursors) in origin. The cysts are lined by specialized epithelium that is instrumental in the formation of tooth enamel. This tissue usually atrophies and becomes part of the gingiva after the enamel is formed; a cyst forms when fluid accumulates between the atrophied or reduced enamel epithelium and the crown of an unerupted tooth. These cysts are most common around the mandibular third molar (wisdom tooth) and are seen most often in teenagers/young adults. They present as a painless bone expansion and appear on x-ray study as a well-circumscribed, unilocular radiolucency, often with a sclerotic rim. The relationship between the cyst and the tooth varies. Giant cell tumor, desmoid tumor, and osseous tumors associated with tuberous sclerosis are solid, not cystic lesions. The former two lesions tend to be more erosive on x-ray study and have a largely solid composition. Bony tumors arising in the context of tuberous sclerosis are uncommon, but tend to be solid. Cherubism is a rare autosomal dominant disorder that begins in childhood and in which mandibular and maxillary bone is replaced by fibrous tissue and cysts. It reportedly improves over time but can be disfiguring.

2018 A 21-year-old woman is evaluated for a painless enlargement along the left lower jaw. Panoramic x-ray study (Panorex) is shown. Which of the following is the most likely diagnosis? A) Cherubism B) Dentigerous cyst C) Desmoid tumor D) Giant cell tumor E) Tuberous sclerosis

The correct response is Option A. This technique for internal fixation of mandibular angle fractures was first described by Michelet in 1972, and biomechanical studies by Champy et al were published in 1976. In his original paper, Champy advocated placement of an internal fixation miniplate along the external oblique line of the mandible. By contouring the plate to this line, rigid fixation is achieved in two planes, taking advantage of the force vectors along the angle of the mandible to help compress the fracture site. Mylohyoid line is not appropriate because the mylohyoid line is located along the lingual surface of the mandibular body and is not used routinely as a location for plate fixation for mandibular fractures. Lingula is not appropriate because the lingula is located along the lingual surface of the mandibular ramus, and is the location where the mandibular nerve (V3) enters the mandible. Plate fixation should be avoided in this area because of the difficulty of exposure and the risk for damaging the mandibular nerve. Mental tubercle is not appropriate because the mental tubercle is not an appropriate place to apply internal fixation when utilizing the Champy technique. Mandibular notch is not appropriate because it is located between the mandibular condyle and the coronoid process and is not an appropriate place to apply internal fixation when utilizing the Champy technique.

2018 A 25-year-old man with a fracture of the mandibular angle is evaluated for open reduction and internal fixation. Which of the following is the most appropriate location for placement of internal fixation when using the Champy technique in this patient? A) External oblique ridge B) Lingula C) Mandibular notch D) Mental tubercle E) Mylohyoid line

The correct response is Option A. Parotidectomy is a surgical procedure performed for benign and malignant tumors of the parotid gland and, uncommonly, for hypersalivation. Specific risks for parotidectomy include facial nerve injury, sialocele formation, and gustatory sweating. Gustatory sweating (or Frey syndrome) results when there is aberrant innervation of cutaneous sweat glands in the cheek skin overlying the parotid bed resulting in sweating and flushing during eating. Loss of taste to the anterior two thirds of the tongue results from injury to the chorda tympani, which arises from the facial nerve as it crosses the tympanic cavity and joins with the lingual nerve, not from injury to the facial nerve within the parotid gland. The stapedius is a small muscle in the middle ear that restricts movement of the stapes bone in response to loud noises. Hyperacusis may occur when the facial nerve is interrupted intracranially, before it exits the stylomastoid foramen and becomes extracranial. Tearing while eating (crocodile tear syndrome or Bogorad syndrome) is an uncommon consequence of nerve regeneration subsequent to Bell's palsy in which efferent fibers from the superior salivary nucleus become improperly connected to nerve axons projecting to the lacrimal glands causing tears while smelling foods or eating. Tardive dyskinesia is a disorder resulting in involuntary repetitive movements including grimacing, tongue and lip movements, and excessive blinking that usually occur as side effects of antipsychotic drugs. It is unrelated to facial nerve dysfunction.

2018 A 45-year-old man with mucoepidermoid carcinoma of the left parotid gland undergoes parotidectomy. Which of the following complications is most likely in this patient? A) Gustatory sweating B) Hyperacusis C) Loss of taste D) Tardive dyskinesia E) Tearing while eating

The correct response is Option A. The area of the lips, gums, teeth, tongue, and anterior hard palate will drain to the submental and submandibular region (Level I). Levels IA (submental) and IB (submandibular) are separated by the anterior digastric muscle. Group II drains the naso-/oro-/hypopharynx and parotid and is called the upper jugular group. Group III ( middle jugular group) drains the naso-/oro-/hypopharynx and larynx. The lower jugular group (level IV) drains the larynx, cervical esophagus, and hypopharynx. Group V is the posterior triangle group and drains the naso- and oropharynx. Group VI is the anterior central group below the hyoid and above the sternal notch. The group drains the thyroid, parathyroid, cervical esophagus, and larynx. More posteriorly the hard palate and soft palate will drain to the retropharyngeal space and deep cervical nodes.

2018 A 59-year-old woman who has worked in rubber manufacturing for 35 years is referred by her primary care physician because of a firm, nontender, 1-cm mass with ulceration on the hard palate between the central incisors and the incisive foramen. This suspected minor salivary gland tumor is most likely to drain into which of the following nodal basins? A) Submandibular nodes B) Parathyroid nodes C) Parotid nodes D) Posterior triangle nodes E) Occipital nodes

The correct response is Option E. Timing of pediatric orbital floor fractures is well studied. Unlike adult fractures, significant delays for surgery in children, especially more than 7 days after injury, is associated with varying degrees of diplopia. Many consider this pathology an emergency and should be treated within 24 hours. Assuming a typical, standard of care approach is performed well from a technical standpoint, only delays in time to treat were shown to predict such a poor outcome.

2018 A 7-year-old boy is brought to the emergency department after being injured in a domestic violence incident. Physical examination shows bruising around the right eye. The patient reports pain and nausea when looking upward. A CT scan shows an entrapped inferior rectus muscle. Three weeks later, the floor of the orbit is repaired with an orbital floor implant. One year later, he continues to have diplopia. Which of the following is the most likely reason for the persistent diplopia? A) Exophthalmos B) Location of prosthesis C) Nerve damage D) Persistent swelling E) Timing of surgery

The correct response is Option D. Rhinophyma occurs on the far end of progressive acne rosacea involving the sebaceous glandular overgrowth of the skin. It most commonly occurs on the nose but phymas can occur elsewhere on the face. In early stages, rhinophyma can be treated with isoretinoin and antibiotics; however, in advanced states it needs to be resected. Ablation modalities include dermabrasion, carbon dioxide laser excision, scalpel excision, or a combination of techniques. Since the procedure resects the skin to mid-dermis, skin grafting or other coverage is unnecessary and can lead to unsatisfactory aesthetic results. The best option is excision with secondary epithelialization.

2018 A 72-year-old man comes to the office for evaluation of a 2-cm growth over the lower half of his nose that has been growing slowly during the past 15 years. A photograph is shown. Medical history includes acne rosacea. Physical examination shows a broad, exophytic mass. Which of the following is the most appropriate management? A) Application of 5-fluorouracil B) Direct excision and coverage with a forehead flap C) Direct excision and coverage with a skin graft D) Direct excision and healing by secondary intention E) Observation only

The correct response is Option A. One of the branches of the facial artery is the submental artery. This vessel provides flow to the so-called submental flap, which can be made myofascial or musculocutaneous as it includes the platysma muscle. The flap is almost adjacent to the defect and provides similar skin in color and texture. The donor site is typically closed primarily after undermining the lower neck. The thoracoacromial artery provides flow to the pectoralis flap. Although the pectoralis myocutaneous flap could be used here, it would be bulky, would have to exclude breast tissue, would not have a similar color match, and would create a tether from the chest to the chin. The superficial temporal artery can allow for a large laterally based forehead flap; in the era prior to free tissue transfer, this flap was used for even intraoral reconstruction. In theory, such a flap could reach the mental area, but the dissection would be very tedious and the donor site alterations significant. The transverse cervical artery can provide a flap of thin supraclavicular skin that, as a free flap, could be used in this area but certainly would not be the simplest option. Likewise the radial forearm free flap based on the radial vessels would not be a good color match and would be a lengthy, complex surgery.

2018 50-year-old woman has a Mohs resection defect starting 15 mm below her lower lip vermilion border and extending 3 cm caudally; the width of the defect is 4.5 cm. Exposed bony mandible is noted in the depth of the wound. A photograph is shown. The simplest and most aesthetically pleasing reconstruction is likely to be based on which of the following arterial branches? A) Facial B) Radial C) Superficial temporal D) Thoracoacromial E) Transverse cervical

The correct response is Option B. This patient has significant hypernasality in the face of a prior straight-line cleft palate repair, but does not have an oronasal fistula. Improving hypernasal speech in patients with prior cleft repairs can be addressed with several different techniques, including sphincter pharyngoplasty, posterior pharyngeal flap, and conversion Furlow palatoplasty. This particular patient has vaulting of the velum, indicative of anterior placement of the levator veli palatini muscles, and also appears to have good palatal elevation. Both of these make him a promising candidate for a conversion Furlow palatoplasty. Posterior pharyngeal flap surgery would be less preferable, given his sleep apnea type symptoms and good palatal mobility. Further speech therapy would be unlikely to provide the patient with any significant improvement in hypernasality. Although tonsillectomy may help with his sleep apnea type symptoms, it is unlikely to provide the patient with significant speech improvement. Although posterior pharyngeal flap surgery is an option in patients with hypernasal speech following palatoplasty, this patient has a history of sleep apnea type symptoms and a mobile velum. Posterior pharyngeal flap would be preferable in a patient with poor motion of the velum, a coronal velopharyngeal port defect, and good lateral wall motion. The use of a palatal lift device can be beneficial in patients with velopharyngeal insufficiency and poor palatal motion who are not candidates for or do not wish to undergo surgery.

2018 A 10-year-old boy with a history of obstructive sleep apnea and prior straight-line cleft palate repair is brought for evaluation of stigmatizing, hypernasal speech. There is no oronasal fistula present, and the velum appears to vault, but the palate elevates well with phonation. Which of the following is the best option for improvement of speech in this patient? A) Continued speech therapy B) Furlow palatoplasty C) Posterior pharyngeal flap D) Tonsillectomy E) Use of a palatal lift device

The correct response is Option C. This CT scan shows right-body and left-angle fractures, both with clinically significant displacement. Modern facial fracture management dictates early primary reduction and repair; therefore observation and soft diet without surgery is incorrect, as is avoiding disturbance of the mandible and allowing 6 weeks of bone healing. Since she has reached the age of skeletal maturity permanent titanium hardware is an accepted modality of treatment; therefore it is incorrect to avoid hardware and rely on non-reduced maxillomandibular fixation. In addition, an angle fracture cannot typically be treated with maxillomandibular fixation alone, but requires a Champy plate or more rigid fixation. Open reduction and internal fixation of the right-body fracture, without repairing the left angle is incorrect, as Champy lines do not preclude the need for fixation. Open reduction and internal fixation of both the right body and the left angle fractures is correct, as both fractures need to be reduced, and then fixated, for the best chance of restoring occlusion

2018 A 16-year-old girl is brought to the emergency department after sustaining fractures to the mandible in a rollover motor vehicle collision. She is intubated during the primary survey because of her critical airway. A CT scan is shown. Which of the following is the most appropriate treatment of this fracture pattern? A) Maxillomandibular fixation only B) Observation and soft diet C) Open reduction and internal fixation of both the right body and the left angle fractures D) Open reduction and internal fixation of the right body fracture only E) Secondary mandible reconstruction after 6 weeks

The correct response is Option E. The above patient scenario is typical of a cleft lip and palate patient with missing lateral incisor on the cleft side and significant maxillary growth restriction leading to class III malocclusion. There are multiple options for surgical correction of the deformity described, however they must address the patient's complaints of malocclusion, address the missing dental unit, and provide a lasting reconstruction. The best option is a two-piece Le Fort I advancement with a surgical canine substitution. This allows restoration of the dental arch and closure of the space, with the canine filling the lateral incisor spot. Postoperatively, the prosthodontist will recontour the canine so it appears like a lateral incisor. This patient is likely skeletally mature at 17 years old (14 to 16 years for females, 16 to 18 years for males). Bilateral sagittal split osteotomy would address the malocclusion without addressing the mid face deficiency. However, there is not enough bone stock for a standard osseointegrated implant, which usually requires at least 10 to 15 mm of alveolar height. Although a fixed bridge will address the missing tooth, neither it nor alveolar transport distraction will fix her malocclusion.

2018 A 17-year-old girl with a left unilateral cleft lip and palate comes to the office because she is dissatisfied with her occlusion and facial appearance. Physical examination shows midface hypoplasia and class III malocclusion. Intraoral examination demonstrates 6 mm of negative overjet and a missing left lateral maxillary incisor with retained space. Occlusal x-ray studies demonstrate 5 mm of alveolar bone height at the cleft margin. Which of the following is the best treatment option for this patient? A) Alveolar transport distraction B) Observation until skeletal maturity C) Prosthodontic rehabilitation with a fixed bridge D) Sagittal split of the mandible with osteointegrated implants E) Two-piece Le Fort I osteotomy with closure of the alveolar gap

The correct response is Option E. This patient has sustained a three-dimensional and sizable defect of the ear. He is missing a portion of the helical rim, the scaphoid fossa to the antihelix. This is a difficult reconstructive challenge. The anatomic components that are missing are skin and underlying cartilage. The best option for obtaining a normal-appearing ear would be reconstruction with a cartilage framework carved to replace the cartilage and to support the overlying skin to retain the form of the ear. This could be done as a stage procedure. Fat injection would not be adequate to replace the missing components or to give adequate shape to the ear. A silicone framework within a scarred bed would be prone to complication, most notably infection and/or extrusion. The defect is too large to be replaced by a composite cartilage graft. Although a temporoparietal fascia flap could potentially be used to cover a cartilage framework, used alone it would not give adequate form to reconstruct this defect.

2018 A 19-year-old man sustains a partial amputation fight-bite injury to the ear. The wound has been revised and a photograph of the residual defect is shown. The patient wants the ear to have a normal appearance. Which of the following is the best option to restore normal aesthetics to the ear? A) Addition of a silicone framework B) Composite cartilage grafting C) Reconstruction with a temporoparietal fascia flap with skin grafting D) Serial fat injections E) Staged autologous reconstruction with costal cartilage

The correct response is Option E. Median cleft lip results from failed fusion of the medial nasal prominence during embryologic development. A unilateral cleft lip results from failed fusion of the medial nasal prominence and the maxillary prominence. An oblique facial cleft results from failure of fusion of the lateral nasal prominence and the maxillary prominence. A lateral oral commissure cleft is produced by the failed fusion of the mandibular and maxillary prominences. A cleft of the primary palate is produced by failure of fusion of the medial and lateral palatine processes.

2018 A 2-year-old has a midline cleft of the upper lip. This cleft results from failure of fusion of which of the following embryologic structures? A) Lateral nasal prominence and maxillary prominence B) Maxillary and mandibular prominences C) Medial and lateral palatine processes D) Medial nasal prominence and maxillary prominence E) Medial nasal prominences only

The correct response is Option C. Stensen duct enters the buccal mucosa at the level of the second maxillary molar, not the mandibular molar. The mental foramen, where the mental nerve enters, is located at the level of the second mandibular premolar. The second maxillary premolar is not a landmark for nerves or ducts. The palatine foramen, where the descending palatine artery and greater palatine nerve exit, is located at the level of the maxillary third molar.

2018 A 20-year-old man is evaluated after his face was slashed during an attack with a broken beer bottle. Physical examination shows a curvilinear laceration from the lateral corner of the palpebral fissure to the angle of the mandible. Which of the following best describes the location of the Stensen duct ampulla? A) Second mandibular molar B) Second mandibular premolar C) Second maxillary molar D) Second maxillary premolar E) Third maxillary molar

The correct response is Option B. Temporomandibular joint (TMJ) dysfunction symptoms are serious, often overlooked complications of facial fractures and their treatments. They can range from clicking and pain to locking, malocclusion, and trismus. Overt ankylosis can occur in rare circumstances. Fractures that result in significant disruption of the condylar/glenoid apparatus are more likely to result in TMJ dysfunction symptoms than more anatomically remote fractures. Condylar fractures are most susceptible to post-fracture TMJ dysfunction. This is especially true in comminuted condylar head fractures. One recent study demonstrated an increase in TMJ dysfunction symptoms in patients with condylar fractures and concomitant contralateral mandibular body/angle fractures. Le Fort I and zygomaticomaxillary complex (ZMC) fractures are unlikely to be associated with TMJ symptoms.

2018 A 22-year-old man is evaluated for multiple facial fractures after he was assaulted. Which of the following fractures is most likely associated with an increased risk of temporomandibular joint dysfunction? A) Bilateral parasymphyseal mandible B) Comminuted unilateral condylar mandible C) Complete Le Fort I maxillary D) Displaced unilateral subcondylar mandible E) Unilateral zygomaticomaxillary

The correct response is Option E. The vestibular approach to the mandible is useful for a number of mandibular procedures. The most important neurovascular structure of significance in the region of the symphysis is the mental neurovascular bundle. The mental nerve is the terminal branch of the inferior alveolar nerve. It gives sensation to the skin and mucosa of the lower lip, skin of the chin, and facial gingiva of the anterior teeth. It is important to preserve this nerve during the surgical dissection in this approach. The mental nerve exits the mental foramen usually inferior to or slightly anterior to the second premolar tooth.

2018 A 22-year-old man is scheduled to undergo surgery to correct a displaced symphyseal fracture of the mandible using a vestibular approach. To avoid injury to the mental nerve, the mental foramen is best identified in which of the following locations? A) Anterior to the first premolar B) Inferior to the lateral incisor C) Inferior to the lower canine tooth D) Inferior to the second molar E) Inferior to the second premolar

The correct response is Option B. The patient described has benign or idiopathic masseter hypertrophy. The classic patient comes to the office because of aesthetic complaints but is otherwise asymptomatic. There are numerous treatments, including partial myotomy, mandibular angle reduction, and contralateral augmentation. Of all the options, the best is botulinum toxin type A injections. These injections are minimally invasive and reversible, and they do not require an anesthetic. On the basis of its low-risk profile, it is the best first-line treatment listed. Amyloidosis is a progressive systemic disease that can cause myopathy and painful bilateral masseter hypertrophy. It is unlikely in this stable, unilateral case. Bruxism can cause masseter hypertrophy, which is most often bilateral. A bite guard will prevent tooth damage, but it will not correct hypertrophy.

2018 A 25-year-old woman comes to the office because of a 5-year history of stable facial asymmetry. Physical examination shows fullness of the right mandibular angle; no facial tenderness, masses, or lymphadenopathy is noted. Maxillofacial CT scan shows enlargement of the right masseter muscle compared with the left. Which of the following is the most appropriate first-line treatment for this problem? A) Fabrication of a bite guard to treat bruxism B) Injection of botulinum toxin type A into the right masseter muscle C) Open subtotal myectomy of the right masseter muscle D) Placement of a left mandibular angle prosthesis E) Referral to oncology to rule out systemic amyloidosis

The correct response is Option A. Autologous bone is the most appropriate material for inlay cranioplasty for a child this age. Each of the other options has been used in the pediatric population, but they carry important limitations. Porous polyethylene can be used to reconstruct large cranial defects, but is expensive to fabricate, has the potential to become unstable in a growing cranium, and has a higher overall infection rate than autologous bone. Bone morphogenetic protein (BMP) has shown promise in animal models for inducing bone formation in large cranial defects, but it is expensive for a defect of this size; it is a powerful mitogen and can result in dural ossification and, potentially, induction of tumor growth. Calcium phosphate pastes also have been advocated but are very expensive and for a defect of this size, the long-term potential osseointegration is dubious. Furthermore, the risk of infection and/or implant extrusion is higher. Titanium mesh may be appropriate for some children with limited life expectancy or functional needs, but these implants can be bent, offer limited impact

2018 A 3-year-old boy is brought for evaluation of a right frontotemporal cranial defect. A photograph is shown. Which of the following materials is most appropriate for reconstruction? A) Autologous bone B) Bone morphogenetic protein (BMP) C) Calcium phosphate paste D) Porous polyethylene E) Titanium mesh

The correct response is Option D. A higher prevalence of dental anomalies is expected in children with cleft lip and palate. Agenesis is the most prevalent anomaly found typically in greater than 50% of patients. The most commonly affected tooth is the permanent lateral incisor on the cleft side. Supernumerary teeth are the second most frequently occurring dental anomaly. Dysplastic teeth, ectopic teeth, and translocation of the teeth are all less common than tooth agenesis in this patient population

2018 A 3-year-old girl with a cleft lip and palate is brought to the clinic because her parents are concerned that her teeth are not coming in properly. Given her diagnosis, which of the following is the most likely finding? A) Dysplastic teeth B) Ectopic teeth C) Supernumerary teeth D) Tooth agenesis E) Tooth translocation

The correct response is Option E. The photographs show Stahl ear, a congenital ear anomaly that is characterized by an abnormal third crus of the antihelix. This is a fairly uncommon deformity, and multiple surgical methods have been described. Compared with other ear anomalies, there is usually enough local cartilage in Stahl ear, so most authors recommend local cartilage flaps or grafts, and otoplasty techniques, to remove the third crus and reconstruct a more normal antihelical fold. Ear anomalies that lack significant cartilage may require larger cartilage grafts, such as rib. Multi-stage surgery is for reconstruction of microtia, which has a severe paucity of normal cartilage and soft-tissue structures.

2018 An 8-year-old boy is brought to the clinic with a right congenital ear anomaly that affects his interaction with his peers. A photograph is shown. Which of the following is the most appropriate treatment? A) Ear molding B) Multiple stages of surgery for rib cartilage construct, elevation of construct, and soft-tissue reconstruction C) Surgery with cadaveric cartilage D) Surgery with distant rib cartilage E) Surgery with local cartilage

The correct response is Option A. Any penetrating injury on a line from the tragus to the mid upper lip may injure the parotid duct or gland. The parotid system can be divided into 3 regions: Region A: gland Region B: duct superficial to the masseter muscle Region C: duct from the masseter to where it enters the mouth opposite the 2nd maxillary molar Additionally, the buccal branch of the facial nerve often runs with the duct. It can cross the superficial layer of the masseter after leaving the parotid gland. Injury manifests as weakness of the upper lip on animation. Injury not repaired acutely can manifest later as salivary fistulae or sialoceles. Most can be managed conservatively with drainage, pressure, and antisialagogues. Trying to find and repair the duct late after the injury is difficult and may not lead to resolution of the symptoms. This is usually reserved for early injury and repair. If these fail, then a superficial parotidectomy can be considered for recalcitrant salivary collections.

2018 A 32-year-old man is evaluated because of painless swelling of the right cheek 2 months after being assaulted with a knife and sustaining a laceration across the mid cheek. The laceration was sutured by the emergency department physician assistant. Which of the following is the most appropriate management at this time? A) Drainage of the collection and administration of antisialagogues B) Open debridement with skin grafting C) Surgical exploration with reanastomosis of the injured duct D) Total parotidectomy with facial nerve preservation E) Ultrasound-guided needle aspiration only

The correct response is Option C. The photograph demonstrates through transillumination an absent normal levator muscle sling found in an occult submucous cleft palate. Velopharyngeal insufficiency (VPI) after tonsillectomy is a rare but known complication but can occur as frequently as 1:3000 to 1:10,000 tonsillectomies. Most of these complications occur in patients with either an occult or overt submucous cleft palate. With this anatomic variant, the principle of repair is either to restore normal anatomy with a two-flap palatoplasty or to pair the levator muscles into a functional sling. Techniques to lengthen the palate, such as a Furlow repair, help to obturate the nasal escape and also to improve VPI speech. As such, 8 weeks after VPI onset, the speech pathologist will have difficulty correcting the anatomic deficiency of lacking levator sling. The tensor tenopexy stabilizes the tendon of the tensor tympany to the hamulus during cleft palate repair, and stents open the eustachian tube but will not improve VPI speech.

2018 A 33-year-old woman is evaluated for hypernasal speech and nasal escape with phonation. She underwent removal of the tonsils and adenoids 10 weeks ago. Transillumination of the palate is shown. Which of the following is the most appropriate approach to restore normal speech in this patient? A) Fat grafting to tonsil pillars B) Orticochea pharyngoplasty C) Palatoplasty D) Tensor tenopexy E) Observation and speech pathology

The correct response is Option E. Saethre-Chotzen syndrome is an autosomal dominant disorder associated with TWIST1 gene mutation. It causes an asymmetrical brachycephaly, bicoronal or unicoronal synostosis, low frontal hairline, ptosis of the eyelids, and a prominent crus of helix. Crouzon syndrome is an autosomal dominant disorder associated with an FGFR2 gene mutation. The patient with this syndrome typically comes to the office because of hydrocephalus, elevated intracranial pressure, Chiari I malformation, bicoronal synostosis and brachycephaly, exorbitism, mid-face hypoplasia, anterior open bite, and normal extremities. Apert syndrome is an autosomal dominant disorder and shows symptoms of elevated intracranial pressure, bicoronal synostoses, turribrachycephaly, enlarged anterior fontanel, bitemporal widening, occipital flattening, anterior open bite, and complex syndactyly of the hands and feet. Pfeiffer syndrome is also an autosomal dominant disorder associated with FGFR2 mutation and reveals symptoms of hydrocephalus and a high risk of Chiari malformation, turribrachycephaly with bicoronal synostoses, exorbitism, hypertelorism, down-slanting palpebral fissures, an anterior open bite, and broad thumbs and/or broad halluces. Muenke syndrome is associated with the FGFR3 gene mutation and comes to the office with coronal synostosis, elevated intracranial pressure, sensorineural hearing loss, and abnormal middle phalanges.

2018 A 4-year-old boy is brought to the office for evaluation of the shape of his head. Physical examination shows an asymmetrically shaped head, low frontal hairline, ptosis of the eyelids, and a prominent crus of helix. Which of the following syndromes is most likely affecting this child? A) Apert B) Crouzon C) Muenke D) Pfeiffer E) Saethre-Chotzen

The correct response is Option A. Since the early 1990's when positional deformational cranial deformities were first identified with the "back-to-sleep" campaigns, they have now become the most common etiology of cranial and facial asymmetry. Prior to that time, torticollis was the most common cause. Craniosynostosis is a much less frequent cause and occurs in the following order of decreasing frequency: unicoronal, lambdoid, zygomaticotemporal. Correct identification of most of the causes of this clinical constellation of findings can be made on clinical examination, but most would still get a CT scan to verify the diagnosis and rule out associated anomalies except for torticollis, which is always a clinical diagnosis.

2018 A 5-year-old boy is referred for evaluation of facial asymmetry. His parents report that they have noticed asymmetry since birth and believed it would improve with time, but it has not. The patient has no history of surgery or physical therapy. On physical examination, the physician notes skull asymmetry and facial asymmetry, including the orbits; there is no obvious neck tilt. A CT scan of the head is most likely to show which of the following? A) Deformational plagiocephaly B) Lambdoid craniosynostosis C) Torticollis D) Unicoronal craniosynostosis E) Zygomaticotemporal craniosynostosis

The correct response is Option B. Reconstructive strategies for partial and hemiglossectomy reconstruction are different than for subtotal and total glossectomy reconstruction. For partial and hemiglossectomies, the ideal reconstruction preserves the mobility of the remaining tongue, maintaining its ability to manipulate foods, articulate words, and sweep the oral cavity clean. Such defects are best reconstructed with a thin, pliable flap that resists contraction during the healing process. Of the choices listed, the radial forearm fasciocutaneous free flap best fits those requirements. The pectoralis major myocutaneous pedicled flap is usually too thick to permit ideal movement and can have problems with reach and tethering to the chest as the proximal flap and pedicle contact during healing. The temporalis muscle pedicled flap is unlikely to reach the entire defect and the muscle is likely to contract and stiffen with time, limiting tongue movement. The rectus abdominis myocutaneous free flap is usually too thick to permit unrestricted movement of the remaining hemitongue and is better suited to subtotal and total glossectomy reconstruction, which is best effected with a bulky flap that diverts food and liquids laterally into the pharynx rather than into the larynx. A split-thickness skin graft, while thin, usually contracts substantially, limiting tongue movement. Also, if there is communication into the neck following tongue resection with neck dissection, the skin graft would be at high risk for resulting in a fistula.

2018 A 60-year-old woman undergoes left hemiglossectomy for squamous cell carcinoma of the oral tongue. Reconstruction with which of the following is most likely to achieve the best functional outcome? A) Pectoralis major myocutaneous pedicled flap B) Radial forearm fasciocutaneous free flap C) Rectus abdominis myocutaneous free flap D) Split-thickness skin graft E) Temporalis muscle pedicled flap

The correct response is Option E. Defects of the lip can be categorized by the percentage of the total lip affected. Small fullthickness defects of the lip (25-33%) as described in this case are best repaired with primary closure using a vertical excision of remaining lip structures to enable a tension-free closure. Care should be taken to precisely realign the lip and repair the white roll to avoid step-offs that are easily noticeable. Larger defects (33-50%) are usually repaired using flaps. These flaps can be categorized as transoral flaps (e.g., Abbe or Estlander flaps) or circumoral advancement/rotation flaps (e.g., Gilles flap or Karapandzic technique). Subtotal lip defects may be repaired with bilateral circumoral advancement/rotation flaps, while total lip defects generally require reconstruction with free flaps. Skin grafts are rarely used for lip reconstruction (particularly if the white roll is involved) because these repairs result in a patch-like appearance that is highly noticeable.

2018 A 62-year-old woman is evaluated 1 hour after undergoing Mohs micrographic resection of a squamous cell carcinoma of the lower lip. After a clear-margin excision, the resulting defect is a full-thickness 25% central lip defect. Which of the following is the most appropriate repair method for this defect? A) Coverage using the Karapandzic technique B) Coverage with a V-Y advancement flap C) Coverage with an Abbe flap D) Full-thickness skin grafting E) Primary closure

The correct response is Option E. The temporal branch of the facial nerve is found just deep to the temporoparietal fascia. The facial nerve exits the stylomastoid foramen and the main trunk, pes anserinus, can be found 1 cm inferior and posterior, midway between the tragal pointer and the posterior belly of the digastric muscle. It then arborizes into 5 branches; temporal, zygomatic, buccal, marginal and cervical. Interconnections between the zygomatic and buccal offer some additional regenerative potential if one of those branches is injured. Most mimetic facial muscles are innervated from the deep surface such as the temporalis. Exceptions are the buccinator, levator anguli oris, and mentalis. Injury to the temporal or cervical branches can leave more lasting deformities so management of injury is important. Avoiding injury is the best way to prevent injury. Therefore, when dissecting in the temporal zone it is critical to avoid entering deep to the temporoparietal fascia.

2018 A 63-year-old woman undergoes a rhytidectomy with brow lift. Ipsilateral weakness of the forehead muscles is noted in the recovery room. The suspected injury is most likely immediately deep to which of the following? A) Deep layer of the deep temporal fascia B) Subdermal connective tissue C) Superficial layer of the deep temporal fascia D) Superficial temporal fat pad E) Temporoparietal fascia

The correct response is Option D. Hyponasal speech describes the sound production when not enough air gets through the velopharyngeal sphincter, resulting in a muffled nasal voice, as if someone was holding his or her nose. This is in contrast to hypernasal speech, where the velopharyngeal sphincter is incompetent, resulting in leakage of excess air with an airy nasal voice. In this scenario, the most likely reason for hyponasal speech is an exuberant posterior pharyngeal flap that is blocking too much of the velopharyngeal complex. The two reasons to treat this are for speech issues that prevent normal interaction and obstructive sleep apnea (OSA). Given that the patient has no problems with people understanding her speech, the most reasonable next step among the choices is polysomnography to evaluate for OSA. None of the other choices are appropriate workups for patients with hyponasality. Some centers may proceed with video nasendoscopy to look at the anatomy, but this was not one of the answer choices.

2018 A 7-year-old girl with a history of cleft lip and palate is brought to the office because of hyponasal speech, which is confirmed by the speech language pathologist. The patient and her family report that people have no trouble understanding her speech. Medical history includes primary cheiloplasty, primary rhinoplasty, primary palatoplasty, and posterior pharyngeal flap. Which of the following is the most appropriate next step in management? A) CT scan of the face B) Genetics referral C) MRI of the palate D) Polysomnography E) Swallow study

The correct response is Option A. The Cutler-Beard flap is the preferred flap for full-thickness defects of the upper eyelid that are greater than 66% of total eyelid width. This technique utilizes a full-thickness eyelid flap from the lower eyelid to provide a wide flap for eye coverage. In the first stage, the flap is raised and advanced into the upper eyelid defect. The flap is then inset, covering the eye. The inferior pedicle is divided in a second stage at 4 to 6 weeks and final flap inset is performed. The Tenzel semicircular flap is an excellent technique for anterior lamellar defects of the upper or lower lid, but requires a second flap for posterior lamella reconstruction of fullthickness defects and is not generally the preferred technique for a large defect of the upper eyelid. The Hughes tarsoconjunctival flap is a similar procedure to the Cutler-Beard flap, but is used for lower lid defects. Full-thickness skin grafting would not restore the posterior lamellar defect, and direct closure would not likely be able to close a 70% defect of the upper eyelid properly

2018 A 70-year-old man has a biopsy-confirmed squamous cell carcinoma of the upper eyelid. Excision of the lesion will produce a full-thickness defect of the central upper eyelid that is 70% of its width. Reconstruction of the defect with which of the following is most appropriate? A) Cutler-Beard flap B) Full-thickness skin graft C) Hughes flap D) Primary closure E) Tenzel flap

The correct response is Option D. Crouzon-related obstructive sleep apnea and increased intracranial pressure are best treated with frontofacial advancement without adjustment of the orbital width, which is best achieved using the monobloc osteotomy. Orbital box osteotomy will not treat increased intracranial pressure or sleep apnea. Le Fort III osteotomy will treat obstructive sleep apnea but will minimally affect increased intracranial pressure. Lastly, facial bipartition osteotomy is not needed in this case because of the normal intercanthal distance and lack of telorbitism. Le Fort I addresses neither the orbits nor the intracranial pressure.

2018 A 9-year-old boy with Crouzon syndrome is brought to the craniofacial clinic because of obstructive sleep apnea. Apnea hypopnea index is 40. He has a negative overjet of 12 mm, moderate exorbitism, and signs of increased intracranial pressure (>25 mm Hg), which was found during an intracranial pressure monitoring trial last month; intercanthal distance is normal (25 mm). Which of the following types of osteotomy is the most appropriate treatment for this patient? A) Facial bipartition B) Le Fort I C) Le Fort III D) Monobloc E) Orbital box

The correct response is Option C. This is a macrocystic lymphatic malformation (LM) that was prenatally diagnosed. Former terms for this malformation include cystic hygroma. The most important management issue is control of the airway, and sometimes these children have to be delivered by EXIT procedures. After delivery, the airway should be evaluated and secured before making management decisions. While some macrocystic LMs can be resected, this is a risky procedure in a neonate, and this LM extends toward midline structures. Sclerotherapy can be performed in neonates and is now the first choice of treatment. Infantile fibrosarcoma appears different from LM on MRI. Infantile fibrosarcomas appear as heterogeneous vascular masses and not as a cystic structure. Propranolol is efficacious in the treatment of problematic infantile hemangiomas. Infantile hemangiomas do not start proliferating until after birth, so this is not an infantile hemangioma. A congenital hemangioma would also appear as a vascular heterogeneous mass rather than a cystic structure, and there is no evidence that congenital hemangiomas respond to propranolol therapy

2018 A full-term baby is born via cesarean delivery because of a prenatally diagnosed mass in the posterior neck. An MRI is shown. After birth, there is no airway compromise. Which of the following is the most appropriate next step in management? A) Administration of propranolol B) Incisional biopsy C) Sclerotherapy D) Surgical resection E) Observation only

The correct response is Option D. The patient described meets the criteria for neurofibromatosis 1 by clinical diagnosis. Genetic testing is also possible. To make a clinical diagnosis, two or more of the following criteria are required: • Six or more café au lait spots 5 mm or larger (prepubertal) or 15 mm or larger (postpubertal) • Family history • Two or more neurofibromas • Freckling in armpits or groin • Lisch nodules • Bony dysplasia (sphenoid wing in this case) • Tumor on optic nerve A patient with Goldenhar syndrome has facial asymmetry, but does not have café au lait spots. Colobomas are common in patients with this syndrome, but Lisch nodules are not. Muenke syndrome is associated with craniosynostosis. Kabuki syndrome has distinctive associated facial features with long palpebral fissures, everted lower lids, broad nasal tip, arched brows, protruding ears, intellectual disability, and often microcephaly. Nevus sebaceous is a well-circumscribed hamartomatous lesion composed of sebaceous glands. There is a risk for malignant transformation.

2018 A newborn male is evaluated because of asymmetry of the orbit and eyelid. Physical examination shows a dozen 6- to 10-mm tan macules, axillary freckles, pigmented nodules on the iris, and soft, fleshy, raised, soft-tissue lesions on the forehead and scalp. Which of the following is the most likely diagnosis? A) Goldenhar syndrome B) Kabuki syndrome C) Muenke syndrome D) Neurofibromatosis E) Nevus sebaceous

The correct response is Option D. Treacher Collins syndrome, named after Edward Treacher Collins, the English ophthalmologist who first described this condition in 1900, is characterized by widely varying degrees of micrognathia, underdevelopment and/or clefting of the midfacial bones (e.g., zygoma), microtia or anotia with or without hearing loss or middle ear atresia, eyelid coloboma, and often respiratory compromise. It occurs with an estimated frequency of 1:50,000 live births, and the overwhelming majority of cases are caused by an autosomally inherited mutation in the TCOF1 gene that leads to underproduction of treacle protein, a neural crest precursor. Mutations in FGFR2, FGFR3, TWIST1, and EFNB genes are commonly associated with syndromic forms of craniosynostosis. Gain-of-function mutations in the fibroblast growth factor receptors (FGFR) can lead to various syndromes that manifest craniosynostosis including: Apert syndrome (FGFR2), Pfeiffer syndrome (FGFR1 and FGFR2), Crouzon syndrome (FGFR2), Jackson-Weiss syndrome (FGFR2), Muenke syndrome (FGFR3), Crouzonodermoskeletal syndrome (FGFR3), and thanatophoric dysplasia (FGFR3). TWIST1 loss-of-function mutations can lead to another craniosynostosis syndrome— Saethre-Chotzen syndrome. EFNB1 mutations are associated with craniofrontonasal syndrome, an X-linked disorder characterized by hypertelorism, craniosynostosis, clefting of the nasal tip, and digital anomalies.

2018 A newborn male is evaluated in the office for cleft of the soft palate. Physical examination shows severe micrognathia with labored breathing, underdevelopment of the mid face, microtia, and coloboma of the lower eyelids. Mutation in which of the following genes most likely caused these findings? A) EFNB1 B) FGFR2 C) FGFR3 D) TCOF1 E) TWIST1

The correct response is Option C. The Jones test is used to evaluate lacrimal drainage. Divided into two parts, the Jones I test investigates lacrimal outflow under normal physiologic conditions. A drop of sterile 2% fluorescein solution or a moistened fluorescein strip is placed into the conjunctival fornix and a cotton-tipped wire applicator is placed into the inferior nasal meatus in the region of the ostium of the nasolacrimal duct at 2 and 5 minutes to check for fluorescein. As this test occasionally yields abnormal results in normal patients, it is not uniformly performed. The Jones II test determines the presence or absence of fluorescein when the residual fluorescein is flushed from the conjunctival sac with clear saline to determine whether there is reflux of fluorescein. Naso-orbital-ethmoid (NOE) fractures can be challenging fractures, and either through direct instrumentation with transcanthal wiring or from the fractures themselves, the lacrimal drainage system can be affected. Postoperative epiphora can be very common and is present in at least 50% of patients who have undergone open reduction and internal fixation (ORIF) of an NOE fracture. After 3 to 6 months approximately half of this epiphora resolves, with the other half of patients (25%) requiring consideration for other investigations to evaluate lacrimal drainage. Schirmer test is used to evaluate dry eyes and is not appropriate in this patient.

2018 A patient underwent open reduction and internal fixation of naso-orbital-ethmoid fractures 12 months ago, and epiphora was noted on follow-up examination. After 6 months of observation and persistent epiphora, which of the following is the most appropriate next step to evaluate the function of the patient's nasolacrimal system? A) Conjunctivorhinostomy tube placement B) Continued observation, as function is likely to return C) Jones tests D) Lacrimal system flushing E) Schirmer tests

The correct response is Option D. Sphincter pharyngoplasty is a secondary (speech) procedure for cleft palate that rotates the posterior tonsillar pillars as superiorly based flaps to line the posterior pharynx, thereby narrowing the velopharyngeal sphincter. The posterior tonsillar pillars contain the palatopharyngeus muscles. The other muscles are not used in sphincter pharyngoplasty: levator veli palatini runs through the palate to elevate the palate; palatoglossus runs through the anterior tonsillar pillar to depress the palate; musculus uvulae is the muscle of the uvula, and tensor veli palatini contributes to eustachian tube pressure modulation.

2018 For patients with velopharyngeal incompetence, which of the following muscles is used to perform sphincter pharyngoplasty? A) Levator veli palatini B) Musculus uvulae C) Palatoglossus D) Palatopharyngeus E) Tensor veli palatini

The correct response is Option A. The pectoralis major muscle has three major blood supplies. In general, the internal mammary perforators and the thoracoacromial vessels are dominant, with additional perfusion through the lateral thoracic artery. Turnover pectoralis flaps for sternal reconstruction based on the internal mammary perforators are generally well perfused. Likewise, in most situations a pedicled myofascial or myocutaneous flap based on the thoracoacromial vessels (with the other blood supplies ligated) is robust and can be used for most types of head and neck reconstruction. Although described earlier, the flap became the "workhorse" of head and neck reconstruction after Ariyan's classic article in 1979. Versus a free tissue transfer, the pectoralis flap has many detractors, including being tethered to its pedicle. In about 6% of cases, the lateral thoracic vessel is the dominant pedicle to the flap and needs to be incorporated for maximal perfusion for head and neck reconstruction. This of course further decreases some of the mobility of the flap. This phenomenon is usually easily discernible by the larger caliber of the lateral thoracic vessels versus the thoracoacromial vessels. In this case, this was the most likely injured vessel, causing the described outcome. The transverse cervical vessels do supply the posterior thorax, and the intercostal vessels do supply the anterior chest, including breast tissue, but neither gives important perfusion to the pectoralis flap. The lateral pectoral nerve travels with the thoracoacromial vessels and, if not checked when turning the flap, can kink the vascular pedicle and, if not severed, does not allow the flap to decrease in size because of lack of denervation of the muscle. The medial pectoral nerve is another innervation to the pectoralis major muscle and is usually severed during flap elevation. The thoracodorsal artery does not supply the pectoralis muscle

2018 An 80-year-old man comes to the office because of recurrent squamous cell carcinoma of the cervical skin. Following reconstruction with a pectoralis myocutaneous flap, the distal half of the skin paddle appears ischemic and eventually exhibits necrosis and fullthickness skin loss. Transection of which of the following structures is most likely responsible for this complication? A) Lateral thoracic artery B) Medial pectoral vein C) Posterior intercostal vasculature D) Thoracodorsal artery E) Transverse cervical vessel

The correct response is Option E. The submandibular gland duct, also known as the Wharton duct, ends in the area of the floor of the mouth and is typically blocked when cancer invades this area. As a result, there is backup of salivary content, the gland enlarges, becomes firm and painful, and can even become infected. This event has no prognostic implications. However, an enlarged node, such as the perifacial nodes, can mimic this finding and can upstage a cancer patient. If a bimanual examination of the gland cannot be performed because of the patient's pain, a CT scan is easily confirmatory. In general, lymph nodes are mobile, with the rare exception of external bony erosion. Nonetheless, a complete head and neck physical examination can be key to interpreting the findings described. Bony expansion from a floor-of-mouth cancer is possible, but will not usually present as a painful mass. In general, fat is not involved. Nerve involvement can also occur, and perineural invasion in the lingual nerve can cause its enlargement, but not with this presentation. A sublingual infection or hematoma (e.g., Ludwig angina) can present with what is typically a swollen bilateral submandibular mass, as the infection/abscess extends below the mylohyoid muscle. This process can cause immediate airway compromise and many recommend an emergent tracheostomy. This pathology is almost always related to bacterial infections of the mouth, most commonly related to dental caries.

2018 An 80-year-old man presents with floor of mouth cancer on the right side. In addition to the 1-cm area of ulceration, there is a hard, warm, nonmobile, painful mass in the submandibular area. This mass most likely originated from which of the following types of tissue? A) Bone B) Fat C) Muscle D) Nerve E) Salivary

The correct response is Option C. The patient described has Goldenhar syndrome, a severe form of hemifacial microsomia with variable ear anomalies, mandibular hypoplasia leading to occlusal cant and oral commissure asymmetry, and soft-tissue deficiency on the affected side. Ocular findings associated with Goldenhar syndrome are variable and common (50% incidence) and can occur as epibulbar dermoids as described in this patient as well as microphthalmia, eyelid and optic nerve colobomas (interruption of a circular structure of the eye). Colobomas can be asymptomatic (affecting the iris alone), can lead to exposure keratopathy (in the case of eyelid colobomas), or can lead to visual disturbances (optic disc/nerve coloboma), such as visual field deficits or amblyopia. Care must be taken to identify visual disturbances early in order to implement contralateral eye penalization, or patching, to maximize visual development of the affected eye. The other tests target areas that are not affected in patients with Goldenhar syndrome.

2018 An otherwise healthy 5-year-old girl has a yellow, cystic mass on the left sclera, a left preauricular branchial cleft remnant, elevation of the left oral commissure, and softtissue deficiency of the left face. It is most appropriate to order which of the following tests to assess for significant comorbidities? A) Echocardiography B) Fluorescence in situ hybridization of the branchial cleft remnant C) Funduscopic examination D) MRI of the brain E) Plain x-ray studies of the hands

The correct response is Option A. The vascular supply of the Le Fort I osteotomy segment was studied by utilizing standard latex injection techniques. Anatomic dissections of 10 fresh cadavers demonstrated interruption of the descending palatine arteries with preservation of the ascending palatine branch of the facial artery and the anterior branch of the ascending pharyngeal artery within the attached posterior palatal soft-tissue pedicle in all specimens following Le Fort I maxillary osteotomy. These ascending arterial branches entered the soft palate at a position approximately 1 cm posterior to the pterygomaxillary junction, which was disrupted during the Le Fort I maxillary osteotomy. Separate ink injections of total maxillary osteotomy segments confirmed vascular perfusion of the ipsilateral hemimaxillary segment by the ascending palatine artery. Thus vascular supply of the mobilized Le Fort I maxillary segment is by means of the ascending palatine branch of the facial artery and the anterior branch of the ascending pharyngeal artery in addition to the rich mucosal alveolar anastomotic network overlying the maxilla.

2018 During a Le Fort I osteotomy, the descending palatine artery is disrupted and ligated. The blood supply to the mobilized maxilla is primarily from which of the following arteries? A) Ascending pharyngeal B) Greater palatine C) Internal maxillary D) Lesser palatine

The correct response is Option C. During genioplasty or horizontal osteotomy of the mandible, the genioglossus muscle is identified after down fracture and separation of the mobile and nonmobile segments. The genioglossus muscle is not only a source of blood supply but also acts to advance the tongue during skeletal sleep apnea surgery and contributes to posterior pull on potential relapse of the mobilized chin segment. This important muscle is innervated by the hypoglossal nerve.

2018 During genioglossus advancement, which of the following nerves innervates the affected muscle? A) Facial nerve B) Glossopharyngeal nerve C) Hypoglossal nerve D) Lingual nerve E) Vagus nerve

The correct response is Option C. A zygoma fracture involves displacement of the zygoma that articulates with the frontal bone, maxilla, and sphenoid. In order to stabilize the fracture after adequate reduction, the zygomaticofrontal, zygomaticomaxillary buttress, and infraorbital rim need to be fixated. If there is a large (>2 cm2 ) defect in the orbital floor after reduction, reconstruction of the orbital floor is also necessary to prevent enophthalmos. While the nasomaxillary buttress is one of the vertical buttresses of the face, the zygoma does not articulate with the nasal bones

2018 Proper reduction of a zygomaxillary complex (ZMC) fracture requires reduction and realignment of which of the following? A) Zygomatic arch, infraorbital rim, alveolus B) Zygomaticofrontal suture, infraorbital rim, alveolus C) Zygomaticofrontal suture, zygomaticomaxillary buttress, infraorbital rim D) Zygomaticofrontal suture, zygomaticomaxillary buttress, orbital floor E) Zygomaticomaxillary buttress, infraorbital rim, alveolus

The correct response is Option A. Hillocks 1 to 3 arise from the first branchial arch and give rise to the tragus, helical root, and helical crus. Hillocks 4 to 6 arise from the second branchial arch and give rise to the antihelix, antitragus, and lobule.

2018 The hillocks arise from which of the following branchial arches? A) First and second B) Second and third C) Third and fourth D) Fourth and fifth

The correct response is Option C. The patient pictured has a rare craniofacial (Tessier No. 4) cleft. Rare craniofacial clefts are characterized by variable soft-tissue and bony involvement. This patient has a cleft lip that begins lateral to the philtral column and courses laterally to the alar margin. Additionally, this patient has soft-tissue deficiency of the left medial malar region, as well as medial lower lid malposition and medial canthal dystopia. Of all of the options given, these findings suggest an increased risk for nasolacrimal duct abnormalities or inferior canalicular system malposition. Velopharyngeal dysfunction is common in patients with cleft lip and palate. Anosmia can be present in midline craniofacial anomalies, such as craniofrontonasal dysplasia. Rare craniofacial (Tessier No. 4) clefts are not associated with sensorineural hearing loss or craniosynostosis.

2018 The patient shown in the photograph is most likely to have which of the following comorbidities? A) Anosmia B) Craniosynostosis C) Epiphora D) Sensorineural hearing loss E) Velopharyngeal dysfunction

The correct response is Option C. The mechanism of palatal shelf formation must be regulated precisely so that vertical palatal shelves are adhesion-incompetent while they are in close contact with other structures, but once they are raised above the tongue, they rapidly acquire adhesion capability if they are not to remain cleft. Control of periderm differentiation by the membrane-bound signalling molecule jagged 2 (JAG2) is important in this process. Another factor central to this process is interferon regulatory factor 6 (IRF6). The gene IRF6, which has a causal association with van der Woude syndrome, is also linked strongly to the isolated form of clefting. This finding has been replicated in many different populations and ethnic groups. Variants of genes linked to syndromic forms of cleft lip with or without cleft palate that have a mendelian mode of inheritance can also produce phenocopies of non-syndromic clefts. This observation suggests that a strategy of choosing variants of genes associated with syndromic forms of cleft lip with or without cleft palate as candidates for investigations into the cause of non-syndromic clefts could be productive. Other examples of mendelian-inherited syndromes and related genes that, if mutated, could result in or modify the expression of cleft lip with or without cleft palate include Kallmann syndrome (FGFR1), ectrodactyly, ectodermal dysplasia, and clefting syndrome (TP63), X-linked clefting and ankyloglossia (TBX22), Gorlin syndrome (PTCH1), and Margarita Island ectodermal dysplasia (PVRL1[heterozygous]). Although discovery of the genetic cause of van der Woude or popliteal pterygium syndromes will have no immediate therapeutic benefit, advantages for diagnosis are instant, and this knowledge will be potentially useful in genetic counseling. If one gene mutation, which can be identified by prenatal diagnosis, causes cleft lip, cleft lip and palate, or cleft palate alone in a proportion of people, identification of individuals at high risk for having children with the same defect will be possible.

2018 Which of the following is the protein encoded by the gene mutated in van der Woude syndrome and popliteal pterygium syndrome? A) Fibroblast growth factor 10 (FGF10) B) Fibroblast growth factor receptor 2b (FGFR2b) C) Interferon regulatory factor 6 (IRF6) D) Membrane-bound signaling molecule jagged 2 (JAG2) E) Sonic hedgehog (SHH)

The correct response is Option D. The posterior pharyngeal flap for velopharyngeal insufficiency (VPI) is generally elevated with a superior base to keep the flap high (at the junction of the naso- and oropharynx) rather than low in the oropharynx. The mucosa and superior constrictor muscle and buccopharyngeal fascia (visceral pretracheal fascia) are elevated, exposing the loose areolar tissue of the retropharyngeal space anterior to the prevertebral fascia. The buccopharyngeal fascia is thin and closely invests the constrictor muscles. Some surgeons elevate the flap superficial to the buccopharyngeal fascia. Some fibers of the middle constrictor may be included in longer pharyngeal flaps for large gaps, but the inferior constrictor is too caudad to be included. The palatopharyngeus muscle is included with sphincter pharyngoplasty, not pharyngeal flaps. The prevertebral fascia is thick and is attached to the buccopharyngeal fascia by loose areolar tissue. This is an avascular plane that is a potential space called the retropharyngeal space. The prevertebral fascia is the deepest layer of cervical fascia and surrounds the cervical column and associated musculature. The investing layer of the deep cervical fascia is the most superficial layer and contains the platysma. The pretracheal layer or buccopharyngeal layer invests the constrictors and esophagus musculature.

2018 Which of the following layers is included with a posterior pharyngeal flap? A) Investing layer of the deep cervical fascia B) Palatopharyngeus muscle C) Prevertebral fascia D) Superior pharyngeal constrictor muscle

The correct response is Option B. Although all of the facial muscles listed are innervated by branches of the facial nerve, only the mentalis muscle is innervated by the marginal mandibular branch. The orbicularis oris, buccinator, and risorius muscles are all innervated by the buccal branches of the facial nerve. The platysma muscle is innervated by the cervical branch of the facial nerve

2018 Which of the following muscles is innervated by the marginal mandibular branch of the facial nerve? A) Buccinator B) Mentalis C) Orbicularis oris D) Platysma E) Risorius

The correct response is Option D. The facial nerve (VII) arises from the brainstem and travels through the internal auditory canal where it gives rise to the greater superficial petrosal nerve from the sensory nerve bodies of the geniculate ganglion. The greater superficial petrosal nerve runs through the pterygoid canal and synapses at the pterygopalatine ganglion. Postsynaptic fibers innervate the lacrimal gland and control tear production. From the geniculate ganglion, the facial nerve runs through the tympanic cavity and then downward within the mastoid portion of the temporal bone, where it gives rise to fibers that innervate the stapedius muscle as well as form the chorda tympani nerve, which supplies taste fibers to the anterior two-thirds of the tongue as well as sending fibers to the submandibular ganglion that ultimately control saliva production from the sublingual and submandibular glands. Upon emerging from the stylomastoid foramen, the facial nerve gives rise to the posterior auricular nerve, which controls muscles that move the pinna, as well as motor nerve branches that innervate the posterior belly of the digastric muscle and stylohyoid muscle. The facial nerve then passes through the parotid gland and divides into multiple branches that control the muscles of facial expression. Understanding the anatomy of the facial nerve can often help locate the level of facial nerve lesions or injury. Loss of stapedius muscle function results in greater movement of the stapes bone in response to loud noises, resulting in hyperacusis. Paralysis of the stapedius muscle implies that the facial nerve is injured distal to the geniculate ganglion, but proximal to where it emerges from the stylomastoid foramen and becomes extracranial. The mylohyoid and tensor tympani muscles are innervated by the trigeminal (V) nerve. The tensor, cricothyroid, and levator veli palatini muscles are innervated by the vagus (X) nerve.

2018 Which of the following muscles is supplied by the facial nerve? A) Cricothyroid B) Levator veli palatini C) Mylohyoid D) Stapedius E) Tensor tympani

The correct response is Option E. Concerning the condyle itself, its blood supply is mostly derived from three sources. A branch of the inferior alveolar artery courses upward through the neck of the condylar process, where it anastomoses liberally with vessels from the attached musculature. Another major contributor to the condyle and its articular surface derives from the temporomandibular joint (TMJ) capsule, with its lush vascular plexus. A large supply of blood also comes from branches of the lateral pterygoid muscle through its attachment at the pterygoid fovea. Of these three sources, the medullary blood supply from a branch of the inferior alveolar artery was found to be the most important source in monkeys and, presumably, in man. Fracture of the subcondylar or neck region of the condylar process could therefore disrupt the main vascular supply to the condyle. There is another ramification of the loss of medullary blood supply from fracture of the condylar process. Surgical access to the condylar process in order to perform open reduction and internal fixation requires exposure and dissection of some of the soft tissues from the condylar process to permit manipulation and attachment of fixation devices. Therefore, surgery further diminishes the blood supply to a segment of bone that has already been severely compromised. If maintenance of blood supply to the condyle is important, the best choice is a surgical approach that can minimize the amount of soft-tissue stripping from the fractured condylar process and maintain, as much as possible, the attachment of the TMJ capsule and the lateral pterygoid muscle. Thus, if the preauricular approach is chosen, one should not enter the capsule of the joint as one might for an intra-articular surgery. Doing so can disrupt the already compromised blood supply to the condyle.

2018 Which of the following provides the most significant blood supply to the mandibular condylar head? A) Articular disk B) Capsule of the temporomandibular joint C) Deep branch of the superficial temporal artery D) Facial artery E) Medullary branch of the inferior alveolar artery

The correct response is Option E. More than 90% of patients have chronic effusion of middle ear prior to repair. Therefore, all patients with cleft palate should be screened for hearing loss and for fluid in the middle ear due to eustachian tube dysfunction. There is emerging evidence and controversy with regard to treatment of the tensor veli tendon and the hamulus. However, this controversy and the tensor tenopexy, tensor transection, or fracture of the hamulus and their effects on the eustachian tube are beyond the scope of the question. What is clear is that the cleft patient population is at risk for complications related to fluid in the middle ear. Unfortunately, if this condition is unrecognized, elements of preventable hearing loss will occur.

2018 Which of the following represents the percentage of patients with isolated cleft palate who are most likely to have middle ear effusion? A) 10% B) 30% C) 50% D) 70% E) 90%

The correct response is Option A. All phases of the Epstein-Barr virus life cycle are associated with human disease. In immunocompromised individuals, infected cells increase in number, and eventually B-cell growth control pathways are activated, inducing transformation and leading to malignancies such as nasopharyngeal carcinoma, Burkitt lymphoma, post-transplant lymphomas, and gastric carcinomas. Human papillomavirus (HPV) is increasingly recognized as a pathogenic risk factor for oropharyngeal cancer development. Accumulating molecular and epidemiological data now show that high-risk types of HPV are responsible for a subset of oropharyngeal cancer. Oral verrucous and squamous cell carcinomas have been reported in patients infected with hepatitis C virus, and the infection has been found to be more prevalent in patients with oral lichen planus. Infection with HIV is not known to be directly pathogenic in malignant transformation, but rather it increases the susceptibility to opportunistic infections and viral-promoted cancers. Human herpesvirus 8, also known as Kaposi sarcoma-associated herpesvirus, has been found in nearly all tumors in patients with Kaposi sarcoma.

2018 Which of the following viruses is implicated in the pathogenesis of nasopharyngeal carcinoma? A) Epstein-Barr virus B) Hepatitis C virus C) Human herpesvirus 8 D) Human immunodeficiency virus E) Human papillomavirus

The correct response is Option E. In the clinical scenario described above, the recommended procedure would be a superiorly based pharyngeal flap. Certainly all of the complications listed above can occur with a pharyngeal flap, but the majority of patients will snore for several weeks following surgery as the swelling is resolving. A flap that is too wide may result in obstructive sleep apnea in up to 38% of patients, but this too usually improves with time. The surgical failure rate ranges from 2 to 22%. Hyponasality and infection are less common complications but have been described in the literature as well.

2018 - not scored A 6-year-old boy is brought to the clinic for evaluation of speech problems. His parents report that his teacher and peers have difficulty understanding him at school. He has no history of oral surgery. Speech testing shows hypernasality. Nasopharyngoscopy confirms the diagnosis and shows good lateral wall motion, involution of the tonsils and adenoids, and no overt submucous cleft palate. Which of the following is the most common complication of the procedure recommended to correct the patient's speech? A) Failure to correct the hypernasality B) Hyponasality C) Infection D) Obstructive sleep apnea E) Snoring

The correct response is Option C. This is a growing child with a large calvarial defect in a problematic location. Therefore, the defect needs to be addressed sooner rather than later, due to the potential risk for trauma and the child's desire to be active. The gold standard for reconstruction of defects of this size and location is split calvarial bone grafting. Rib grafting would work, but would necessitate another donor site and contouring issues. A custom implant or methylmethacrylate would fix the problem in the short term, but would not grow with the child, necessitating additional surgery in the future.

2015 A 10-year-old boy with a history of fronto-orbital advancement for craniosynostosis as an infant is brought to the office for evaluation. He would like to play football, but his parents are concerned about a persistent calvarial defect. Physical examination shows a 4 × 4-cm area of the right forehead that has no bony coverage; dural pulsations are observed. Which of the following is the most appropriate management? A ) Fabrication of a custom implant B ) Reconstruction with methylmethacrylate C ) Split calvarial bone grafting D ) Split rib grafting E ) Observation and delayed treatment until after completion of growth

The correct response is Option A. Choanal atresia is a unilateral or bilateral anatomic abnormality of the posterior nasal passages and choanae, which prevents nasal gas exchange in newborns. The classic presentation of bilateral choanal atresia is cyanosis that improves with crying. Nasal airway obstruction can also become apparent when attempting to breast-feed the baby. On clinical examination, there would be no fogging of a mirror when held under the nares. The remaining abnormalities are other causes of respiratory obstruction in the pediatric patient.

2015 A 12-hour-old male newborn has cyanosis that improves with crying. Which of the following is the most likely diagnosis? A ) Choanal atresia B ) Laryngomalacia C ) Macroglossia D ) Micrognathia E ) Subglottic stenosis

The correct response is Option E. Progressive hemifacial atrophy, also known as Parry-Romberg syndrome, is a rare disorder characterized by a self-limiting, slow, progressive, unilateral facial atrophy affecting the skin, subcutaneous tissue, muscle, and osteocartilaginous adjacent structures. It involves one or more trigeminal dermatomes with symptoms usually appearing in the first or second decade of life. The progression of atrophy usually lasts a variable 2 to 10 years. The etiology and pathogenesis of this disease are uncertain. The severity of soft-tissue involvement appears to be independent of age of onset, facial distribution, or extent of the disease process; however, age of onset has been shown to correlate with the degree of bony hypoplasia. Onset of the disease at an earlier age during skeletal immaturity has a higher likelihood of skeletal involvement. Clinical features of hemifacial atrophy include enophthalmos, cheek depression, deviation of the mouth and nose towards the affected side, ipsilateral show of teeth, and tongue atrophy. In addition, extracutaneous involvement has been described, including hemiatrophy of the contralateral or ipsilateral arm, trunk or leg, as well as dental, ocular, and neurologic abnormalities such as enophthalmos, headaches, facial pain, and seizures. An age correlation with these features has not been demonstrated. Facial paresthesia has been described but motor function remains intact.

2015 A 12-year-old girl is evaluated for left-sided progressive hemifacial atrophy. Onset of symptoms started at 10 years of age. She has a history of migraines and experienced a seizure 1 year ago. Physical examination shows left-sided facial atrophy including cheek depression, enophthalmos, and dry skin with hyperpigmentation and alopecia. Maxillofacial CT scan shows associated skeletal changes. Which of the following clinical features is most likely associated with the early onset of the disease process? A ) Enophthalmos B ) Facial paralysis C ) Hyperpigmentation D ) Seizure disorder E ) Skeletal atrophy

The correct response is Option B. This CT scan shows craniosynostosis of multiple sutures including the coronal, lambdoid, and a portion of the sagittal suture, which is characteristic of a Kleeblattschädel deformity. Brachycephaly is characterized by bicoronal craniosynostosis alone and is most commonly seen in syndromic craniosynostosis. Scaphocephaly is isolated involvement of the sagittal suture and is the most common type of craniosynostosis. Metopic synostosis is the cause of trigoncephaly and this suture is open in the CT scan. Lambdoid, which produces posterior plagiocephaly synostosis, is seen in the CT scan but not in isolation and is the least common of the single suture synostoses.

2015 A 2-day-old male newborn is evaluated for the skull findings shown in the CT scan. Which of the following best describes the anomaly in this patient? A ) Brachycephaly B ) Kleeblattschädel deformity C ) Posterior plagiocephaly D ) Scaphocephaly E ) Trigonocephaly

The correct response is Option B. Total lower lip reconstruction is very challenging. A prosthetic lower lip may have acceptable static appearance but does not afford competency or dynamic function. An anterolateral thigh flap would be bulky and adynamic. Innervated regional advancement flaps are not indicated for total lip loss and would lead to microstomia. A radial forearm flap with a tendon graft can achieve an acceptable appearance and competence. The radial forearm flap is unable to achieve spontaneous lower lip movement that is as symmetrical as that of the functional gracilis muscle flap. If a split-thickness skin graft from the scalp is used, the color match can be superior to radial forearm flap as well. The functional gracilis flap is most likely to achieve this patient's goals.

2015 A 20-year-old man is evaluated for reconstruction of the lip after sustaining a traumatic blast injury to the face with complete loss of the lower lip. The patient wishes to have a symmetric, dynamic, and competent lower lip. Which of the following treatment plans is most likely to achieve the patient's desired outcome? A ) Innervated anterolateral thigh flap B ) Innervated gracilis muscle C ) Innervated regional advancement flaps (Karapandzic) D ) Prosthetic lower lip appliance E ) Radial forearm flap with tendon graft

The correct response is Option B. The patient described has bilateral masseter hypertrophy. Treatment options for this condition include muscle relaxants, injection of botulinum toxin type A, or resection of the internal layer of the masseter muscle. Superficial parotidectomy is indicated for benign and malignant tumors of the parotid gland. Resection of the submandibular gland is indicated for recurrent sialadenitis (infection) or obstructive sialodocholithiasis (salivary stones), as well as for benign tumors such as pleomorphic adenomas. Marginal mandibulectomy may be indicated for certain benign and malignant tumors of the intraoral cavity. Suction-assisted lipectomy will not treat masseter hypertrophy.

2015 A 20-year-old woman with a history of bruxism is evaluated because of a 3-year history of gradual widening of the lower third of the face. Physical examination shows rectangular appearance of the face; occlusion shows no abnormalities. Anteroposterior x-ray study discloses bone spurs at both angles of the mandible. Which of the following is the most appropriate next step in management? A ) Excision of the submandibular gland B ) Injection of botulinum toxin type A to the masseter muscle C ) Marginal mandibulectomy D ) Suction-assisted lipectomy of the cheek E ) Superficial parotidectomy

The correct response is Option C. If parotid duct injury is not repaired immediately, saliva can leak into the surrounding soft tissues. This leakage most commonly increases the risk for sialocele (pseudocapsule), followed by salivary fistula formation. Wound infection, parotid gland atrophy, and xerostomia (dry mouth) are uncommon. Studies have shown that correction of the more common complications may require surgical or medical treatments such as use of anti-sialogogues, radiation therapy, parasympathetic denervation (tympanic denervation), cauterization of the fistulous tract, reconstruction of the duct, or superficial or total parotidectomy.

2015 A 22-year-old man is brought to the emergency department after sustaining a stab wound to the face. The patient is hemodynamically stable, and physical examination shows a laceration that extends from the tragus of the right ear to the right oral commissure. Which of the following is the most likely primary complication of saliva extravasating into the wound because of parotid duct injury? A ) Parotid gland atrophy B ) Salivary fistula C ) Sialocele D ) Wound infection E ) Xerostomia

The correct response is Option E. According to the Children's Hospital of Boston's experience with arteriovenous malformations, the Schobinger stage of the lesion is most important when considering the individual's recurrence risk for these lesions. In their experience, lesion recurrence was more associated with advanced Schobinger staging than with quiescent lesions, whether these were treated with embolization alone or embolization followed by resection. Enlargement of the malformation describes a Schobinger Stage I lesion, which has the lowest risk for recurrence in the Boston Children's series. The presence of these lesions, while concerning to parents, typically has a recurrence risk of 80% with embolization alone and 21% with embolization followed by resection. Patient sex, age, and lesion location did not correlate with lesion recurrence in the Boston Children's series. Quiescence of the malformation represents a Schobinger Stage III lesion, which is the highest Schobinger stage description among the options listed. This stage lesion has a recurrence risk of 99% with embolization alone and 81% with embolization followed by resection.

2015 A 22-year-old woman comes to the office because of oral bleeding and a 20-year history of a radiologically defined arteriovenous malformation. Embolization and resection of the tumor are planned. Which of the following factors is most likely to be associated with an increased risk for recurrence? A ) Enlargement of the malformation B ) Patient age C ) Patient gender D ) Quiescence of the malformation E ) Ulceration

The correct response is Option D. This patient has a skeletal class II deformity with a retrognathic mandible and normal maxillary projection. The SNA angle of 82 degrees (N 80-84) indicates a normally positioned maxilla relative to the cranial base, while the SNB angle of 75 degrees (N 78-80) indicates a retrognathic mandible relative to the cranial base. The ANB angle confirms the class II deformity (>4 degrees). A mandibular sagittal split osteotomy with advancement will correct this deformity. Maxillary impaction is used to treat vertical maxillary excess. LeFort I maxillary advancement will worsen this patient's deformity.

2015 A 22-year-old woman comes to the office because she is unhappy with the appearance of the lower third of her face. On examination, she has a class II occlusion. Lateral cephalometric evaluation shows an SNA angle of 82 degrees (N 80-84), an SNB angle of 75 degrees (N 78-80), and an ANB angle of 7 degrees. Cranial base anatomy shows no abnormalities. Which of the following orthognathic procedures is most appropriate in this patient? A ) LeFort I maxillary advancement B ) LeFort I maxillary advancement with mandibular setback C ) Maxillary impaction D ) Sagittal split mandibular osteotomy with advancement E ) Sliding genioplasty

The correct response is Option C. It is most likely that this patient has progressive, but now stable, hemifacial atrophy, or Parry-Romberg syndrome. The course is characterized by progressive unilateral atrophy of the soft tissues of the face. The underlying skeleton may be affected in severe forms of the disease. Surgery is indicated when the changes stabilize. The recommended treatment is free tissue transfer, preferably a microvascular parascapular flap, followed by a secondary autologous fat grafting for refinement. Augmentation of the maxilla is not indicated in the absence of bony atrophy. Staged cross-facial nerve grafting followed by microvascular gracilis transfer is indicated for absence of facial nerve function. A pedicled temporalis muscle flap would likely not provide sufficient bulk where needed and potentially leave a hollow at the donor site. Alloplastic augmentation of the soft tissues with hyaluronic acid is only approved for volume loss due to human immunodeficiency virus infection.

2015 A 23-year-old man comes to the office because of a progressive 15-year history of severe unilateral volume loss in the face. The patient says he is unhappy with the changes in his facial appearance but has not noted any changes recently. On examination, volume loss is significant and appears to be limited to the soft tissue. No bony deficit is noted. Which of the following is the most appropriate management? A ) Alloplastic bony augmentation of the maxilla B ) Autologous soft-tissue augmentation with a cross-facial nerve graft followed by microvascular gracilis muscle transfer C ) Autologous soft-tissue augmentation with a microvascular parascapular flap D ) Autologous soft-tissue augmentation with a pedicled temporalis muscle flap E ) Hyaluronic acid augmentation of the soft tissues

The correct response is Option A. Although it is most commonly seen after parotidectomy, Frey syndrome has also been encountered after condylar fracture of the mandible and treatment. The syndrome is thought to result from damage to auriculotemporal parasympathetic nerve fibers with subsequent aberrant regeneration and innervation of sympathetic fibers to the sweat glands. The facial, inferior alveolar, greater auricular, and lingual nerves are not thought to be the underlying cause of Frey syndrome.

2015 A 24-year-old man with a history of left facial trauma and condylar fracture of the mandible is evaluated because of redness and perspiration of the left cheek and ear after ingesting certain foods. Aberrant regeneration of which of the following nerves is the most likely cause of this patient's symptoms? A ) Auriculotemporal B ) Facial C ) Great auricular D ) Inferior alveolar E ) Lingual

The correct response is Option E. 21365 is the appropriate code for the zygomaticomaxillary complex (ZMC) fracture repair since the repair requires multiple incisions and the fracture crosses the infraorbital foramen. The orbital floor repair is not considered "bundled" with the repair of the ZMC fracture and should be billed separately as a distinct procedure. 21360 is not the appropriate code for this complicated ZMC fracture. 21390 is correctly added to this code, however. 21360 is the appropriate code for a simple repair of a ZMC fracture and does not include reduction and repair of the concurrent orbital floor fracture. 21390 is not the appropriate code without 21365. 21365 is the appropriate code for the ZMC fracture repair, however, this option omits the code for the reduction and repair of the orbital floor fracture.

2015 A 25-year-old man is evaluated after sustaining a left zygomaticomaxillary complex (ZMC) fracture. On exploration of the fracture through the upper buccal sulcus, left brow, and left trans-conjunctival incisions, the fracture crosses the infraorbital foramen, and the left orbital floor fracture is significantly depressed and displaced. Open reduction and internal fixation (ORIF) of the orbital floor and ZMC fractures is performed. Which of the following is the most appropriate CPT code for this patient? CPT Code Description 21360 ORIF malar fracture only 21365 ORIF of "complicated" malar fracture only 21390 orbital blowout fracture repair, periorbital approach with alloplastic or other implant A ) 21360 B ) 21365 C ) 21390 D ) 21360 and 21390 E ) 21365 and 21390

The correct response is Option D. The gracilis muscle has been widely used for facial reanimation because of its many advantages. The muscle is located in the inner thigh, which makes harvest easy in a supine position and keeps the donor site well hidden. It has a single constant vascular pedicle of adequate length for transfer. Although it is able to generate sufficient force for animation, it has only one direction of pull and is thus best suited for restoring only one component of smiling. There is no overlying tendon, and there is a single dominant nerve that is not able to reach the contralateral side of the face.

2015 A 25-year-old woman is evaluated for unilateral facial paralysis after she sustained an episode of Bell palsy that did not respond to oral administration of corticosteroids. Methods for functional reconstruction using a gracilis muscle transfer are discussed. Which of the following is an advantage of this muscle transfer for this patient? A ) Ability of the nerve to reach the contralateral side B ) Ability to incorporate overlying tendon C ) Multiple directions of pull D ) Reliable vascular pedicle E ) Two dominant nerves

The correct response is Option A. To obtain maximum compression at the fracture line using the spherical gliding principle, the most appropriate next step is to drill for a second screw eccentrically, away from the line of fracture, through a plate hole located across the fracture line. Concentric or neutral drilling occurs in the center of a plate hole, while eccentric drilling occurs in the periphery (corner) of the plate hole (either away or closer to the line of fracture). The figure above illustrates the spherical gliding principle in a dynamic compression plate. The special geometry of the plate hole—together with eccentric, away from the fracture line, placement of the screw that has a spherically shaped head—allows interfragmentary compression in an axial direction when the screw is driven fully into the plate hole. For appropriate fracture compression to occur, the head of the screw that was placed first must be well seated into the plate hole, stabilizing the plate against the underlying bony segment. Drilling concentrically (neutrally) through a plate hole located across the fracture line would cause no axial movement of the underlying bone fragments upon tightening of the screw against the plate, generating no further compression at the fracture line. Drilling eccentrically, closer to the line of fracture, through a plate hole located across the fracture line would increase the fracture gap upon tightening of the screw against the plate. Placement of a second screw in a plate hole located on the same side of the first screw (in relation to the line of fracture) would have no effect over the fracture line. Also, after eccentric (away from the line of fracture) placement of one screw on each side of the fracture, the remaining screws should be placed concentrically.

2015 A 32-year-old man undergoes open reduction and internal fixation of a fracture in the left body of the mandible. A six-hole dynamic compression plate is chosen for the inferior mandibular border. The first screw is placed bicortically through a plate hole immediately adjacent to the line of fracture. To obtain maximum compression at the fracture line using the spherical gliding principle, the most appropriate next step is to drill for a second screw at which of the following locations (A-E)? A ) B ) C ) D ) E )

The correct response is Option B. The patient in the photograph has metopic craniosynostosis. The primary clinical features of trigonocephaly are a palpable metopic ridge, bossing with thickened bone at the glabella, bifrontal narrowing, recession of the superior orbital rims, temporal narrowing, and hypotelorism. Trigonocephaly is also known as a keel-shaped deformity. The metopic suture is the first cranial suture to fuse and this usually occurs at approximately 8 months of age. The other cranial sutures generally fuse in adulthood. Treatment depends on the age of presentation and extent of deformity. Options for management are varied and range from no intervention for minimal deformity, burring of the metopic ridge only, endoscopic synostectomy with helmet therapy, and bilateral fronto-orbital advancement.

2015 A 4-month-old male infant is brought to the office for evaluation of an abnormal head shape that has been present since birth. The patient is developmentally appropriate and has no other abnormalites. A photograph is shown. At which of the following ages does the involved suture normally fuse? A ) 2 weeks B ) 8 months C ) 3 years D ) 10 years E ) 22 years

The correct response is Option D. While any of the methods listed can potentially improve speech in the patient described, the lack of significant velar motion coupled with poor lateral pharyngeal wall motion makes pharyngeal flap the most predictable alternative of those listed. Intravelar veloplasty effectively restores and repositions the levator sling, but will not overcome the poor lateral wall motion. Furthermore, it is probable (although not certain) that the muscles were properly positioned during the initial operation. Fat augmentation and Furlow palatoplasty can decrease the coronal gap, but will do little to address the deficient lateral pharyngeal wall mobility. Sphincter palatoplasty has yielded good results in patients with this condition, but poor muscle tone and coronal closure make this method less likely to produce a competent pharyngeal sphincter than a properly designed pharyngeal flap.

2015 A 4-year-old girl with velocardiofacial syndrome is evaluated for hypernasal speech. She underwent protracted speech therapy after repair of an isolated cleft of the soft palate at age 9 months. Physical examination shows a well-healed palate and trace elevation of the soft palate with phonation. Videofluoroscopy shows poor motion of the velum; adequate lateral pharyngeal wall motion is noted. To improve this patient's speech, which of the following is the most appropriate management of her velopharyngeal insufficiency? A ) Fat augmentation of the posterior pharyngeal wall B ) Furlow palatoplasty C ) Intravelar veloplasty D ) Reconstruction with a superiorly based pharyngeal flap E ) Sphincter pharyngoplasty

The correct response is Option E. Oral cavity cancers are staged based on the following criteria set forth by the American Joint Committee on Cancer: Primary tumor staging for oral cavity cancers (T) TX Primary tumor cannot be assessed T0 No evidence of primary tumor Tis Carcinoma in situ T1 Tumor = 2 cm in greatest dimension T2 Tumor >2 cm but not more than 4 cm in greatest dimension T3 Tumor >4 cm in greatest dimension T4a Moderately advanced, local disease · Lip - Tumor invades through cortical bone, inferior alveolar nerve, floor of mouth, or skin of face · Oral cavity - Tumor invades adjacent structures (e.g., through cortical bone, into deep extrinsic muscle of the tongue, maxillary sinus, or skin of face) T4b Very advanced, local disease · Tumor invades masticator space, pterygoid plates, or skull base and/or encases internal carotid artery The cancer described is a stage T4aN0M0 cancer of the floor of mouth. The T-stage is 4a based on cortical mandibular invasion demonstrated by radiographic imaging. Concern for mandibular invasion should be raised whenever a tumor abuts or is fixed to the mandible. Mandibulectomy is indicated. Cortical invasion of the mandible is an indication for segmental mandibulectomy, in which the full thickness of the involved mandible and grossly disease-free margin are removed by osteotomies. Marginal mandibulectomy involves removal of the alveolar ridge and varying amounts of the inner or lingual table of the mandible depending on the location of the tumor. Marginal mandibulectomy is performed when cancers abut the mandible or invade the periosteum, but do not grossly invade the cortex of the bone. Although clinically and radiographically the neck does not have nodal metastases (stage N0), surgical treatment is usually performed due to the risk for occult nodal metastases (20% or more in some studies). Such dissection also facilitates reconstruction by exposing potential recipient blood vessels for microvascular free tissue transfer.

2015 A 45-year-old woman is evaluated for a 2.5-cm, biopsy-proven squamous cell carcinoma of the left floor of the mouth. Imaging studies show cortical mandibular invasion, but no enlarged cervical lymph nodes or distant metastatic disease. A tracheostomy is performed. Immediate reconstruction is planned. Which of the following is the most appropriate surgical treatment for this cancer? A ) Wide local excision alone B ) Wide local excision and marginal mandibulectomy C ) Wide local excision and neck dissection D ) Wide local excision, marginal mandibulectomy, and neck dissection E) Wide local excision, segmental mandibulectomy, and neck dissection

The correct response is Option C. In the case of Möbius syndrome and most pediatric patients with facial paralysis in which the facial muscles are no longer available, a microneurovascular transfer with a muscle flap is the preferred treatment. The gracilis is the most advocated muscle used for this purpose. Pediatric patients are generally very motivated and do well with microneurovascular transfer with a muscle flap. In Möbius syndrome, the sixth and seventh cranial nerves are commonly involved. Other cranial nerves may be involved as well. In addition to the facial muscles not being available for reconstruction, the cranial nerves are also not available in patients with Möbius syndrome. Hence, transfer to the ipsilateral or contralateral facial nerve is not a viable option for reconstruction in patients with Möbius syndrome, as the facial nerve does not function. Use of the hypoglossal nerve in patients with Möbius syndrome is relatively contraindicated due to worsening of tongue function. The "babysitter" technique utilizes the hypoglossal nerve as a donor nerve. The masseteric nerve is preferred over the spinal accessory and phrenic nerves as it provides better motor strength and lower morbidity in the muscle transfer.

2015 A 5-year-old girl with Möbius syndrome is brought to the office for evaluation of facial paralysis. Physical examination shows masklike facies, inability to animate the face bilaterally, and generalized hypoplasia of the tongue. Which of the following is the most appropriate donor nerve to restore facial function in this patient? A ) Hypoglossal B ) Ipsilateral facial C ) Masseteric D ) Spinal accessory E ) Phrenic

The correct response is Option A. The issue of religious beliefs and medical management is a difficult ethical scenario that can arise in the care of pediatric patients. It is generally accepted in many countries with the support of international law that a minor should not be put at risk because of the religious beliefs of his/her parents. In this case, a blood transfusion is clearly needed but is not a medical treatment accepted by Jehovah's Witnesses. In the United States, the American Academy of Pediatrics recommends that in cases of "an imminent threat to a child's life," physicians in some cases may "intervene over parental objections." In general, a contingency of bloodless surgery programs for pediatric patients is the reservation of the legal right to give blood if an emergent need arises, despite the lack of parental consent. It is generally accepted that blood products may be given to minors if it is in the best interest of the patient despite parental religious beliefs or other objection. This course of action has been supported a number of times in the court of law.

2015 A 5-year-old girl, who is a Jehovah's Witness, is scheduled to undergo extensive cranioplasty. Bloodless surgery has been scheduled at an institution offering a blood conservation program. During the procedure, the patient becomes hemodynamically unstable from excessive, ongoing intraoperative bleeding and is found to have a hemoglobin level of 6.0 g/dL. The patient's parents refuse to consent to a blood transfusion. Which of the following is the most appropriate next step in management? A ) Administration of blood products B ) Administration of erythropoietin C ) Initiation of vasopressors D ) Resuscitation with albumin E ) Resuscitation with hypertonic saline

The correct response is Option A. Apraclonidine is an alpha-2 receptor agonist and is believed to increase muscle tone of the sympathetically innervated Müller muscle located in the upper eyelid. Beta-2 adrenergic stimulation of the levator palpebrae superioris is not appropriate because apraclonidine does not act as the beta-2 receptor, and it is not believed to stimulate contraction of the levator palpebrae superioris. Muscarinic parasympathetic inhibition of the levator palpebrae superioris is not appropriate because apraclonidine does not act as the muscarinic parasympathetic receptor. Nicotinic parasympathetic inhibition of the Müller muscle of the upper eyelid is not appropriate because apraclonidine does not act as the nicotinic parasympathetic receptor.

2015 A 50-year-old woman comes to the office 2 weeks after receiving botulinum toxin type A injections for forehead rhytids. She is pleased with the results but has developed unilateral eyelid ptosis. Apraclonidine 0.5% eyedrops are prescribed to improve upper eyelid function. Which of the following best describes the mechanism for improvement of upper eyelid ptosis in this patient? A ) Alpha-2 adrenergic stimulation of the Müller muscle of the upper eyelid B ) Beta-2 adrenergic stimulation of the levator palpebrae superioris C ) Muscarinic parasympathetic inhibition of the levator palpebrae superioris D ) Nicotinic parasympathetic inhibition of the Müller muscle of the upper eyelid

The correct response is Option A. Initial treatment of acute suppurative sialadenitis begins with aggressive medical management. This includes prompt fluid and electrolyte replacement, oral hygiene, reversal of salivary stasis, and antimicrobial therapy. Stimulation of salivary flow is done by use of sialogogues such as lemon drops. Warm soaks and massage promote secretion and drainage of the gland. Oral cultures are typically contaminated by oral flora and therefore do not direct antibiotic treatment. Needle aspiration is more accurate in isolating the cause of suppurative parotitis. Incision and drainage is reserved for cases resistant to medical management. Surgical removal of the gland is not recommended in the case of an actively infected gland.

2015 A 54-year-old man comes to the office because of swelling of the left side of the face 3 days after cholecystectomy. Physical examination shows erythema and purulent drainage from the parotid duct. Which of the following is the most appropriate initial management? A ) Antibiotic therapy and sialogogues B ) Aspiration of the mass C ) Incision and drainage of the mass D ) Oral cultures and oral cavity antibiotic irrigation E ) Superficial parotidectomy

The correct response is Option B. Rhinocerebral mucormycosis is a rare opportunistic infection of the sinuses, nasal passages, oral cavity, and brain caused by saprophytic fungi. The infection can rapidly result in death. Rhinocerebral mucormycosis commonly affects individuals with diabetes and those in immunocompromised states. The diagnosis of mucormycosis is established by obtaining a biopsy specimen of the involved tissue, and frozen tissue samples should be immediately evaluated for signs of infection. Microscopic characterization of non-septate hyphae, rhizoids, columellae, sporangia, and sporangiospores helps to define genus and species within the order Mucorales. Optimal therapy requires a multidisciplinary approach that relies on prompt institution of appropriate antifungal therapy with amphotericin B (AmB), reversal of underlying predisposing conditions, and, where possible, surgical debridement of devitalized tissue. Surgery should be considered early, and if possible, emergently with the goal of removing all necrotic tissue. Repeated debridements are frequently necessary and the extent of surgery should ideally be guided by evaluation of frozen tissue sections examined histologically. In the scenario provided, orbital involvement will most likely require sacrifice of the eye.

2015 A 55-year-old man with a history of smoking and poorly controlled diabetes mellitus comes to the emergency department because of a 12-hour history of unilateral proptosis, impaired vision, and severe headaches. Intranasal examination shows dried crusting and black discoloration of the lateral nasal wall and turbinates. Radiology shows a 4-cm mass within the maxillary sinus that extends into the orbit. Biopsy of the mass shows nonseptate hyphae. Which of the following is the most appropriate next step in management? A ) Craniofacial resection and free flap B ) Emergent debridement of the sinuses and orbital exenteration C ) Intravenous administration of an antibiotic D ) Outpatient oral antifungal medications E ) Referral to medical oncology

The correct response is Option D. Oral cavity cancers are staged based on the following criteria set forth by the American Joint Committee on Cancer (AJCC): Primary tumor staging for oral cavity cancers (T) TX Primary tumor cannot be assessed T0 No evidence of primary tumor Tis Carcinoma in situ T1 Tumor = 2 cm in greatest dimension T2 Tumor > 2 cm but not more than 4 cm in greatest dimension T3 Tumor > 4 cm in greatest dimension T4a Moderately advanced, local disease Lip - Tumor invades through cortical bone, inferior alveolar nerve, floor of mouth, or skin of face Oral cavity - Tumor invades adjacent structures (e.g., through cortical bone, into deep extrinsic muscle of the tongue, maxillary sinus, or skin of face) T4b Very advanced, local disease Tumor invades masticator space, pterygoid plates, or skull base and/or encases internal carotid artery The cancer described in the scenario is a stage T2N0M0 cancer of the lip. Surgical excision with margin control is the most appropriate and probably the most common treatment for this lesion, although Mohs micrographic surgery may also be an option for surgical removal. Reconstruction would be performed after attaining adequate surgical margins and can be performed utilizing local tissue flaps in most patients. 5-Fluorouracil cream can be used in the treatment of actinic keratosis and superficial basal cell cancers. It is also used in the treatment of some squamous cell cancers, but its efficacy is lower and would not be indicated for a T2 cancer. In curettage and electrodessication, small tumors are scraped off with a curette and electrocautery destroys residual tumor and controls bleeding. It is used for superficial squamous cell cancers without high-risk characteristics in noncosmetically sensitive sites and would not be appropriate for this lesion. Also, surgical margins cannot be evaluated with this technique. Photodynamic therapy involves application of a photosensitizing agent, such as 5-aminolevulinic acid (5-ALA), that is taken up by the cancer cells.The next day, the medicated areas are activated by a strong light. The treatment selectively destroys cancer cells while causing minimal damage to the surrounding tissue. It is used to treat actinic keratosis and is not approved by the Food and Drug Administration for treatment of squamous cell carcinoma, although experience suggests that it is effective for small, superficial cancers. Radiation therapy alone can be used in cosmetically and functionally important sites such as the lip or in patients who are poor candidates for surgery because of medial comorbidities. Given the good functional and cosmetic prognosis of surgical resection and reconstruction in this patient, irradiation would likely be reserved for a recurrent cancer. Combined surgery and irradiation is usually reserved for T4-stage cancers or those with nodal metastases.

2016 A 50-year-old man with a long-term history of sun exposure is evaluated because of a 2.5-cm biopsy-proven squamous cell carcinoma of the central lower lip. Physical examination shows no enlarged cervical lymph nodes. Which of the following is the most appropriate treatment for this cancer? A ) Combined surgery and radiation B ) Curettage and electrodessication C ) Photodynamic therapy D ) Surgical excision E ) Treatment with 5-fluorouracil for 3 months

The correct response is Option B. The patient has Van der Woude syndrome, an autosomal dominant condition affecting 1:35,000 to 1:100,000 persons. The pathognomonic features include lower lip pits, as seen in the photograph, and clefting of the palate, lip, or both. This syndrome is the most common single-gene cause of cleft lip and palate. IRF6 codes for a transcription factor that is involved in the early development. The mutated copy of the gene decreases the amount of active protein and results in the defects associated with this syndrome and popliteal pterygium syndrome. The remaining genes and their products have been implicated in craniofacial morphogenesis and etiopathogenesis of cleft lip/palate. Nevertheless, none are associated with lip pits or are an autosomal dominant cause of facial clefting.

2015 A 6-month-old girl is brought to the office for evaluation of an isolated cleft of the soft palate. A photograph is shown. Which of the following genes is most likely defective in this patient? A ) FGFR2 B ) IRF6 C ) MSX1 D ) SHH E ) TGF-beta

The correct response is Option C. The clinical findings are suggestive of right unilateral lambdoidal synostosis (synostotic plagiocephaly). If craniosynostosis is suspected, the child may need surgical correction of the defect. Therefore, it is incorrect to pursue reassurance and conservative therapy without confirming the presence of a prematurely fused suture. Lambdoidal synostosis is the least common premature fusion of a suture (2 to 3% of all synostosis cases). The physical findings have some similarities and distinct differences from the frequently seen positional deformational plagiocephaly. Positional plagiocephaly can have ipsilateral forehead protuberance and anterior displacement of the ipsilateral ear. Lambdoid synostosis, on the other hand, is associated with ipsilateral mastoid bossing and protuberance of the contralateral forehead. The position of the ear, once another landmark to help distinguish between positional and synostotic plagiocephaly, with lambdoid synostosis associated with posterior displacement of the ipsilateral ear, has been questioned in recent years. A literature review showed that the ipsilateral ear has been reported to be nondisplaced, anteriorly displaced, or posteriorly displaced. Therefore, the significance of the position of the ear is unclear at this time. There is no need for plain film series before head CT scan. Imaging studies are performed to confirm synostosis before surgical intervention.

2015 A 6-month-old girl is evaluated because of a "flat head." Physical examination shows right posterior flatness of the cranium with a transcranial difference of 7 mm (minimal to moderate), and mastoid bossing on the ipsilateral side. The right ear and left forehead are anteriorly displaced. The parents report that the child prefers to turn her head to the right. The patient is otherwise healthy and reaching developmental milestones. Which of the following is the most appropriate next step in management? A ) Cranial orthotic molding helmet B ) Crib positioning and physical therapy only C ) CT scan of the head D ) X-ray studies of the skull E ) Reassurance

The correct response is Option D . Anterior open bite, also known as apertognathia, is vertical separation of the maxillary and mandibular anterior teeth. It is caused by premature contact of the posterior molars, most commonly following bilateral subcondylar mandible fracture. When present, a unilateral subcondylar/condylar fracture causes an open bite on the side opposite the fracture.

2015 A 6-year-old boy is brought to the emergency department following facial trauma from falling on his bicycle handlebars. Which of the following mandible fracture locations is most commonly associated with anterior open bite? A ) Angle B ) Body C ) Coronoid D ) Subcondylar/condylar E ) Symphysis/parasymphysis

The correct response is Option B. Alveolar bone grafting should be performed during the time of transitional dentition. Specifically, it has the greatest chance for success after the incisors erupt, but before the eruption of the canine. Tooth development/eruption varies from child to child, so there is no set age for bone grafting. Alveolar cleft width will determine the amount of bone grafting, but not the timing. Severity of alveolar collapse will affect the duration of palatal expansion, but not timing. Occlusal status should not play a role in surgical decision-making for alveolar cleft grafting, as it will be addressed later with either orthodontics or jaw surgery after skeletal maturity.

2015 A 6-year-old boy with a repaired unilateral complete cleft lip and palate presents for an annual cleft team clinic visit. Initiation of palatal expansion is discussed with the child's parents. Timing for initiation of palatal expansion should be based upon which of the following? A ) Alveolar cleft width B ) Canine eruption C ) Occlusal status D ) Patient age E ) Severity of alveolar collapse

The correct response is Option E. A Cutler-Beard flap is a two-stage lid switch flap, taken from the lower lid. Some of its disadvantages include two-stage reconstruction requiring eye occlusion for a number of weeks, sacrifice of lower lid tissue (and, commonly, a subsequent ectopion), and lack of intrinsic support. Many modifications have improved on its design including those that provide support. There are many reports of other types of flaps, but for a defect of this size and the anatomic requirements, the best choice is a lid switch flap.

2015 A 60-year-old man undergoes Mohs micrographic resection of an upper eyelid squamous cell carcinoma. A photograph is shown. Which of the following reconstruction procedures will provide the most functional and aesthetically pleasing result in this patient? A ) Free nasoseptal grafting B ) Reconstruction with a forehead flap C ) Reconstruction with a free flap D ) Reconstruction with a glabellar flap E ) Reconstruction with a lid switch flap

The correct response is Option A. The facial artery generally starts as part of the lingual-facial trunk, then travels below the hypoglossal nerve before it enters into the submandibular gland and along the lateral border of the mandible. Failure to recognize this structure could cause injury and subsequent loss of motor function of the ipsilateral tongue. Ipsilateral hypoglossal (XII) nerve injury causes the tongue to move toward the side of damage, resulting in dysarthria, and problems moving solid food to the oropharynx. Vocal cord paralysis is related to a recurrent laryngeal or vagus (X) nerve injury, which could happen after superior laryngeal artery or common carotid dissection, respectively. Shoulder drop is related to accessory (XI) nerve injury, which has anatomic relation to the occipital artery. Tongue numbness is from an injury to the lingual nerve (related mostly to the laryngeal artery and submandibular duct). Lip elevation is related to a marginal mandibular (V3) nerve injury—this nerve runs with the facial artery lateral to the mandible, but not below the margin of the mandible.

2015 A 60-year-old man undergoes microvascular anastomosis. The proximal facial artery off the external carotid artery is to be dissected and used as a recipient vessel. During the procedure, a large, overlying, nerve-like structure is inadvertently transected. Which of the following is the most likely consequence? A ) Dysarthria B ) Lip elevation C ) Shoulder drop D ) Tongue numbness E ) Vocal cord paralysis

The correct response is Option E. Merkel cell tumor is an unusual and highly aggressive skin cancer. More than 50% of Merkel cell tumors occur in the head and neck region. Risk factors for Merkel cell tumors are exposure to sun and ultraviolet light, and immunosuppression. There is a recent association with Merkel cell polyomavirus. Merkel cell generally presents as a firm, painless nodule (up to 2 cm in diameter) or as a mass (>2 cm in diameter). Although classically described as red in color, it may be flesh-colored or blue. It often enlarges rapidly. The standard of management is surgical excision combined with radiation therapy. Radiation therapy decreases local recurrence rates. Node-negative patients with no distant metastasis treated with surgery and radiation have 5-year survival rates of approximately 90%. Mohs micrographic surgery and wide local excision are both accepted modalities of surgical resection. It is well known that surgery alone is insufficient to cure or control Merkel cell tumors. Consideration should be given to evaluation of the lymph nodes. Sentinel node biopsy is a common modality.

2015 A 60-year-old woman comes to the office for evaluation of a firm, violaceous, 3-cm lesion of the left cheek. An incisional biopsy shows a Merkel cell tumor. The residual margins are positive. Which of the following is the most appropriate management? A ) Chemotherapy B ) Laser ablation C ) Mohs micrographic surgery D ) Radiation therapy and chemotherapy E ) Wide local excision and radiation therapy

The correct response is Option E. Over the past few decades, the use of chemotherapy/radiation as the primary curative treatment for oropharyngeal cancer has increased. In part, this has to do with cure rates and tissue preservation, but it is also due to the rise in human papillomavirus-positive oropharyngeal cancer. Not surprisingly, there has been a marked increase in osteoradionecrosis of the mandible, in particular. The most common cause of pathologic fracture after radiation therapy in the mandible is tooth extraction, usually the third molar, and a subsequent angle injury, as in this case. With a large, multi-decade experience in the use of osseous free flaps, especially the fibular flap, these cases are now routinely managed by resection of the affected bone and immediate reconstruction. Hyperbaric oxygen, as a single modality for osteoradionecrosis, is at best controversial and would not cure a pathologic fracture. Intravenous antibiotics can treat osteomyelitis, but in a case of osteoradionecrosis and a fracture, a short course of adjuvant antibiotics (along with appropriate surgery) would be sufficient, at best. Open reduction and internal fixation of this fracture would not suffice either, because necrotic bone will not heal, even if put into juxtaposition. Although it is important to assume that any pathology in cases like these are cancerous until proven otherwise, nevertheless, this scenario as described is very common and the constellation of events plus the imaging indicates that oncology's role in this case would be limited at best.

2015 A 62-year-old woman who underwent chemotherapy/radiation protocol for oropharyngeal cancer 10 years ago has onset of severe pain after a dental extraction. Subsequent CT scan shows a pathologic fracture of the mandibular angle. Which of the following is the most appropriate management? A ) Hyperbaric oxygen therapy B ) Long-term intravenous antibiotic therapy C ) Oncology consultation D ) Open reduction and internal fixation of the mandible E ) Resection and coverage with a fibular free flap

The correct response is Option B. Gold weight prostheses are commonly used for upper eyelid reanimation in patients with facial nerve dysfunction. The weight required can be estimated preoperatively by using two-sided tape to secure various test weights to identify the prosthesis that brings the upper eyelid to within 2 to 4 mm of the lower lid and completely covers the cornea. A common mistake is to use a weight that is too heavy in an effort to completely close the upper eyelid, resulting in upper eyelid ptosis and obstruction of field of view. Revision surgery is often needed in these cases. Contralateral symmetry is rarely achieved with gold weights and is not a criterion for assessment. Similarly, the speed with which the upper eyelid is closed is usually slower than the contralateral eyelid and is not a factor in weight selection.

2015 A 65-year-old man is evaluated for right facial nerve paralysis. Upper eyelid reanimation is planned. The gold weight prosthesis is selected. Which of the following considerations is most appropriate for this patient? A ) Allowing coverage of the upper limbus at rest B ) Bringing the upper eyelid to within 2 to 4 mm of the lower eyelid C ) Enabling complete closure of the upper eyelid D ) Enabling the most rapid closure of the upper eyelid E ) Providing the best symmetry with the contralateral eye

The correct response is Option B. The circumferential defect described in this patient requires coverage with a tubularized flap that can span the length of the defect and reestablish continuity of the alimentary track. Gastric pull-up is not a good option in this case because of its high morbidity and poor perfusion in the most proximal region of the gastric flap. Coverage with the pectoralis flap or deltopectoral flap is not an appropriate option because these flaps cannot be tubularized in a single-stage reconstruction. The spit fistula would not restore alimentary tract continuity, and it should only be used if no other reconstructive options are available or if the patient is medically unstable. The anterolateral thigh flap is the best option in this case because it can be tubularized to span the defect. In most cases, the resulting reconstruction is highly effective with restoration of swallowing function in the majority of patients.

2015 A 68-year-old man with a history of laryngeal cancer treated with chemoradiation 2 years ago has a recurrence. He is scheduled for total laryngopharyngectomy with circumferential resection of the pharynx extending from the floor of the mouth to 2 cm above the manubrium. Which of the following is the most appropriate single-stage reconstruction? A ) Construction of a spit fistula B ) Coverage with an anterolateral thigh flap C ) Coverage with a deltopectoral flap D ) Coverage with a pectoralis flap E ) Use of gastric pull-up

The correct response is Option E. The primary goal of cleft palate repair is normal speech and swallowing. Velopharyngeal competence, the ability to completely close the velopharyngeal sphincter, is required for the normal production of all but the nasal consonants (in English: /m/, /n/, and /ng/). Velopharyngeal insufficiency (VPI) is defined as the inability to completely close the velopharyngeal sphincter. The primary effects of VPI are nasal air escape and hypernasality. Video fluoroscopy and nasopharyngoscopy can detect the sagittal deficiency closure pattern occurring in patients with VPI after cleft palate surgery. Speech articulation errors (i.e., distortions, substitutions, and omissions) are secondary effects of VPI. The result is decreased intelligibility of speech. The velopharyngeal port is bordered anteriorly by the velum, bilaterally by the lateral pharyngeal walls, and posteriorly by the posterior pharyngeal wall. VPI can be diagnosed by both subjective and objective means. Perceptual evaluation of speech by an experienced speech language pathologist is the standard. Multiview video fluoroscopy and nasopharyngoscopy both provide visual information (i.e., closure pattern and closure rating) that is valuable for surgical planning. However, the need to avoid radiation if centers are migrating away from fluoroscopy has caused most cleft centers to migrate to direct nasopharyngoscopy. MRI is emerging as a technology for evaluating VPI, but it would not be the first choice for diagnostic workup. Overall, examination during anesthesia in the operating room would likely not be required, but instead a complete examination and nasopharyngoscopy would likely be performed comfortably in the office. Experience with nasopharyngoscopy has In-Service Examination grown in most comprehensive cleft centers and has become an invaluable tool for surgical planning. Lateral cephalogram does not provide a dynamic evaluation of the pharynx.

2015 A 7-year-old girl with a history of repair of cleft palate is evaluated because of possible velopharyngeal insufficiency. In addition to evaluation of the patient's speech by trained speech pathologists, which of the following is the most appropriate diagnostic tool? A ) Cine MRI B ) CT scan C ) Examination during anesthesia D ) Lateral cephalogram E ) Nasopharyngoscopy

The correct response is Option A. The most appropriate coverage of the oncologic defect in this patient is with a dermal regeneration template, such as Integra, followed by skin autograft. Integra is a synthetic bilaminate neodermis composed of a collagen lattice covered with a thin silastic sheet. A single-layer version (collagen only) is also available, allowing stacking of the product for increased soft-tissue thickness. Vascularization of the collagen layer usually occurs in 3 to 4 weeks, at which point the silastic sheet is removed and a thin split-thickness skin autograft is applied. In the absence of pericranium, burring of the exposed calvarium down to healthy bleeding bone is recommended. A skin autograft applied directly to calvarial bone denuded of periosteal coverage is unlikely to "take." Interpolated scalp flaps, most likely requiring grafting of a secondary defect (donor site), would be appropriately indicated for coverage of a midparietal 8-cm defect. These are large flaps, based on the major blood vessels supplying the scalp, with an area of undermining that frequently involves the entire scalp. Unfortunately for this patient with multiple previous craniotomies, the resulting scars impose an unacceptably high risk for flap ischemia. Similarly, a scar-free, well-vascularized pericranial flap large enough to cover the described defect is unlikely to be found in this patient. The safety and success of free tissue transfer in patients depending on LVADs for hemodynamic stability is still to be determined.

2015 A 72-year-old man with advanced congestive heart failure who recently received a left ventricular assist device (LVAD) comes to the office with an ulcerated mass in the mid-parietal region. Punch biopsies reveal squamous cell carcinoma of the skin. The patient has a history of bilateral temporal and midline craniotomies for resection of symptomatic meningiomas. Wide local excision of the tumor creates a scalp defect measuring 8 cm in diameter, with calvarial bone denuded of periosteum at its base. Which of the following is most appropriate for coverage of this patient's defect? A ) Dermal regeneration template, followed by skin autograft B ) Fasciocutaneous free tissue transfer C ) Full-thickness skin autograft D ) Interpolated scalp flaps, with skin autograft to cover the secondary defect E ) Pericranial flap, covered with skin autograft

The correct response is Option D. In considering nasal reconstruction, the surgeon must adequately describe the location of the defect since it is important in choosing a reconstructive method. The classically described nasal subunits, based on location, include the ala(s), tip, soft triangle(s), sidewall(s), and dorsum. The current defect is limited to the nasal side wall. Bilobed flaps are ideal for circular defects located at the nasal tip. Through recruitment and rotation of lax tissue from the nasal dorsum or sidewall, it shifts tissue towards the tip. Similarly, dorsal nasal flaps rotate tissue from the nasal dorsum to reconstruct tip defects. The defect shown is also too large to close with either a bilobed or dorsal nasal flap. A cheek flap would be inadequate for a side-wall defect, since it would efface the important anatomic junction between the cheek and the nose. While either a full-thickness or split-thickness skin graft could adequately close this side-wall defect, the increased thickness of a full graft would have better cosmesis with less secondary contracture and distortion. A full-thickness graft should be harvested from an area anatomically as close as possible to the defect. While local flaps are preferred on the face, the nasal side wall is considered a privileged area for skin grafting since the native skin is thin and there is strong underlying bony structure to resist contractile forces of skin grafts. In other areas of the nose, skin grafts are generally avoided. The defect shown could also have been closed with a forehead flap, but this was not listed as an option.

2015 A 78-year-old woman undergoes resection of a melanoma on the nose. Margins are free of tumor. A photograph is shown. Which of the following methods of reconstruction is most appropriate for this defect? A ) Bilobed flap B ) Cheek flap C ) Dorsal nasal flap D ) Full-thickness skin graft E ) Partial-thickness skin graft

The correct response is Option B. Although each of the disorders or syndromes listed is associated with a periocular soft-tissue or orbital abnormality, only neurofibromatosis is associated with agenesis of the sphenoid wing. While plastic surgeons primarily manage the soft-tissue manifestations of neurofibromatosis, the disorder is associated with skeletal abnormalities including sphenoid wing aplasia, macrocephaly, scoliosis, and tibial bowing. Sphenoid wing aplasia causes brain herniation through the middle cranial fossa into the orbit with pulsatile exophthalmos, vertical dystopia, and an enlarged orbital volume. Surgical correction involves bone grafts or titanium mesh to reconstruct the sphenoid wing with reduction of the temporal lobe into the middle cranial fossa. Unilateral coronal synostosis is characterized by foreshortening of the orbital roof on the affected side. Synostosis of the coronal suture causes superior displacement of the lesser wing of the sphenoid with the radiographic appearance of the harlequin deformity. Patients may have strabismus occurring secondary to relative paresis of the superior oblique muscle. The child may tilt the head to compensate. Treacher Collins syndrome is associated with lower eyelid colobomas, zygomatic hypoplasia with hypoplastic or absent zygomas and inferolateral orbital rim clefting, absence of eyelashes, and bilateral mandibular hypoplasia. Hemifacial microstomia is a variant of craniofacial microsomia, distinguished by the presence of concomitant ocular abnormalities, including epibulbar dermoids. The Tessier No. 5 cleft begins behind the canine and extends through the maxillary sinus to the orbital floor. Colobomas of the lateral lower eyelids and clefting of the upper lip medial to the oral commissure are associated.

2015 A 9-year-old boy is brought to the office because of a long-standing history of pulsatile exophthalmos and proptosis of the right side. CT scan shows absence of the greater sphenoid wing. Which of the following is the most likely diagnosis? A ) Hemifacial microstomia B ) Neurofibromatosis C ) Tessier No. 5 cleft D ) Treacher Collins syndrome E ) Unilateral coronal synostosis

The correct response is Option E. Patients with rarer facial clefts are classified according to the system proposed by Tessier in 1973. Each cleft is classified by the soft-tissue findings and the bony defects as they relate to the orbit. Nos. 0 through 7 occur in the lower half of the face as delineated by the orbital commissure, and Nos. 9 through 14 occur in the upper half of the face. The most common facial cleft is the No. 7 cleft centered in the region of the zygomaticotemporal suture. It includes variable expression of macrostomia and hypoplasia of the zygoma.

2015 According to the Tessier cleft classification types, at which of the following areas is the most common facial cleft centered? A ) Frontonasal suture B ) Frontozygomatic suture C ) Nasomaxillary buttress D ) Superior orbit E ) Zygomaticotemporal suture

The correct response is Option D. Myxomas are slow-growing benign tumors. When they present in the infant face, they are most common in the maxilla or mandible. They present as a painless, progressive facial swelling and should be surgically removed with a clear margin. These tumors should have a clear margin to prevent incomplete resection and continued growth. They are not always well circumscribed, so a normal margin or tissue plane should be resected with the tumor.

2015 An 11-month-old male infant is noted by his parents to have a painless, progressive, right maxillary growth. At an outside facility, an incisional biopsy is performed. The pathology shows sinonasal myxoma. Postoperative MRI shows residual tumor with surrounding inflammation. Which of the following is the most appropriate treatment plan? A ) Chemotherapy and radiation therapy B ) Chemotherapy only C ) Curettage debulking of the tumor D ) Surgical resection with clear margin E ) Observation

The correct response is Option B. Salivary gland tumors are relatively rare and make up about 3 to 4% of all head and neck neoplasms. Approximately 80% of salivary gland tumors originate in the parotid gland. Approximately 80% of parotid gland tumors are benign. Facial paralysis may be associated with malignant tumors and is a sign of neural invasion. Malignant tumors may also metastasize to the regional lymph nodes and to distant sites. Mucoepidermoid carcinoma is the most common malignancy of the parotid gland and the second most common malignancy of the submandibular and minor salivary glands. Mucoepidermoid carcinomas contain two major elements: mucus-secreting cells, and epithelial cells of the epidermoid variety. Low-grade tumors are associated with a predominance of mucus-secreting cells lining cysts and intervening nests of well-differentiated epidermoid cells. High-grade tumors show few or no mucus-secreting cells and the epidermoid cells are poorly differentiated. Intermediate-grade tumors are defined by less cyst formation than low-grade tumors with nests of epidermoid and less differentiated intermediate cells. The biologic behavior of mucoepidermoid carcinomas correlates with their histologic grade. On the basis of the nerve invasion, the appropriate treatment for this patient includes radical parotidectomy with facial nerve sacrifice. A neck dissection should also be performed for high-grade lesions or those with suspicious adenopathy. Postoperative radiation therapy is usually recommended for higher-grade mucoepidermoid cancers. Pleomorphic adenoma, also known as benign mixed tumor, is the most common benign tumor of the parotid gland. This tumor is histologically characterized by epithelial and connective tissue elements, with stellate and spindle cells interspersed with a myxoid background. Warthin tumor (papillary cystadenoma lymphomatosum) is the next most common tumor of the parotid gland and is also benign. Warthin tumors predominantly occur in males and are bilateral in 10% of patients. Histologically, they are characterized by papillary cysts and mucoid fluid as well as nodules of lymphoid tissue. Hemangiomas are the most common salivary gland tumors found in children, and usually involve the parotid gland. Like other hemangiomas, they are benign and characterized by a rapid growth phase around the age of 1 to 6 months, followed by gradual involution over 1 to 12 years. Histologically, the tumors are composed of capillaries lined by proliferative endothelial cells. Squamous cell carcinoma is a malignant tumor that rarely involves the parotid gland, in comparison with the skin and aerodigestive tract. When squamous cell cancers occur in the parotid gland, they are usually of metastatic origin, although primary squamous cancers of the salivary glands do occur. They are histologically identical to squamous cell cancers arising from other sites with epithelial cells that form sheets or compact masses that invade adjacent connective tissue. Round nodules of keratinized squamous cells, known as "keratinous pearls," are the hallmark of well-differentiated squamous cell carcinoma.

2015 An otherwise healthy 35-year-old woman is evaluated for a 3-cm left parotid mass. Physical examination shows weakness of facial muscles on the side of the tumor. CT scan shows several enlarged cervical lymph nodes. Parotidectomy is performed, and pathologic examination shows a mixed population of poorly differentiated epithelial cells and intermediate cells with occasional secretory cells and neural invasion. Which of the following is the most likely diagnosis? A ) Hemangioma B ) Mucoepidermoid carcinoma C ) Pleomorphic adenoma D ) Squamous cell carcinoma E ) Warthin tumor

The correct response is Option C. Although the hemangioma shown is sizable, judicious mobilization of the scalp and galeal scoring allow the resultant defect to be easily closed without resorting to delayed rotational flaps, tissue expanders, or skin grafting. Single-staged excision and linear closure is the simplest, most predictable manner of closing small to intermediate-sized scalp defects such as the one in the patient described. The maximum defect using this technique is not well defined, but at least one series has demonstrated successful primary closure of defects in infants of up to 7 cm without using the other alternatives listed or tissue expansion. Delaying the resection for another 2 years might be productive in a younger child, but further involution is unlikely in a 4-year-old. Staged excision is possible, but unnecessary for this lesion; moreover, repair along the edge of the friable hemangioma can lead to wound problems.

2015 An otherwise healthy 4-year-old boy is brought to the office for evaluation of a large scalp hemangioma. A photograph is shown. The family has requested removal of the hemangioma before he enters school. Which of the following is the most appropriate management in this patient? A ) Deferral of surgery for 2 years B ) Excision and skin grafting C ) Single-stage excision and closure D ) Staged excision and closure E ) Tissue expansion

The correct response is Option D. Dermoid cysts are common in the lateral brow. They present as firm, well-circumscribed, slow-growing masses that have the potential for infection or continued growth. Surgical excision is recommended and no imaging is required. CT scan or MRI require sedation and are unnecessary risks for this patient with a lateral dermoid. Midline masses do require imaging because of the risk for intracranial excision.

2015 An otherwise healthy term 6-month-old male infant is evaluated for a mobile, firm, well-circumscribed mass at the right lateral brow in the area of the zygomaticofrontal suture. Which of the following procedures is the most appropriate next step in management? A ) CT scan B ) Fine-needle aspiration of the mass C ) MRI D ) Surgical excision of the mass E ) Ultrasonography

The correct response is Option E. Ameloblastomas are benign odontogenic tumors derived from odontogenic epithelium. They are typically slow growing, and present in the fourth or fifth decade of life as a mandibular mass in most individuals (80%). Odontogenic cementoblast tissue is not appropriate because this tissue gives rise to an extremely rare benign odontogenic tumor, the cementoma. Gingiva is the mucosal covering of the alveolar bone. Enamel is the dense compound of teeth.

2015 In patients with ameloblastoma, which of the following cell populations gives rise to this tumor? A ) Cementoblast tissue B ) Enamel C ) Gingiva D ) Nerve root E ) Odontogenic epithelium

The correct response is Option B. Closed reduction has historically been the standard treatment for subcondylar fractures of the mandible. Its widespread use is attributed to the idea that closed reduction results in fewer complications with similar functional and aesthetic outcomes compared with open reduction and internal fixation (ORIF). For instance, complications such as facial nerve damage and excessive scarring are significantly decreased because of the noninvasive nature of this approach. However, as highlighted by ongoing debate, a consensus regarding outcomes between open and closed reduction is not evident in the literature. In short, some studies conclude that both approaches produce roughly similar results, while other studies have associated an array of unfavorable outcomes with closed reduction. These outcomes include facial asymmetry, deviation upon mouth opening, skeletal malocclusion, and chronic pain of the temporomandibular joint (TMJ). The fact that many of these parameters lack standardization in the duration of the treatment further obscures the debate. Larger studies with consistent parameters are needed to reassess outcomes with the surgical techniques and technology present today. However, it is unlikely that a large enough trial will deliver granular evidence to conclusively quell this debate. Another controversial point regarding closed reduction is the length of time a patient should spend in maxillomandibular fixation (MMF). Many surgeons choose to apply fixation for a very short period (i.e., 2 weeks) to avoid ankylosis of the TMJ secondary to forced immobilization during MMF. While the etiology of ankylosis is not completely understood, it is hypothesized that trauma leading to intracapsular hematoma results in fibrosis and excessive bone formation, ultimately causing hypomobility of the affected side. Given the current hypothesis, ankylosis of the TMJ is most likely a manifestation of direct injury within the joint capsule or condylar head itself. It is imperative to point out that as a result, there should be a lower risk of ankylosis in subcondylar fractures compared with fractures of the condylar head. Therefore, the position of the fracture line relative to the joint capsule should be closely examined, and a longer period of MMF should be employed if there is no involvement of the condylar head, disc, or capsule. A longer period of MMF commonly results in better union of the fractured segments with no increase in the incidence of ankylosis. In a nondisplaced fracture or minimally displaced fracture with a functional occlusion, 4 to 6 weeks of MMF is recommended, followed by 2 to 3 weeks of guiding elastics. The same treatment applies in the case of a nondisplaced bilateral fracture. However, this scenario is less common because the force parameters to cause the bilateral fracture are often greater and tend to displace the fracture fragments significantly, necessitating ORIF. The patient will most likely develop shortening of the masseter muscle and stiffness with prolonged immobilization during 8 to 10 weeks. With bilateral instability, the fracture pattern is unstable and soft diet alone would likely lead to malocclusion. A gunning splint is often used in edentulous mandible only and is not functional MMF.

2015 In patients without loss of posterior vertical height, which of the following is the most appropriate duration of maxillomandibular fixation (MMF) for treatment of minimally displaced bilateral subcondylar fractures of the mandible? A ) 1 week B ) 4 weeks C ) 8 weeks D ) None; stabilize mandible with gunning splint for 8 weeks E ) None; treat patient with soft diet

The correct response is Option D. Most isolated orbital fractures involve the orbital floor, a majority of which is made up of the maxillary bone. The maxillary bone is quite thin behind the infraorbital rim, and is perforated by the infraorbital nerve passing in a canal below it. Most pure blowout fractures involve the orbital floor. Long-term epidemiologic data regarding the natural history of orbital bone fractures are important for the evaluation of existing preventive measures and for the development of new methods of injury prevention and treatment.

2015 Isolated orbital fractures most commonly occur in which of the following bones? A ) Ethmoid B ) Frontal C ) Lacrimal D ) Maxillary E ) Zygomatic

The correct response is Option A. Osteoconduction is the ability of a material to encourage bone to grow toward and along its surface. Osseointegration is defined as the direct chemical bonding of an alloplast to the surface of bone without an intervening layer of fibrous tissue. These qualities are important in identifying an appropriate bone substitute in craniofacial reconstruction. Hydroxyapatite is a bone substitute that has capacity for both osteoconduction and osseointegration and is the base for many of most widely used bone substitutes. Hydroxyapatite is the principal mineral component of bone and comprises 60% of the calcified human skeleton. It has been used clinically for more than 25 years. It is biocompatible, and all forms are resistant to absorption after implantation. Silicone products do not osseointegrate or osteoconduct. Polymethylmethacrylate causes an extreme exothermic reaction associated with the setting process and is deleterious to adjacent bone and soft tissue, even with vigorous saline irrigation. It can be designed or shaped ex vivo to avoid thermal injury. Titanium osseointegrates, but it does not osteoconduct. It provides ideal protection and reconstruction in certain clinical situations where infection is of higher concern. High-density porous polyethylene implants have pore sizes ranging from 100 to 300 µm. These aid in tissue ingrowth and implant fixation. These implants may be coated with polyhydroxyethylmethacrylate and calcium hydroxide. The hydrophilic nature and calcium coating result in osteoconductivity and a fibro-osseous matrix.

2015 Which of the following bone substitutes has the capacity for osteoconduction and osseointegration? A ) Hydroxyapatite B ) Polymethylmethacrylate C ) Porous polyethylene D ) Silicone E ) Titanium

The correct response is Option D. When surgically reducing a zygomaticomaxillary complex (ZMC) fracture (tripod fracture), the surgeon uses multiple landmarks to confirm adequate reduction. The lateral orbital sidewall is the most reliable landmark as it is a three-dimensional landmark (junction of frontal bone, sphenoid, and zygoma). Other landmarks include the zygomaticofrontal suture, the zygomaticomaxillary lateral buttress, the orbital rim, and the zygomatic arch. Because the orbital rim is easily visualized, surgeons may rely heavily on this landmark for alignment. Sometimes due to orbital swelling or poor visualization, the lateral orbital wall is not easy to assess for adequate reduction of the fracture. If the orbital rim appears to be reduced (well aligned) but the ZMC fracture is not well reduced, then an ipsilateral unreduced naso-orbito-ethmoid (NOE) fracture is the most likely reason. It is important to reduce the NOE fracture before aligning the orbital rim for the ZMC fracture reduction. Postoperative deformity will ensue if the NOE fracture is not reduced and the ZMC is plated in position based on the orbital rim alignment. The deformity will lead to enlarged orbital volume and facial widening, loss of malar projection, and enophthalmos. Proper reduction and initial management are important, as secondary corrections are more difficult and have more morbidity.

2015 Which of the following concomitant fractures is most likely to affect the perceived reduction when performing open reduction and internal fixation of a zygomaticomaxillary complex fracture? A ) Anterior maxillary wall comminution with loss of bone B ) Articular tubercle of the zygomatic arch C ) Le Fort I fracture D ) Naso-orbito-ethmoid fracture E ) Orbital floor blowout fracture

The correct response is Option C. Craniofacial clefts are highly variable and can range from mild forme fruste to severe with involvement of all the layers of the soft tissue and skeletal structures. Tessier described a classification system still in use today based on the bony and soft-tissue landmarks involved in the clefts. Corrective surgery is required in stages, with the timing based on the level of functional problems, including ocular exposure, and airway problems early on. The clefts can be broadly grouped into midline and paramedian clefts (numbers 0 to 14, 1 to 13, 2 to 12), oro-nasal-ocular clefts (numbers 3 to 11, 4 to 10, 5 to 9), and lateral clefts (numbers 6, 7, and 8). In this system, the numbers 0 to 6 refer to clefts below the orbital and numbers 8 to 14 are above the orbit, with 7 being truly lateral.

2015 Which of the following congenital clefts involves the lower eyelid? A ) 0 B ) 1 C ) 5 D ) 8 E ) 9

The correct response is Option A. Osteoradionecrosis (ORN) of the mandible is uncommon but can occur in up to 10% of patients after undergoing radiation therapy for oral cancers. The risk increases once radiation doses exceed 6500 cGy. Most reports of ORN have dental caries and extraction sites as precipitating factors. Periodontal disease can also lead to ORN. After undergoing radiation therapy, patients can develop oral candidiasis and xerostomia, and they may also have edentulous mandibles with dental implants after reconstruction. However, these do not increase the risk for ORN. Surgical resection and hyperbaric oxygen therapy are the mainstays of treatment.

2015 Which of the following factors is most likely to increase the risk for osteoradionecrosis secondary to radiation therapy? A ) Dental caries B ) Edentulous mandible C ) Oral thrush D ) Osseointegrated implants E ) Radiation dose of 3500 cGy

The correct response is Option E. The American Association for Accreditation of Ambulatory Surgery Facilities (AAAASF) requires surgeons to be Board Certified or Board Eligible with a Board recognized by the American Board of Medical Specialties. A patient who underwent general anesthesia needs a responsible adult to supervise him/her for 12 to 24 hours. Surgeons are required to demonstrate that they hold unrestricted hospital privileges at an acute-care hospital within 30 minutes' driving time of the ambulatory facility. If pediatric patients are cared for, at least one member of the team needs to be certified in Pediatric Advance Life Support (PALS). Ambulatory care facilities are inspected every 3 years by the AAAASF.

2015 Which of the following is a standard for accreditation of an ambulatory surgery facility as it relates to plastic surgeons? A ) All team members of the ambulatory facility caring for pediatric patients must be certified in Pediatric Advanced Life Support B ) Ambulatory facilities are inspected every 5 years C ) Patients undergoing ambulatory surgery during general anesthesia require a responsible adult to monitor them for 1 to 2 hours after discharge from the ambulatory surgery center D ) Plastic surgeons working in the ambulatory surgery facility must be board certified by the American Board of Plastic Surgery E ) Surgeons operating in an ambulatory surgery center are required to demonstrate that they hold unrestricted hospital privileges at an acute-care hospital within 30 minutes' driving time of the facility

The correct response is Option C. The temporomandibular joint is classified as a ginglymoarthrodial joint since it has both hinge and sliding components during jaw opening. These functions take place in the two separate compartments in the joint, upper and lower, that are effectively separated by an articular disc. During the first 20 mm of jaw opening, the condyle rotates in the lower compartment (space between condylar head and articular disc) in a nearly pure hinge motion. For further opening to take place, the condyle translates (or shifts) forward with the articular disc through the upper compartment (space between the articular disc and the joint surface). The other options describe other joint configurations. Examples of each are: saddle, thumb basilar joint; pivot, atlasaxis (C1-2 neck); gliding, tarsal bones in the foot; ellipsoid, radiocarpal articulation.

2015 Which of the following terms best describes the temporomandibular joint? A ) Ellipsoid (condyloid) B ) Gliding (arthrodial) C ) Hinge-sliding (ginglymoarthrodial) D ) Pivot (trochoid) E ) Saddle (ephippial)

The correct response is Option C. The palatopharyngeus muscle is one of the two structures (arches) that surround the oropharyngeal tonsils. Its action is to pull the pharynx upward during swallow to help separate the oropharynx from the nasopharynx. A sphincter pharyngoplasty transposes bilaterally the palatopharyngeal muscles to create an even greater barrier from the oropharynx to the nasopharynx to decrease velopharyngeal insufficiency (VPI). The hyoglossus muscle is an extrinsic tongue muscle and, like the stylohoid muscle, has one of its attachments to the hyoid bone. Neither of these muscles is commonly used to treat VPI. The levator veli palatini is an important muscle to mobilize in primary hard palate reconstruction in cleft cases, but not in secondary situations where the muscle is not functional. The pharyngeal constrictors are used in pharyngeal flaps and are primarily employed in secondary situations with a sagittal closure pattern.

2016 A 10-year-old boy who previously underwent palatoplasty for cleft palate is evaluated because of velopharyngeal insufficiency associated with minimal palatal elevation and coronal closure pattern. The most appropriate procedure to treat this patient targets which of the following muscles? A ) Hyoglossus B ) Levator veli palatini C ) Palatopharyngeus D ) Pharyngeal constrictor E ) Stylohyoid

The correct response is Option D. Cleft lip and/or palate is found in up to 20% of patients with CHARGE (coloboma of the eye, heart defects, atresia of the nasal choanae, retardation of growth and/or development, genital and/or urinary abnormalities, and ear abnormalities and deafness) syndrome. It is the second most common syndrome associated with cleft lip and palate, following van der Woude syndrome. CHARGE syndrome is believed to be caused by a microdeletion or mutation in the CHD7 (chromodomain helicase DNA-binding protein 7) gene. Over the years, there have been other conditions found to be associated with this syndrome, including limb anomalies, renal anomalies, omphalocele, cranial nerve anomalies, and tracheoesophageal fistula. Syndactyly is not typically associated with CHARGE syndrome. Intrahepatic hemangiomas are not typically associated with CHARGE syndrome. Craniosynostosis is not typically associated with CHARGE syndrome. Lip pits are commonly found in patients with van der Woude syndrome.

2016 A 12-hour-old male newborn is evaluated in the neonatal intensive care unit. A diagnosis of CHARGE syndrome is suspected. Review of the medical record and physical examination show congenital heart defect, eyelid colobomas, microtia, hypogonadism, and growth retardation. Which of the following additional conditions is/are most likely in this patient? A ) Craniosynostosis B ) Intrahepatic hemangiomas C ) Lower lip pits D ) Orofacial cleft E ) Syndactyly

The correct response is Option B. This patient has hemifacial microsomia (also referred to as craniofacial microsomia, Goldenhar syndrome, or oculoauriculovertebral spectrum), the second most common congenital facial malformation after cleft lip/palate. It is estimated to occur in 1:5600 to 1:20,000 live births. Unilateral (or less commonly, bilateral) facial hypoplasia is the primary manifestation, and it can involve ocular structures (orbital size, position, globe development, coloboma, epibulbar dermoids), mandibular structures, auricular structures (preauricular tags and sinuses, microtia or anotia, hearing loss), cranial nerve palsies (cranial facial nerve [VII] is most common); and soft-tissue underdevelopment (including facial clefts). In addition, patients can have cardiac, musculoskeletal, central nervous system, and renal manifestations. The etiology of hemifacial microsomia (HFM) appears to be heterogeneous. Early investigators using animal models of the condition believed that this was the result of an isolated vascular event in the stapedial artery. While this mechanism cannot be completely dismissed and may have a role in some cases, the prevalence of extracranial findings strongly suggests a more sublime pathogenesis: faulty neural crest cell (NCC) migration. Both genetic and environmental factors have been demonstrated to alter or impair NCC activity. Recent studies have implicated genetic factors in the development of this disorder. Nevertheless, the majority of patients with HFM have no family history and less than 2% of cases arise from autosomal dominant inheritance. Certain vasoactive and nonsteroidal anti-inflammatory drug (NSAID) medications can increase the risk of HFM; maternal ingestion of omega oils has not been associated. Maternal smoking can increase the risk of having a child with HFM, but the association is weak and this is not causative in the vast majority of patients. Uterine deformation can lead to facial asymmetry; however, this typically improves with growth and does not lead to the ear and jaw anomalies in the patient described.

2016 A 6-month-old male infant is evaluated for facial asymmetry noted at birth. Physical examination shows symmetry of the upper one-third of the face, but the right cheek is less prominent than the left. The occlusal plane is canted upward to the right, and the chin point is deviated to the right. In addition, the right ear is small and constricted, and an adjacent preauricular soft-tissue appendage is noted. He has a palsy of the right marginal mandibular nerve. He is otherwise healthy. The parents should be informed that the asymmetry was most likely caused by which of the following? A ) Autosomal dominant mutation B ) Disruption of normal developmental sequence C ) Maternal smoking D ) Omega oil ingestion during pregnancy E ) Uterine deformation

The correct response is Option C. Full-thickness nasal alar defects must be reconstructed with all missing lamellae including lining, support, and coverage in addition to all subunits that are missing. Although many reconstructive options exist, each with their respective benefits and drawbacks, only those options which provide lining, support, and coverage will successfully address the defect in question. Of the options listed, only a forehead flap and skin graft for lining with septal cartilage reconstructs all missing lamellae. Although a composite helical root graft comprises all three nasal lamellae, it is not big enough to address both the alar and the soft triangle nasal subunits.

2016 A 14-year-old girl is evaluated for a dog bite injury to the left nasal alar rim that she sustained 6 months ago. Physical examination shows full-thickness loss of the left alar and soft triangle subunits of her nose. Which of the following treatment options best addresses all missing components? A ) Composite helical root graft B ) Conchal cartilage graft and bilobed flap for coverage C ) Forehead flap and skin graft for lining with septal cartilage graft D ) Forehead flap with nasolabial flap for lining E ) Nasolabial flap with full-thickness skin graft for lining

The correct response is Option D. Load-bearing osteosynthesis differs from load-sharing osteosynthesis in that the fracture plate assumes all of the load of the given bone rather than distributing the load among the plate and the bone. Examples of load-sharing osteosynthesis include lag screw fixation, maxillomandibular fixation, nonlocking mandibular border plate, and monocortical miniplate (Champy principle) fixation. An example of load-bearing fixation is a mandibular locking reconstruction plate for a comminuted fracture.

2016 A 16-year-old boy is evaluated for multiple fractures to the mandible after crashing his ATV into a tree. Which of the following modalities best takes advantage of load-bearing osteosynthesis in the management of this patient's fractures? A ) Bicortical border plate for a mandibular parasymphyseal fracture B ) Champy plate fixation for a mandibular angle fracture C ) Lag screw fixation for a displaced fracture of the mandibular symphysis D ) Locking reconstruction plate for a comminuted mandibular body fracture E ) Maxillomandibular fixation for a minimally displaced mandibular subcondylar fracture

The correct response is Option C. Le Fort I advancement and lengthening can result in dramatic changes to facial appearance. These include increased midfacial projection and fullness, increased upper lip vermilion fullness, decreased upper lip height, decreased depth of the nasolabial folds, and increased tooth show in repose and smile. Additionally, as the bony platform of the caudal piriform region is advanced, the alar base widens, and the tip rotates cephalad, increasing the nasolabial angle. It is important to counsel patients on this expected outcome preoperatively so that they are fully informed of the facial changes they will experience.

2016 A 17-year-old boy is evaluated for Le Fort I advancement and lengthening procedure. Medical history shows repair of bilateral cleft lip and palate in infancy. Postoperatively, which of the following facial changes is most likely in this patient? A ) Decreased nasolabial angle B ) Decreased tooth show in repose C ) Increased alar base D ) Increased depth of nasolabial folds E ) Increased length of upper lip

The correct response is Option B. van der Woude syndrome is an autosomal dominant condition affecting 1:40,000 to 1:100,000 live births. It involves lower lip pits or mounds, and cleft lip and/or palate. Fifteen percent do not have the associated pits, so genetic associations are useful in diagnosis and counseling. IRF6 is the mutation associated with van der Woude syndrome as well as popliteal pterygium syndrome (webbing behind the knee, lower lip pits, cleft lip and/or palate, and genital hypoplasia). PTCH1 is found in Gorlin syndrome, also known as basal cell nevus syndrome. FGFR1 is associated with Kallmann syndrome (hypogonadotrophic hypogonadism with anosmia, cleft lip and/or palate, renal aplasia/agenesis, dental defects). The majority of orofacial clefts are nonsyndromic (70% of all cleft lip/palate; 50% of all cleft palate only). Cleft lip/palate is more common in males (2:1) and cleft palate only is more common in females (1:2). 22q11.2 deletion is associated with DiGeorge sequence or velocardiofacial syndrome (cardiac anomalies, abnormal facies, thymic aplasia, cleft palate, hypocalcemia/hypoparathyroidism—CATCH). Trisomy 13 is associated with Patau syndrome (cleft palate, CNS disorders, microcephaly, polydactyly, rocker bottom foot, urogenital defects, and cardiac anomalies.

2016 A 2-month-old male infant is evaluated for cleft lip and lower lip pits. Medical history shows his father also had a cleft lip at birth. Which of the following is the most likely genetic defect in this patient? A ) FGFR1 B ) IRF6 C ) PTCH1 D ) Trisomy 13 E ) 22q11.2

The correct response is Option C. The facial skeleton acts as a buffer and barrier against injury to the brain and deep structures. Reconstruction of the buttress system is integral to reforming the form and function of the facial skeleton. Zygomaticomaxillary complex (ZMC) fractures, sometimes referred to as tripod fractures, may involve fractures of the following: zygomaticotemporal suture, maxillary sinus, and occasionally sphenoid bone. Correction involves reduction of the fracture, commonly with miniplate fixation or with 2-point wire fixation. These fractures can also involve the orbital floor and maxillary wall. Symptoms may include numbness in the maxillary nerve (V2 of trigeminal (V) nerve), subconjunctival hemorrhage, diplopia, and enophthalmos. Orbital floor fractures can be isolated, as in a blowout, or involve other structures (e.g., orbital rim, orbital medial/lateral walls, orbital apex). Additional fractures are typically associated with higher-force injury and may require more extensive surgical repair. Diagnostic study typically includes CT scan of the facial bones to assess the fractures and to delineate the extent of the facial trauma. There are three main indications to treat isolated orbital fractures: 1. Entrapment of the extraocular muscles: this may cause injury and permanent dysfunction of the globe if not reduced. 2. Prevention of globe malposition: malposition of the globe may occur leading to diplopia or enophthalmos. This may not be apparent immediately after the injury because of soft-tissue edema. Reduction is indicated if there are signs of early enophthalmos, displacement of more than 50% of the orbital floor, or significant soft-tissue displacement. 3. Lateral orbital wall displacement: when the injury is more severe, the lateral orbital bone (sphenoid bone) may impact the orbital apex or even the midcranial fossa. The CT scan should be examined for muscle entrapment, herniation of orbital contents, and associated fractures (naso-orbito-ethmoid, zygomaticomaxillary, Le Fort). Open reduction and internal fixation can be performed emergently or after edema has subsided, and usually after the orbit has been examined by an ophthalmologist for ocular injury. Repair of the orbital floor can be done using several different types of materials—alloplastic and autologous. This is usually dictated by the surgeon's preference and the extent of the injury.

2016 A 21-year-old man is evaluated because of pain and double vision 2 weeks after being punched in the face. An orbital fracture is suspected. Which of the following characteristics of an orbital floor fracture is the most common indication for surgical repair? A ) Concomitant naso-orbito-ethmoid fracture B ) Contralateral zygomaticomaxillary complex fracture C ) Herniation of orbital contents with diplopia D ) Medial wall fracture without additional symptoms E ) Nondisplacement

The correct response is Option A. Sialocele and cutaneous fistula may occur as a complication of a major salivary gland injury or after attempted parotid gland or duct repair. Prior to intervention, the surgeon should confirm the presence of salivary fluid in the aspirate by evaluating for amylase (>1000 U/L). This helps to rule out hematoma vs. infection. At this time, the accepted initial treatment of sialocele and cutaneous fistula is conservative management. This includes pressure dressings, repeated aspirations, limited intake by mouth, and the use of antisialagogues to decrease salivary flow. Most studies have shown that most sialoceles and fistulas will resolve within 2 to 3 weeks with this approach. Secondary repair of the duct is generally difficult because of granulation and scar tissue. This runs the risk for delayed or poor wound healing as well as facial nerve injury. Radiation therapy has been used in the past but has been found to have a high failure rate in ductal injuries. More than 6 weeks is needed for gland atrophy to occur after radiation. There is also the risk of inducing malignancy, which has led to the abandonment of radiation for the treatment of sialoceles and cutaneous fistulas. Antisialagogues primarily have a role as an adjunct therapy to decrease salivary flow. Alone, they have had mixed reviews in the literature and have been found generally ineffective in major salivary duct injuries. Tympanic neurectomy has a high failure rate and does not speed recovery. Internalization of salivary flow could be considered for failures of conservative management. It has lower surgical risk than wound exploration and duct repair.

2016 A 21-year-old man undergoes repair of the parotid gland and Stensen duct after sustaining an injury while being assaulted with a glass bottle. Ten days later, salivary leakage from the wound is noted. Which of the following is the most appropriate next step in management? A ) Pressure dressings and limited oral intake B ) Radiation therapy C ) Re-exploration of the wound and repair of the duct injury D ) Tympanic neurectomy E ) Observation only

The correct response is Option C. The patient described has an ameloblastoma. Ameloblastomas are benign tumors of odontogenic origin. Treatment is surgical. Conservative management, such as local curettage, is associated with high recurrence rate. The most appropriate treatment is segmental mandibulectomy, reconstruction, and dental rehabilitation. Because meloblastoma is benign, neither adjuvant therapy nor neoadjuvant therapy is indicated.

2016 A 25-year-old woman is evaluated because of facial swelling around the jaw and loosening teeth. The swelling has worsened progressively. Physical examination shows unilateral right facial swelling around the third molar. CT scan of the mandible shows a radiolucent, multicystic, unilocular lesion in the right mandibular angle and confirmed root resorption. Which of the following series of treatments is most appropriate for this patient? A ) Local curettage of the lesion followed by cancellous bone graft reconstruction B ) Neo-adjuvant radiation therapy, segmental mandibulectomy, and reconstruction C ) Segmental mandibulectomy and reconstruction D ) Segmental mandibulectomy, reconstruction, and postoperative chemotherapy after adjuvant therapy E ) Segmental mandibulectomy, reconstruction, dental rehabilitation, and sentinel node biopsy

The correct response is Option E. Only Saethre-Chotzen syndrome is not associated with a fibroblast growth factor receptor (FGFR) mutation, but it is instead associated with a mutation of the TWIST gene. It is characterized by brachycephaly with maxillary hypoplasia, a prominent ear crus, and syndactyly. Pfeiffer syndrome (characterized by brachycephaly, mild syndactyly, and broad toes and thumbs) is associated with mutations of both FGFR1 and FGFR2. Apert syndrome (characterized by brachycephaly, syndactyly, and cognitive delay) is autosomal dominant and associated with mutations of FGFR2. Crouzon syndrome is also autosomal dominant and characterized by shallow orbits, craniosynostosis, and maxillary hypoplasia. Antley-Bixler syndrome has four phenotypes, some of which are associated with FGFR mutations and characterized by craniosynostosis, choanal atresia, and radiohumeral synostosis.

2016 A 3-month-old female infant is brought to the office for evaluation of head shape. Examination shows brachycephaly and abnormal orbits. Genetic testing is negative for fibroblast growth factor receptor (FGFR) mutation. Which of the following is the most likely diagnosis? A ) Antley-Bixler syndrome B ) Apert syndrome C ) Crouzon syndrome D ) Pfeiffer syndrome E ) Saethre-Chotzen syndrome

The correct response is Option B. This child demonstrates the classic presentation of deformation plagiocephaly with his posterior occiput flattening and compensatory ipsilateral forehead bossing with anterior shifting of his ear on the same side. There are many reasons for deformational plagiocephaly, especially with the current practice of "back to sleep." Treatment of this focuses on removing the pressure on the affected side. His head tilt and decreased motion are consistent with torticollis. Initial treatment is stretching and occupational therapy to restore usage and balance of his neck muscles. Although helmet therapy can help alleviate pressure on this side, it is not addressing the issue. Further studies have demonstrated that deformational plagiocephaly can be treated with positional changes and behavior modification up until 7 to 8 months of age without difference in head asymmetry compared with those children who began helmet therapy at a younger age. There is no difference in children who fail positional changes compared with those who initiate helmet therapy immediately. At this child's age of 3 months and with obvious torticollis, the most appropriate initial therapy should be focused on resolving his torticollis and giving him a trial of nonhelmet therapy.

2016 A 3-month-old male infant is brought to the office by his parents for evaluation of skull asymmetry that has worsened since birth. Birth history includes prolonged labor that required cesarean delivery. Physical examination shows flattening of the right posterior occiput with ipsilateral forehead bossing. From a superior view, the right ear is 1 cm more anterior than the left ear, and the anterior fontanelle is open without any bulging. The child's head tilts to the right and has decreased range of motion when looking to the left. The left side of his neck feels tighter and more rigid compared with the right side. Which of the following is the most appropriate initial management of this child? A ) Cerebral palsy evaluation B ) Occupational therapy of the neck C ) Posterior cranial vault expansion D ) Passive molding helmet therapy E ) Sternocleidomastoid muscle release

The correct response is Option C. This child most likely has branchio-oto-renal (BOR) syndrome, an autosomal dominant syndrome. Two genetic mutations (EYA1 and SIX1) have been identified. The clinical findings include auricular malformations, preauricular skin pits, hearing loss, branchial fistulae, external auditory canal stenosis, and renal anomalies. Renal ultrasound should be performed to rule out renal anomalies. Family history of hearing loss should also raise suspicion. Ear anomalies can also be associated with the oculo-auriculo-vertebral spectrum (OAV), within which hemifacial microsomia falls. OAV spectrum may have associated mandibular, facial nerve, and ocular findings (e.g., epibulbar dermoid coloboma). Other possible associations include congenital heart defects, such as ventricular septal defect or tetralogy of Fallot. This child does not have other craniofacial findings of hemifacial microsomia, thus cardiac echocardiography is not indicated. Internal cerebral arterial anomalies may be associated with PHACE syndrome (P, posterior fossa; H, hemangioma; A, arterial anomalies; C, cardiac defects; E, eye anomalies). Investigation of cerebral circulation by MRI of the brain would be prompted by a facial hemangioma, which is absent in this child. While resection of the preauricular pits may be offered, it does not aid in ruling out other findings and there is no urgency to perform surgical resections in this child at 3 months of age. Repeating a hearing test will not confirm or rule out OAV or BOR syndrome.

2016 A 3-month-old male infant is evaluated because of bilateral preauricular pits and small ears. Physical examination is otherwise unremarkable. The patient failed the newborn hearing test; the parents have not repeated the test. They are not very concerned, because the father is also hearing impaired. Which of the following is the most appropriate next step in management? A ) Echocardiography B ) MRI of the brain C ) Renal ultrasound D ) Repeat hearing test E ) Surgical treatment of the preauricular pits

The correct response is Option E. Flat tympanograms indicate the eustachian tubes are not draining properly. The action of the tensor veli palatini is key in the equalization of pressure between the middle ear and nasopharynx. In a child with a cleft palate, the tensor is abnormally inserted into the levator palatini, and as a result, there is an increased incidence of ear eustachian tube dysfunction. The primary function of the levator veli palatini is velopharyngeal closure by velar elevation and retrodisplacement. The palatopharyngeus is responsible for controlling the velopharyngeal sphincter by controlling velar size, shape, and position. The palatoglossus works to modulate speech by velar depression and tongue elevation. The superior constrictor is important for pharyngeal closure during swallowing and speech.

2016 A 3-year-old boy is evaluated for unilateral cleft lip and palate. Tympanogram shows bilateral noncompliance. Which of the following muscles is most likely responsible for this finding? A ) Levator veli palatini B ) Palatoglossus C ) Palatopharyngeus D ) Superior constrictor E ) Tensor veli palatini

The correct response is Option C. Tracheostomy and assisted ventilation is the most appropriate management. The patient described demonstrates a significant number of central and mixed apneas, indicating that the initiation of breathing by central drive is diminished and that tracheostomy with assisted ventilation set at a base ventilation rate, such as synchronized intermittent mandatory ventilation (SIMV), would be optimal to ensure adequate minute ventilation. Continuous positive airway pressure (CPAP) until 6 to 8 years of age and then a facial advancement is incorrect. CPAP will assist with obstructive apnea, but will not treat central apnea because no baseline ventilation rate is set using CPAP mode. Hypertelorism surgery within the next year is incorrect because hypertelorism surgery will not treat sleep apnea without facial advancement. Only the wide orbit position would be treated with an orbital box osteotomy or facial bipartition without advancement. Facial advancement surgery within the next year is incorrect because advancement will correct the obstructive apnea and a portion of the mixed apneas, but will not address the central sleep apnea. Observation with tracheotomy and assisted ventilation will be more prudent until the central apnea rate improves, usually after time with maturation. The patient described will still have persistent central apnea and is at higher risk for relapse of the face due to the age of under 6 years.

2016 A 3-year-old girl with craniofrontonasal dysplasia presents to the craniofacial team because of moderate exorbitism, severe central sleep apnea, and severe hypertelorism (interocular distance of 40 mm). Which of the following is the most appropriate management plan for this patient? A ) Continuous positive airway pressure until 6 or 8 years of age B ) Facial bipartition surgery within the next year C ) Immediate tracheostomy and assisted ventilation D ) Monobloc advancement surgery within the next year E ) Orbital box osteotomy surgery within the next year

The correct response is Option B. The child has already had a lip repair and although she may require revision in the future, the primary goal at this age is palate repair. A repair after age 18 months is considered a late palate repair. The literature supports that late palate repairs have worse speech outcomes. Additionally, internationally adopted children with late palate repairs have higher fistula rates, more hypernasality and velopharyngeal insufficiency, and are more likely to require speech surgery.

2016 A 30-month-old female infant is evaluated for unilateral complete cleft lip and palate. She was adopted from China, where her lip was previously repaired. Her parents are concerned about her palate, which has not been repaired, and the appearance of her lip and nose. Which of the following is the most appropriate next step in management? A ) Repair the palate and perform a rhinoplasty B ) Repair the palate first and revise the lip at a later time C ) Revise the lip and repair the palate simultaneously with a gingivoperiosteoplasty D ) Revise the lip now and repair the palate at a later time E ) Wait one year until more English vocabulary is acquired, and then undergo speech evaluation before palate repair

The correct response is Option A. This is a rare overgrowth disorder involving genetic defect of chromosome 11, commonly characterized by the presence of macrosomia, omphalocele, and macroglossia. These symptoms typically present after 22 weeks' gestation. Additionally, infants with Beckwith-Wiedemann syndrome often display auricular abnormalities, including characteristic ear folds. While a common symptom of diabetic fetopathy is macrosomia, a diagnosis of diabetic fetopathy would be incorrect. Common symptoms of diabetic fetopathy include congenital anomalies of the internal organs, such as nephromegaly or hepatomegaly; no such symptoms are detected in this fetus. The absence of cleft lip and/or cleft palate nephromegaly also precludes the diagnosis of van der Woude syndrome. Additionally, the genetic abnormality responsible for van der Woude syndrome involves chromosome 1, not chromosome 11. Likewise, trisomy 21 (Down syndrome) is not the correct diagnosis, as the chromosomal abnormality responsible for this disease involves chromosome 21, not chromosome 11. Finally, Binder syndrome is a congenital facial malformation most commonly characterized by a flat nose and retrusion of the midface, neither of which are observed in this fetus. Therefore, Beckwith-Wiedemann syndrome is the correct answer.

2016 A 30-year-old primigravid woman is referred to the office because ultrasonography showed polyhydramnios at 22 weeks' gestation. Additionally, the fetus was observed to have macrosomia, omphalocele, macroglossia, and microtia. There is no evidence of nephromegaly or hepatomegaly. A groove between the lobule and antitragus is noted. Chromosomal analysis indicates rearrangements on chromosome 11p15. Which of the following is the most likely diagnosis? A ) Beckwith-Wiedemann syndrome B ) Binder syndrome C ) Diabetic fetopathy D ) Down syndrome E ) van der Woude syndrome

The correct response is Option B. A relative afferent pupillary defect, or Marcus Gunn pupil, results from an optic nerve dysfunction. The physical examination maneuver used to evaluate the optic nerve for a relative afferent pupillary defect is the swinging flashlight test described above. The most common cause of Marcus Gunn pupil is a lesion on the optic nerve, optic neuritis, or severe retinal disease but can also be caused by trauma. The normal response to light in one eye is constriction of both pupils and when the light is taken away both eyes dilate (consensual light reflex). In an abnormal exam the normal eye/optic nerve will cause a normal constriction of both pupils but the light shone in the affected eye will not have normal light perception (minimal pupillary constriction). The afferent pathway refers to the nerve impulse sent from the pupil to the brain via the optic nerve of one eye. The efferent pathway is the impulse sent back to both pupils via CN III and ciliary ganglia, causing both pupils to constrict.

2016 A 40-year-old man is evaluated for orbital fractures from a direct blow to the right eye. A swinging flashlight test is performed to evaluate a suspected afferent pupillary defect. Which of the following is the most appropriate description of this procedure? A ) The flashlight shone into the left eye causes both eyes to constrict; then when the light is shone in the right eye, the right eye constricts but the left eye does not B ) The flashlight shone into the left eye causes consensual constriction, and then both eyes dilate when the light is shone in the right eye C ) The flashlight shone into the left eye causes consensual constriction; then when the light is shone in the right eye, the right eye dilates while the left eye constricts D ) The flashlight shone into the left eye causes consensual dilation followed by paradoxical constriction when swung to the right eye E ) The flashlight shone into the left eye causes the left eye to constrict while the right eye remains dilated; both eyes are constricted when the light is shone in the right eye

The correct response is Option D. A 2-cm mass with a clinically negative neck, pT2cN0, merits a supraomohyoid neck dissection because of the high risk of occult spread in this zone. A supraomohyoid dissection removes the lymph nodes in zones I, II, III; a modified neck dissection covers I-IV. A smaller tumor, pT1N0, can be managed with a "tight 'wait and watch'" strategy. A radical neck dissection includes the sternocleidomastoid muscle and is not necessary for a clinically negative neck. Adding radiation therapy to a reflex neurovascular dystrophy is also not necessary for an N0 neck.

2016 A 42-year-old man is evaluated because of a 2-cm mass in the anterior floor of the mouth. Examination shows no palpable masses in the neck. In addition to surgical tumor removal, which of the following is the most appropriate additional step in management? A ) Radiation only B ) Radical neck dissection C ) Radical neck dissection and radiation D ) Selective neck dissection E ) Observation

The correct response is Option D. In cases of unilateral facial palsy in which the ipsilateral facial nerve is unavailable for use as a donor nerve, nerve transfers are the best option. Of the local options, the ipsilateral masseteric nerve provides the best and most commonly used transfer due to its proximity and relatively low morbidity when harvested. In one study, spontaneous smiling was achieved routinely in 59% of patients and occasionally in 29% of patients by using this technique. Although the hypoglossal nerve is sometimes used as a donor nerve for ipsilateral facial reanimation, its harvest can be associated with considerable oropharyngeal morbidity due to ipsilateral tongue atrophy. A cross-facial nerve transfer does not provide for single-stage reconstruction. In many cases where there are sufficient contralateral branches of the facial nerve for cross-facial transfer, this option can provide optimal function, but its use requires a staged approach. Use of the ipsilateral glossopharyngeal nerve as a donor in this situation would also create significant oropharyngeal function morbidity, and is therefore not commonly used. Use of the ipsilateral spinal accessory nerve as a donor nerve is rarely used due to the fact that its use often requires uncoordinated movements for function. Ipsilateral masseteric nerve is a significantly better choice.

2016 A 45-year-old man with a 10-year history of unilateral facial palsy is evaluated for dynamic reanimation of the lower face. Single-stage free gracilis transfer to restore smiling function is planned. The proximal stump of the ipsilateral facial nerve is not available as a donor nerve. Which of the following options is most appropriate for this patient because it is associated with the lowest donor site morbidity/loss of function? A ) Cross-facial nerve transfer B ) Ipsilateral glossopharyngeal nerve C ) Ipsilateral hypoglossal nerve D ) Ipsilateral masseteric nerve E ) Ipsilateral spinal accessory nerve

The correct response is Option B. This patient has osteonecrosis of the jaw secondary to bisphosphonate medication used to treat her bony cancer metastases. The mandible is more commonly affected than the maxilla. The appropriate next step in management is to begin antiseptic mouth rinses in addition to stressing good dental hygiene. Asymptomatic bone exposure can be followed for progression, and early cases appear to often resolve spontaneously when the bisphosphonates are discontinued. Ruggiero et al. proposed a three-stage classification system and treatment algorithm for bisphosphonate-related osteonecrosis of the jaw based on clinical features. In Stage 1, there is exposed and necrotic bone that is otherwise asymptomatic, as in the patient described above. In Stage 2, there is exposed and necrotic bone with pain and clinical evidence of infection. Antiseptic mouth rinses, oral antibiotics, and superficial debridement are recommended for this stage. Stage 3 includes all the features of Stage 2 and one or more of the following: pathologic fracture, extraoral fistula, and osteolysis extending to the inferior mandibular border. Stage 3 patients require surgical debridement or resection in addition to antiseptic mouth rinses and oral antibiotics. Dental extraction of infected, unsalvageable teeth should be performed, but extractions may also result in further nonhealing wounds and bone exposure. Segmental mandibulectomy is usually followed by immediate reconstruction with osseous or osteocutaneous free flap reconstruction. The role for prophylactic titanium plating of the jaws to prevent pathologic fracture has not been studied.

2016 A 45-year-old woman with a history of metastatic breast cancer previously treated with zoledronate is evaluated because of an area of exposed necrotic mandibular bone measuring 1 × 1 cm. She has no pain, and there is no clinical evidence of infection. Panoramic x-ray study (Panorex) shows no evidence of fracture. Which of the following is the most appropriate next step in management? A ) Aggressive debridement and prophylactic titanium plate placement B ) Antiseptic mouth rinses and observation C ) Dental extraction and intravenous antibiotics D ) Segmental mandibulectomy and osteocutaneous free flap reconstruction E ) Superficial debridement and oral antibiotics

The correct response is Option B. The patient described most likely has a facial fracture and, thus, the most appropriate next step in management is to obtain a maxillofacial CT scan. A retrospective analysis of patients with facial trauma who had maxillofacial CT scan for the evaluation of facial fractures identified five independent criteria highly predictive of facial fracture. Any of the following five physical examination criteria - the Wisconsin criteria - were predictive of facial fracture: bony step-off or instability, periorbital swelling or contusion, Glasgow Coma Scale score less than 14, malocclusion, or tooth absence (sensitivity, 98.2%). Because the patient has a missing tooth, he should be evaluated with a maxillofacial CT scan. Although soft diet and eventual referral to a dentist as well as elevation of the head of bed with cool compress are reasonable recommendations, the first important step is obtaining a diagnosis. Because the patient does not have tenderness of the cervical spine, there is no indication for obtaining cervical spine x-ray studies. Systematic and efficient decision instruments in the management of multisystem trauma patients are critical in identifying all injuries including those of the maxillofacial skeleton and the cervical spine. The accurate use of systematic decision instruments could potentially result in a dramatic decrease in the number of CT scans and x-ray studies performed annually in the United States.

2016 A 50-year-old man is evaluated for facial trauma sustained after falling off a bicycle 5 days ago. The patient reports mid-facial pain and resolved swelling. Physical examination of the face shows no signs of trauma. Intraoral examination shows traumatic loss of tooth No. 8. He has no cervical spine tenderness or pain to active range motion. Which of the following is the most appropriate next step in management? A ) Obtain cervical spine MRI B ) Obtain maxillofacial CT scan to evaluate for facial fracture C ) Recommend elevation of the head of the bed and ice pack to the face D ) Recommend soft diet and referral to a dentist E ) No further management is necessary

The correct response is Option E. This patient has rhinophyma, characterized by progressive hypertrophy of the sebaceous glands in the nose resulting in a bulbous appearance that can be deforming. This can cause significant emotional distress. This is considered an end-stage presentation of rosacea. Treatment consists of topical and oral antibiotics (for mild cases, and for treating rosacea), and surgical excision (tangential excision) and laser therapy. Alcohol abuse was thought to be associated with this condition, as the facial appearance can be ruddy/red/flushed, but there is no scientific study that demonstrates any association of alcohol use with development of rhinophyma. Rhinophyma is not associated with exposure to radiation. There is no evidence that sun exposure causes rhinophyma, although malignant lesions have been coincidentally found within rhinophyma tissues. There is no association between psoriasis and rosacea or rhinophyma.

2016 A 50-year-old man is evaluated for progressive growth of his nose. He desires improvement of the appearance of the nose, which is causing him distress. On examination, the nose has thickened skin and glands in the nasal tip and a bulbous appearance. Tangential excision of the nose is planned. This patient most likely has a history of which of the following risk factors? A ) Alcohol abuse B ) Excessive sun exposure C ) Psoriasis D ) Radiation treatment for teenage acne E ) Rosacea

The correct response is Option A. The preganglionic parasympathetic innervation of the parotid gland originates in the inferior salivatory nucleus located in the brainstem, and travels within the glossopharyngeal nerve. Shortly thereafter, these fibers then travel within the tympanic and lesser petrosal branches to the otic ganglion. Within the otic ganglion, these fibers then synapse with the postganglionic cells, whose axons then travel within the auriculotemporal nerve to reach the parotid gland. The great auricular nerve is incorrect because there are no preganglionic or postganglionic parasympathetic fibers to the parotid gland carried within the great auricular nerve. The facial (VII) nerve is incorrect because there are reganglionic parasympathetic nerve fibers that are in close proximity to the facial nerve, and these are carried within the tympanic and lesser petrosal branches of the glossopharyngeal nerve. There are no preganglionic or postganglionic parasympathetic nerve fibers to the parotid gland carried within the facial nerve. The marginal mandibular nerve is incorrect because there are no preganglionic or postganglionic parasympathetic fibers to the parotid gland carried within the marginal mandibular nerve. The external carotid nerve plexus is incorrect because this option describes the location of the postganglionic sympathetic nerve fibers to the parotid gland. Sympathetic innervation of the parotid gland originates within the spinal cord, and the preganglionic axons then travel to the superior cervical ganglion, where they synapse with the postganglionic cells, and then travel along the external carotid arterial plexus to reach the parotid gland.

2016 A 50-year-old man who underwent superficial parotidectomy for a benign tumor 9 months ago comes to the office because of a 6-month history of gustatory sweating. Which of the following nerves carries the parasympathetic postganglionic nerve fibers to the parotid gland in a healthy patient? A ) Auriculotemporal B ) External carotid plexus C ) Facial (VII) D ) Great auricular E ) Marginal mandibular

The correct response is Option A. Eyelid reconstruction is often a challenging endeavor, and depends upon thorough knowledge of the local anatomy, reconstructive options, and extent of resection. Eyelid defects are commonly classified into three categories—less than 25%, 25 to 50%, and greater than 50%—in order to help the surgeon choose the best reconstructive options. Whenever reconstructing an eyelid defect, it is important to identify the structures involved and be sure to reconstruct the lamellae that have been resected. In this patient, there is a defect that involves both the anterior (skin and orbicularis) and posterior (conjunctiva and tarsus) lamellae, and both must be addressed. Of the available options, the only one that appropriately provides the correct amount and quality of anterior and posterior lamellae for a greater than 50% defect is a Hughes tarsoconjunctival flap and Tripier lid switch flap. A Tenzel semicircular flap would be appropriate for a 50% or less defect of the lower eyelid, but would not be the best option for a 65% defect. Although a local cheek V-Y advancement flap and a buccal mucosal graft provides both anterior and posterior lamellar reconstruction, a local V-Y advancement flap of the appropriate size would likely create a cosmetically unappealing contour, and using buccal mucosa for the posterior lamella would not provide the amount of support needed for the lower eyelid to prevent ectropion postoperatively. Although the combination of a Mustardé flap and a hard palate mucosal graft is a possibility because it provides both anterior and posterior lamellar reconstruction, the use of a hard palate mucosal graft would incorporate keratinized epithelium into the reconstruction, and, therefore, would be less desirable than the use of conjunctiva because it can cause corneal irritation. The Cutler-Beard flap provides both anterior and posterior lamellar reconstruction and, therefore, the buccal mucosal graft is redundant and unnecessary.

2016 A 50-year-old woman is evaluated after undergoing Mohs micrographic surgery of the lower eyelid to remove a basal cell carcinoma 2 days ago. On examination, a 65% defect of the lower eyelid involving both the anterior and posterior lamellae is noted. Nether canthi are involved. Which of the following is the most appropriate reconstructive option for this patient? A ) Hughes tarsoconjunctival flap and Tripier lid switch flap B ) Local cheek V-Y advancement flap and buccal mucosal graft C ) Mustardé flap and hard palate mucosal graft D ) Tenzel semicircular flap and periosteal flap E ) Upper eyelid Cutler-Beard flap and buccal mucosal graft

The correct response is Option C. This patient has a bilateral maxillectomy defect following resection via a Le Fort I osteotomy. While small defects can be successfully addressed with prosthetic palatal obturators that fit through the wound margins and clasp to the remaining teeth, larger defects can rarely be obturated because of their weight and instability due to lack of dentition and an alveolar ridge. Bone grafting is also not an option for such a large defect, particularly one resulting from a malignancy where postoperative radiation therapy is likely. In general, bone grafts are only indicated in benign conditions with bone loss less than 5 cm in length. Even when these conditions are met, they require coverage with well-vascularized tissue. Temporalis muscle flaps can be used to reconstruct palatal defects, but in this case, the skeletal elements of the mid face have been removed. Temporalis muscle flaps alone will result in loss of mid face projection. Similarly, the anterolateral thigh free and rectus abdominis myocutaneous free flaps are frequently used to reconstruct posterior maxillary defects but, though bulkier than temporalis muscle flaps, will not maintain midfacial projection in this patient with loss of the entire palate and premaxilla. The most appropriate reconstruction for this patient is the fibula osteocutaneous free flap. This flap will restore midfacial height, width, and projection. It has adequate bone stock for osseointegrated implant placement dental restoration as well. The skin paddle is used to close the palatal defect, separating the oral cavity from the nasal cavity. Several other osteocutaneous free flaps, including the iliac crest and scapular osteocutaneous free flaps, have also been used for similar reconstructions.

2016 A 50-year-old woman with hypertension and diabetes comes to the office because of a large mid-palatal cancer. She undergoes a total palatectomy. A photograph is shown. Which of the following is the most appropriate method of reconstruction? A ) Anterolateral thigh free flap B ) Bilateral temporalis muscle flaps C ) Fibula osteocutaneous free flap D ) Iliac crest bone grafting E ) Rehabilitation with a palatal obturator

The correct response is Option D. It is important to note that any patient experiencing pain like this should be worked up for recurrent disease. Nevertheless, this question addresses two issues in head and neck reconstruction. First, most non-oral cavity head and neck cancers are now treated primarily with chemoradiation protocols. This leaves surgery for salvage or for dealing with the morbidities of the therapy. In this case, the patient is now experiencing bilateral osteoradionecrosis with a pathologic fracture on the right. Had this problem stopped with the left side, debridement and letting the mandible sway to the ipsilateral side would be a form of management, although it can be disfiguring. Secondly, from a reconstructive standpoint, this is a challenging case; but advances in flaps and preoperative modeling allow for managing such situations in one procedure. Given how long the fibular free flap can be fashioned and the ability to remove large central pieces of bone, surgeons can create two osseous segments vascularized off the same pedicle. Since many of these patients have difficult necks from a recipient vessel standpoint, decreasing the number of needed anastomoses is beneficial. Two free flaps can be performed at the same time or sequentially, but this adds increasing morbidity and complexity to the situation. In this case, a single fibular free flap with plates designed using stereolithographic modeling was adequate to restore the patient's ability to eat and articulate. It also improved his occlusion, and, with debridement, stopped his pain. Postreconstructive imaging is shown. Now that the right side is involved, simple debridement only would leave the patient potentially an oral cripple. By definition, the bone is of poor quality and simple open reduction and internal fixation will not restore the patient's function or promote proper healing. Hyperbaric oxygen is used for osteoradionecrosis, but numerous studies now question its benefit in craniofacial bone, and it would not address the pathologic fracture on the right. Hyperbaric oxygen was shown to positively influence the osteoradionecrotic tissues prior to and post-free tissue reconstruction. Finally, a free bone graft in this situation, even with additional soft tissue coverage, would not address the major issue of hypoperfusion to the affected bone.

2016 A 55-year-old man is evaluated because of right-sided jaw pain, speech problems, and oral dysphagia 5 years after successfully completing a chemoradiation protocol for base-of-tongue cancer. CT imaging shows no signs of recurrence or distant metastatic disease. A panoramic x-ray study (Panorex) is shown. Which of the following is the most appropriate management of this constellation of symptoms? A ) Debridement of mandible B ) Hyperbaric oxygen therapy C ) Open reduction and internal fixation D ) Osteocutaneous fibular free tissue transfer E ) Rib graft

The correct response is Option E. This patient has T2N1M0 (Stage II) high-grade mucoepidermoid carcinoma, and total parotidectomy with ipsilateral cervical dissection is the appropriate treatment. Because the patient is lymph node positive, both an ipsilateral cervical lymph node dissection and postoperative adjuvant radiotherapy are indicated. A superficial parotidectomy would perhaps be indicated in a low-grade mucoepidermoid carcinoma, but in a high grade lesion, total parotidectomy is more appropriate. Ipsilateral cervical lymph node dissection and postoperative adjuvant radiotherapy would be indicated in this patient, however. Although total parotidectomy and ipsilateral cervical lymph node dissection are appropriate, two factors make postoperative adjuvant radiotherapy an important component of treatment: the node positive status of the neck, and the high grade of the tumor. Chemotherapy remains somewhat controversial in the treatment of mucoepidermoid arcinoma, and, to date, is not a part of standardized therapy.

2016 A 55-year-old woman is evaluated because of a 2-year history of an enlarging right-sided facial mass. Examination shows a roughly 4-cm firm mass in the right parotid region and a firm lymph node in zone III of the ipsilateral neck. Imaging does not show metastatic disease. Fine-needle aspiration of the mass suggests high-grade mucoepidermoid carcinoma of the parotid gland. Which of the following is the most appropriate treatment in this patient? A ) Neoadjuvant chemoradiation and reassessment of tumor response before additional treatment B ) Superficial parotidectomy with ipsilateral cervical lymph node dissection C ) Superficial parotidectomy with ipsilateral cervical lymph node dissection and postoperative adjuvant radiotherapy D ) Total parotidectomy with ipsilateral cervical lymph node dissection E ) Total parotidectomy with ipsilateral cervical lymph node dissection and postoperative adjuvant radiotherapy

The correct response is Option A. The circumflex scapular artery, which is a branch of the subscapular artery, supplies blood to the lateral and medial borders of the scapular bone. The scapula free flap was first described in 1978 by Saijo. It can be harvested as part of a chimeric flap that includes other tissues supplied by the subscapular arterial system, such as the latissimus dorsi muscle, serratus anterior muscle, and scapular or parascapular skin. The tip of the scapula receives its blood supply from the angular branch of the thoracodorsal artery and has also been utilized as a pedicle for the inferior portion of the scapula. While the scapular bone is not as thick as the fibula bone, it provides adequate stability for mandibular reconstruction. Cutaneous branches of the circumflex scapular artery supply the scapular and parascapular skin and, therefore, a skin flap can be harvested simultaneously to close soft-tissue defects. The transverse cervical artery and the dorsal scapular artery primarily supply the trapezius muscle and overlying skin. The thoracoacromial artery supplies the pectoralis major muscle and overlying skin. The lateral thoracic artery supplies both the lateral portion of the pectoralis major muscle and the skin in the axillary region.

2016 A 58-year-old man is evaluated because of floor-of-mouth cancer that is invading the mandible. A segmental mandibulectomy and reconstruction with an osteocutaneous free flap that includes bone from the lateral border of the scapula are planned. The vascular pedicle supplying this flap is based on which of the following arteries? A ) Circumflex scapular B ) Dorsal scapular C ) Lateral thoracic D ) Thoracoacromial E ) Transverse cervical

The correct response is Option D. Nasal air emission, along with hypernasality and facial grimacing, are all findings that are commonly observed in patients with an incompetent velopharyngeal port or velopharyngeal dysfunction (VPD). Consonant omissions and glottal stops are two common pathologic compensatory speech patterns that such patients exhibit but are not directly caused by VPD. Instead, they are maladaptive compensatory speech patterns often present in patients with VPD. Although some patients with VPD can have hyponasality or sibilant lateralization, these are unrelated to VPD or an attempt to compensate for VPD.

2016 A 6-year-old girl with 22q11.2 deletion syndrome is brought to the office by her parents. The parents report trouble understanding the child when she speaks. An incompetent velopharyngeal port is suspected. Which of the following is the most likely indication of this malfunction? A ) Consonant omission B ) Glottal stop C ) Hyponasality D ) Nasal air emission E ) Sibilant lateralization

The correct response is Option E. Daily saliva production is on the scale of 500 to 2000 mL per day. Excessive drooling in patients with cerebral palsy or other severe neurologic impairment is caused by inefficient swallowing. The submandibular glands contribute the most to basal salivary production, approximately 60%. The parotid gland contributes approximately 20% to basal salivary production. Sublingual glands and minor salivary glands each contribute 10%. The lacrimal gland is responsible for production of the aqueous layer of the tear film.

2016 A 6-year-old girl with cerebral palsy is evaluated for sialorrhea. Medical treatment has been unsuccessful. Surgical intervention for drooling control is planned. Which of the following glands contributes the most to basal salivary production? A ) Lacrimal B ) Minor salivary C ) Parotid D ) Sublingual E ) Submandibular

The correct response is Option E. This is a case of Ramsay Hunt syndrome, also known as herpes zoster oticus. It is a variant of typical herpes zoster, in that in its reactivation, it affects both afferent neurons as well as motor axons of the facial nerve. A combination of corticosteroids, narcotics, and acyclovir is the standard of care. Facial myokymia is mainly associated with multiple sclerosis and causes a wormlike motion in the facial muscles. Möbius syndrome is a congenital condition characterized by, among other nerve conditions, facial paralysis. Cholesteatoma is a benign growth in the middle or external ear formed from desquamated keratin. It can act locally and be very aggressive, so much so, that if left untreated, it could cause destruction of the facial nerve. This is not a typical presentation of Lyme disease.

2016 A 60-year-old man is evaluated for a painful unilateral facial rash with blisters, intense ear pain, and complete ipsilateral facial nerve paralysis. Which of the following is the most likely diagnosis? A ) Cholesteatoma B ) Facial myokymia C ) Lyme disease D ) Möbius syndrome E ) Ramsay Hunt syndrome

The correct response is Option B. A bilobed flap is usually used for nasal reconstruction. A 3-mm margin is adequate for most basal cell carcinoma 2 cm or smaller. This will result in an almost 13-mm helical rim defect, slightly too large for primary closure. At this defect size, ear cupping from primary closure can result in a less pleasing final aesthetic result.

2016 A 60-year-old man presents with a 7-mm basal cell carcinoma on the ear. Which of the following is the most appropriate management? A ) Wedge excision with wide 3-mm margins and bilobed flap closure B ) Wedge excision with wide 3-mm margins and chondrocutaneous advancement flap closure C ) Wedge excision with wide 3-mm margins and primary closure D ) Wedge excision with wide 10-mm margins and chondrocutaneous advancement flap closure E ) Wedge excision with wide 10-mm margins and primary closure

The correct response is Option A. In the patient described, with a history of previous radiation therapy and a fullthicknessdefect, a free latissimus dorsi musculocutaneous flap would be the best choice for soft-tissue reconstruction. Skin grafting over an implant cranioplasty would not be a suitable option in this setting. Skin grafts could be considered in nonradiated, partial-thickness defects of the scalp or for coverage of scalp rotation flap donor sites. A pedicled trapezius musculocutaneous flap would not reach the cranial vertex without undue tension and is better suited for full-thickness occipital defects. Scalp rotation flaps are ideal for defects up to 8 cm in diameter in a nonradiated scalp. The patient's history of previous radiation therapy would likely compromise the viability of a large rotation flap in this setting. Negative pressure wound therapy over an implant cranioplasty in a radiated field would not allow for soft-tissue healing and coverage.

2016 A 60-year-old woman with a history of squamous cell carcinoma of the scalp treated with resection, skin grafting, and total scalp radiation therapy is evaluated for osteoradionecrosis of the cranial vertex. After full-thickness debridement, which of the following is the most appropriate method for soft-tissue reconstruction? A ) Coverage with a free latissimus dorsi musculocutaneous flap B ) Coverage with a pedicled trapezius musculocutaneous flap C ) Coverage with a scalp rotation flap D ) Negative pressure wound therapy E ) Skin grafting

The correct response is Option A. Cryptotia is a congenital ear deformity in which the upper pole appears buried beneath the mastoid skin. It is a common auricular malformation in Asians. Children with this condition often present when they are in elementary school, and are unable to wear eyeglasses. The cause of this condition is the abnormal distribution of the intrinsic auricular muscle. Malformation of the conchal bowl results in prominence of the pinna from the head. Incomplete fusion of the six hillocks does not result in cryptotia, nor does it result from failure of the two superior hillocks (3 and 4) to arise from the branchial arches. Failure of the antihelix to furl during weeks 12 to 16 results in a protruding scapha.

2016 A 7-year-old Asian American boy is brought to the office because of congenital cryptotia. Which of the following is the most likely pathophysiologic explanation for his condition? A ) Abnormal distribution of the intrinsic transverse and oblique auricular muscles B ) Failure of the antihelix to furl during weeks 12 to 16 of gestation C ) Failure of hillocks 3 and 4 to arise from the first and second branchial arches D ) Incomplete fusion of the six hillocks E ) Malformation of the conchal bowl

The correct response is Option C. Staged autogenous cartilage reconstruction remains the gold standard to correct microtia. The urgency and method of treatment for associated hearing loss depends on whether the problem is unilateral or bilateral, whether external ear construction is planned, and the condition of the middle ear structures. Bilateral hearing loss can result in problems with language development and learning and requires early intervention to improve or restore hearing. This is usually done with external hearing aids in early childhood followed by atresia repair or a bone-anchored hearing aid (BAHA) later in life. Historically, most authorities have concluded that patients with congenital unilateral hearing loss naturally adjust and experience few functional implications. Consequently, most do not routinely recommend operative correction for unilateral hearing loss. There are some recent reports of improved language development after early biaural hearing restoration, and some clinicians now support early treatment. If correction is considered, the two primary options are atresia repair, wherein the canal is opened and the middle ear is reconstructed, or use of a hearing aid. Atresia repair is usually deferred until the external ear framework is placed or the reconstruction is complete. The success depends on the presence and normalcy of the middle ear structures. Jahrsdoerfer's 10-point scoring system (10 being most suitable for reconstruction) grades the anatomic appearance and relationship of the middle ear structures by temporal CT scan. Middle ear reconstruction is contraindicated for a score of 5 or less. Based on the information provided, this patient with anotia (-1 point), absence of the stapes (-2 points) and absence of the incus (-2 points), has a maximum score of 5 and would not be a good candidate for middle ear reconstruction. Certainly, this procedure would not improve hearing as well as a hearing aid such as the BAHA. BAHA uses osseointegrated implants to affix a hearing aid. This provides excellent correction of conductive hearing loss and is widely used. BAHA is the best surgical option to restore hearing in this patient if so desired. However, placement should be deferred until after the autologous ear reconstruction is completed.

2016 A 7-year-old girl is evaluated because of the ear anomaly shown. CT scan of the temporal bone of the affected ear shows an absent stapes and incus. Construction using autogenous rib cartilage is planned. The girl's family inquires about options for improving hearing to the affected ear. Which of the following is the most appropriate response? A ) Atresia repair should be performed after auricular reconstruction B ) Atresia repair should be performed before auricular reconstruction C ) A bone-anchored hearing aid should be placed after auricular reconstruction D ) A bone-anchored hearing aid should be placed before auricular reconstruction E ) Hearing cannot be improved in this patient

The correct response is Option E. All of the choices are suitable options for reanimation of the eyelid; however, transfer of a regional nerve, such as the motor nerve to the masseter, will result in the quickest reinnervation of the orbicularis oculi muscle. Nerve grafts may be used to overcome a wide neural gap; however, nerve regeneration proceeds slowly at only 1 mm per day. Cross-facial nerve grafts take an extended period of time to regenerate due to the long distance from the contralateral side. While gold weights are immediately effective, they are static procedures. Restoration of a reflexive blink requires neural input from the zygomatic branch of the facial nerve or direct neurotization of the orbicularis oculi muscle. Functional muscle transfers used for eyelid reanimation include the frontalis and temporalis; however, because they are innervated by nerves other than the facial, coordinated movements are not present.

2016 After schwannoma resection with facial nerve sacrifice, which of the following reconstructions provides the quickest restoration of natural reflexive dynamic blink? A ) Cross-facial nerve grafting B ) Functional muscle transfer C ) Gold weight insertion D ) Ipsilateral nerve grafting E ) Nerve transposition

The correct response is Option D. Fullness of the soft-tissue envelope is expected after any severe trauma, regardless of location. This is further compounded by operative dissection to effect open reduction and internal fixation. Although most soft-tissue swelling around a fracture site dissipates in several weeks, there will be persistent thickening of the soft tissue in the area for many months after the trauma. It is nearly impossible to "over-reduce" a zygomaticomaxillary (ZMC) fracture in the sagittal plane. Malreduction of these injuries characteristically results in flattening of the cheek (not fullness), vertical dystopia, or enophthalmos. Prominent hardware can create contour irregularities, but this would be localized and would not lead to global cheek fullness. Furthermore, the plate choices in areas where they could potentially be visible (i.e., the orbital rims) here are quite low profile and would be very unlikely to create excessive soft-tissue fullness. Hardware infection can rarely occur and lead to fullness of the cheek. Nevertheless, the absence of pain and the presence of healed wounds make this improbable. Mucocele from the maxillary sinus after repair of ZMC fracture is a rare but described entity and can result in localized soft-tissue fullness. These often occur in the context of a concurrent sinus infection and present many months to years after the injury. Soft-tissue swelling is usually accompanied by pressure and/or pain, and the symptoms can wax and wane. External drainage is not uncommon.

2016 An 18-year-old woman is evaluated for persistent fullness of the left cheek 3 months after undergoing open reduction and internal fixation of a left zygomaticomaxillary complex fracture. She has no pain. The fracture was fixated at the inferior orbital rim (1.3-mm-thick titanium plate), the frontozygomatic suture (1.3-mm-thick titanium plate), and the lateral buttress (2.4-mm-thick titanium plate). On physical examination, the incisions are well healed. Which of the following is the most likely cause of this patient's cheek fullness? A ) Excessive profile of the lateral buttress and orbital rim plate fixation B ) Malreduction of the fracture C ) Mucocele from the maxillary sinus D ) Residual soft-tissue edema E ) Subclinical hardware infection

The correct response is Option A. The most likely cause of this child's persistent hypernasality is hypotonia of the muscles involved in speech. Because this is neurogenic, it will be difficult to correct with surgery, so the best option becomes a palatal elevator used when talking to close off the nose posteriorly by pushing up the posterior soft palate. Posterior augmentation with fat grafting would decrease the size of the posterior gap but without adequate closure would still not correct the problem. The child has been in therapy and has failed to show improvement so additional therapy especially in light of the neurologic problem is unlikely to do anything but frustrate the child. A sphincter pharyngoplasty is not a viable option because a pharyngeal flap has been done and without adequate muscle function would fail.

2016 An 8-year-old boy with a history of submucous cleft palate presents with persistent velopharyngeal insufficiency. Surgical history includes a Furlow palatoplasty 3 years ago, with subsequent revision to a superiorly based pharyngeal flap for persistent hypernasality 1 year ago. He had no improvement after the second surgery and speech therapy. Physical examination shows an intact, high and wide pharyngeal flap. Nasoendoscopy shows patent lateral oronasal ports and poor palatal and lateral pharyngeal wall motion. Which of the following treatments is most appropriate in this patient? A ) Fabrication of a custom palatal elevator B ) Pharyngeal flap port revision C ) Posterior wall augmentation with fat grafting D ) Speech therapy with retraining E ) Sphincter pharyngoplasty

The correct response is Option A. Centric occlusion is incorrect because centric occlusion (maximal intercuspation) without centric relation (condylar seating within the glenoid fossa) will not lead to reliable postoperative occlusion. Maxillary occlusion plane angle is incorrect because the maxillary plane angle affects open bite tendency and is not a reliable measure of occlusion. Gonial angle is incorrect because the gonial angle along with the mandible occlusal plane are better predictors of prognathism and open bite tendency and are not a reliable measure of occlusion. Centric relation is incorrect because centric relation without centric occlusion will not lead to reliable postoperative occlusion. Centric occlusion and centric relation is the correct response because maximal intercuspation (centric occlusion) coupled with proper mandible condylar position within the glenoid fossa (centric relation) is most likely to result in optimal occlusion after orthognathic surgery.

2016 In a patient undergoing orthognathic surgery with preoperative class III malocclusion, which of the following anatomic relationships must be retained in order to optimize postoperative occlusion? A ) Centric relation and centric occlusion B ) Condylar seating C ) Maximal intercuspation D ) 2 mm of overbite E ) 2 mm of overbite and 2 mm of overjet

The correct response is Option C. Bilateral expansile lesions of the inferior alveolar nerve canal is pathognomonic of neurofibroma. The lesions on the Panorex are both expansile and not locally destructive of bone, as is common in ameloblastoma and odontogenic keratocysts. Central giant granulomas are most often multilocular, with cortical rupture and root atrophy. Osteosarcoma always has cortical destruction.

2016 In the panoramic x-ray study (Panorex) shown, which of the following is the most likely diagnosis of the bilateral expansile lesions? A ) Ameloblastoma B ) Central giant cell granulomas (CGCG) C ) Neurofibromas D ) Odontogenic keratocysts E ) Osteosarcoma

The correct response is Option C. Nondisplaced mandibular symphyseal fracture is incorrect as the mandible growth centers are located in the condyle and posterior aspect of the ascending ramus of the mandible. Displaced bilateral parasymphyseal fractures is incorrect. Nondisplaced bilateral condyle fractures is correct because the growth centers are located in the mandible condyle regions and a significant amount of trauma resulting in displaced factors of the condyles has occurred. Displaced mandible body fractures is incorrect.

2016 In the pediatric population, which of the following mandibular fracture patterns most commonly results in future growth complications? A ) Displaced bilateral parasymphyseal fractures B ) Displaced mandibular body fractures C ) Nondisplaced bilateral condylar fractures D ) Nondisplaced bilateral parasymphyseal fractures E ) Nondisplaced mandibular symphyseal fractures

The correct response is Option A. Descending palatine artery is the correct response since it is at greatest risk after pterygoid osteotomy and down fracture. It is easily visible after down fracture and mobilization of the Le Fort I segment. It is typically injured during osteotomy prior to its division into the greater and lesser palatine arteries. External carotid artery is incorrect because it is within the neck. Its branches ascend in the face and end in the sphenopalatine artery as it enters the pterygoid foramen. Internal maxillary artery is incorrect because it is the larger artery prior to division into multiple smaller branches including the middle meningeal, sphenopalatine, and descending palatine arteries. Greater palatine artery is incorrect because the greater and lesser palatine arteries are below the level of the pterygoid osteotomy. The descending palatine is more proximal and is at greater risk for injury.

2016 Which of the following arteries is most likely to be injured in a patient undergoing Le Fort I osteotomy? A ) Descending palatine B ) External carotid C ) Greater palatine D ) Internal carotid E ) Internal maxillary

The correct response is Option C. Secondary alveolar bone grafting is performed in mixed dentition. The goals of alveolar bone grafting are to stabilize the alveolus and allow adequate bone stock to support tooth health for the permanent canine (if that is the tooth erupting into the cleft). Generally, the timing of bone grafting is done before the canine has fully erupted. If it has fully erupted, the root may be exposed in the cleft and not well surrounded by bone. It is preferable to perform the bone graft after the deciduous teeth are gone, but before the tooth is fully erupted. An appropriate time to perform the bone graft is when the canine is crowning. Early grafting may be detrimental to midface growth. Orthodontic preparation may be required before performing secondary bone grafting, and there must be enough permanent dentition to support this.

2016 Which of the following is the best dental reference to use for determining the timing for secondary alveolar bone grafting in a patient with a unilateral complete cleft lip/palate? A ) Complete eruption of the central incisor next to the cleft B ) Complete eruption of the permanent canine into the cleft C ) Crowning of the permanent canine D ) Loss of the primary canine adjacent to the cleft E ) Loss of the primary ipsilateral central incisor

The correct response is Option C. The origin of the superior thyroid artery is predictable and most commonly it arises from the external carotid artery. It has, however, been described to arise from all three of the other options, just less frequently.

2016 Which of the following is the most common origin of the superior thyroid artery? A ) Bifurcation of the carotid artery B ) Common carotid artery C ) External carotid artery D ) Internal carotid artery

The correct response is Option B. The most common benign tumors of the jaw are ameloblastoma (37%) and keratocystic odontogenic tumor (KCOT) (14%). Ameloblastomas are slow growing, occur in the 4th to 5th decades of life, and arise from odontogenic epithelium. KCOTs are locally aggressive, occur earlier in life, and also arise from odontogenic epithelium. Curettage or enucleation results in higher recurrence rates. Addition of Carnoy's solution (absolute alcohol, chloroform, glacial acetic acid, and ferric chloride) to the enucleated site for 3 minutes addresses the most common issue of local recurrence. Segmental or marginal resection of the mandible is reserved for recurrence after resection locally.

2016 Which of the following is the primary treatment for keratocystic odontogenic tumor? A ) Curettage only B ) Enucleation and chemoablation C ) Enucleation only D ) Marginal mandibulectomy E ) Segmental mandibulectomy

The correct response is Option C. If there is inadequate rotation, the rotation-advancement repair can result in a short lip (white upper lip). A Tennison, or triangular, flap repair can result in an elongated lip and a visible scar across the lower philtrum. The advantage of the rotation-advancement flap is that the design places the scar along the philtral ridge. Straight-line repairs have been shown to result in blunting of Cupid's bow. An overly wide philtrum is a surgical stigma of a repaired bilateral cleft lip.

2016 Which of the following stigmata is most common after a rotation-advancement repair of a unilateral cleft lip? A ) Blunting of Cupid's bow B ) Elongated lip C ) Short lip D ) Triangular scar across the philtrum E ) Widened philtrum

The correct response is Option E. The answer is uvulopalatopharyngoplasty (UP3). The mainstay of UP3 surgery is to remove the uvula and lateral oropharyngeal tissues. The most common site of obstruction in obstructive sleep apnea (OSA) patients is the retropalatal area, including the lateral pharyngeal walls. This latter anatomic area is even more important if the patient has never had a tonsillectomy; in this case, the tonsils are removed in continuity with UP3. UP3 generally decreases the various indexes documented during a sleep study, such as the apnea-hypopnea index, by about 50%. Thus, those at the border of severe and moderate sleep apnea can see their reported indexes go to a level that can be managed by lifestyle changes without the need for a continuous positive airway pressure (CPAP) machine. On the other hand, in patients with very high indexes, these procedures merely make the disease less severe and may allow for a lower setting on the CPAP machine. Although the CPAP machine which applies positive pressure transnasally to patients can "cure" OSA, their compliance rates are low. There are other procedures that can help with OSA, such as a septoplasty or those that deal with the tongue base (e.g., geniohyoid advancement), but neither directly treats the most common site of obstruction. A tracheostomy is a curative treatment for OSA, but it completely bypasses all of the obstructive points without addressing them. A tracheostomy has much morbidity, including a measurable mortality rate, and is reserved for life-threatening cases of OSA. A sagittal split operation of the mandible is used for occlusion, not OSA.

2016 Which of the following surgeries addresses the most common anatomic site of obstruction in patients with obstructive sleep apnea? A ) Bilateral sagittal split osteotomy B ) Geniohyoid advancement C ) Septoplasty D ) Tracheostomy E ) Uvulopalatopharyngoplasty

The correct response is Option A. Aside from the third molars, the maxillary canines are typically the last teeth to erupt of the available options (around 11 to 12 years of age). It is important to give the tooth a large area of viable bone to traverse and become supported by, and alveolar bone grafting is typically performed well in advance of the eruption of the maxillary canine in order to preserve the native tooth. The maxillary central incisor is often the second permanent tooth a child will erupt, typically at age 7 to 8 years. The maxillary lateral incisor is often the third permanent tooth a child will erupt, typically at age 8 to 9 years. The maxillary first premolar is often the fourth permanent tooth a child will erupt, typically at age 10 to 12 years. The maxillary first molar is often the first permanent tooth a child will erupt, typically at age 6 to 7 years.

2016 Which of the following tooth types is most commonly the last one to erupt when the primary teeth are replaced by permanent teeth? A ) Maxillary canine B ) Maxillary central incisor C ) Maxillary first molar D ) Maxillary first premolar E ) Maxillary lateral incisor


Kaugnay na mga set ng pag-aaral

Ethics Chapter 4: Egoism, Altruism & the Social Contract

View Set

Pertumbuhan dan Perkembangan Tumbuhan

View Set

Chapter 4: Rational Choice Theory

View Set